You are on page 1of 126

17.

CO-ORDINATE GEOMETRY

Single Correct Answer Type

1. In ∆ ABC , a2 ( cos 2 B−cos 2 C ) +b 2 ( cos2 C−cos2 A ) +c 2 (cos 2 A−cos 2 B) is equal to

a) 0 b) 1 c) 2 2 2 d) 2 2 2
a + b +c 2(a + b +c )
2. If sin A :sin B: sinC=3 : 4 :5, then cos A : cos B is equal to

a) 4 :3 b) 5 :3 c) 3 :4 d) 3 :5

3. A B C
If A , B ,C are the angles of a triangle, then cot + cot + cot is equal to
2 2 2
c) ∆
2
a) s b) R d) s
2
R s s ∆
4. Coordinates of the foot of the perpendicular drawn from (0, 0) to the line joining ¿) and (a cos β , a sin β )
are

(2 2)
a) a , b
(2
b) a ( cos α +cos β ) , a (sin α +sin β)
2 )
( 2 2 )
c) cos α + β , sin α + β
( 2)
d) 0 , b

5. Three points are A ( 6 , 3 ) , B (−3 , 5 ) ,C (4 ,−2) and P ( x , y ) is a point, then the ratio of area of ∆ PBC and
∆ ABC is

|
a) x+ y−2
7 | b) x− y +2
| 2 | c) x− y−2
| 7 | d) None of these

6. Two vertical poles 20 m and 80 m stands apart on a horizontal plane. The height of the point of
intersection of the lines joining the top of each pole to the foot of the other is
a) 15 m b) 16 m c) 18 m d) 50 m

7. A person on a ship sailing north sees two lighthouses which are 6 km apart, in a line due west. After an
hour’s tailing one of them bears south west and the other southern south west. The ship is travelling at a
rate of
a) 12 km/hr b) 6 km/hr c) d) ( )
3 √ 2 km/hr 6+ 3 √2 km/hr
8. If α , β , γ are the real roots of the equation
3 2
x −3 p x +3 qx−1=0,
Then the centroid of the triangle whose vertices are

( α , α1 ) ,( β , 1β ) and ( γ , 1γ ), is
a) ( p , q) b) (q , p) c) (− p , q) d) (q ,− p)

9. If two vertices of a triangle are (−2 , 3) and (5 ,−1). Orthocentre lies at the origin and centroid on the line
x + y=7 , then the third vertex lies at
a) (7, 4) b) (8, 14) c) (12, 21) d) None of these

10. What is the equation of the locus of a point which moves such that 4 times its distance from the x -axis is
the square of its distance from the origin?
a) 2 2 b) 2 2 c) 2 2 d) 2 2
x + y −4 y=0 x + y −4∨ y ∨¿ 0 x + y −4 x=0 x + y −4∨x∨¿ 0
11. If a 2+ b2=c 2, then s ( s−a ) ( s−b ) (s−c) is equal to

Page|1
a) 2 2 b) 1 a2 b2 c) 1 a2 b2 d) 1 ab
a b
4 2 2
12. The harmonic conjugate of (4 ,−2) with respect to (2 ,−4) and (7 , 1) is

a) (−8 ,−14) b) (2 , 3) c) (−2 ,−3) d) (13 ,−5)

13. If O is the origin and P ( x 1 , y 1 ) ,Q (x2 , y 2) are two points, then OP . OQ sin ∠ POQ=¿ ¿

a) x x + y y b) x y + x y c) x y −x y d) None of these
1 2 1 2 1 2 2 1 | 1 2 2 1|
14. If ∆ ABC , if a=3 , b=4 , c=5 , then the value of sin 2 B is

a) 4 /5 b) 3/20 c) 24 /25 d) 1/50

15. From an aeroplane vertically over a straight horizontal road, the angles of depression of two consecutive
milestones on opposite sides of the aeroplane are observed to be α and β . The height of the aeroplane
above the road is
a) tan α +tan β b) tan α tan β c) cot α cot β d) None of these
tan α tan β tan α +tan β cot α + cot β
16. In ∆ ABC , if ∠ A=45 ° ,∠ B=75° , then a+ c √ 2 is equal to

a) 0 b) 1 c) b d) 2 b

17. Three vertical poles of heights h1 , h2 and h3 at the vertices A , B and C of a ∆ ABC subtend angles α , β∧γ
respectively at the circumcentre of the triangle. If cot α , cot β and cot γ are in AP, then h1 , h2 , h3 are in
a) AP b) GP c) HP d) None of these

18. The area enclosed within the curve |x|+| y|=1 is

a) 1 sq unit b) c) d) 2 sq units
2 √ 2 sq units √ 2 sq units
19. P is a point on the segment joining the feet of two vertical poles of height a and b .The angles of elevation
of the top of the poles from P are 45° each. Then, the squre of the distance between the top of the poles is
2 2
a) a +b b) 2 2 c) 2 2 d) 2 2
a +b 2(a + b ) 4 (a + b )
2
20. By rotating the coordinates axes through 30 ° in anticlockwise sense the equation x 2+ 2 √ 3 xy− y 2=2 a2
changes to
a) 2 2 2 b) 2 2 2 c) 2 2 2 d) None of these
X −Y =3 a X −Y =a X −Y =2 a
21. The x -coordinate of the incentre of the triangle where the mid points of the sides are (0, 1), (1, 1)and (1,
0) is
a) b) c) d)
2+ √ 2 1+ √ 2 2− √ 2 1− √ 2
22. Let A(2,−3) and B(−2 , 1) be vertices of a triangle ABC . If the centroid of this triangle moves on the line
2 x+3 y =1, then the locus of the vertex C is the line
a) 2 x+3 y =9 b) 2 x−3 y=7 c) 3 x+ 2 y =5 d) 3 x−2 y=3

23. The angle of elevation of the top of a tower at a point on the ground is 30 ° . If on walking 20 m toward the
tower the angle of elevation becomes 60 ° , then the height of the tower is
a) 10 m b) 10 c) d) None of these
m 10 √ 3 m
√3
24. 2 A
In a ∆ ABC , if 2 s=a+b+ c and ( s−b ) ( s−c ) =x sin , then the value of x is
2

Page|2
a) bc b) ca c) ab d) abc

25. If p1 , p2 denote the length of the perpendiculars from the origin on the lines x sec α + y cosec α=2 a and

( )
2
P1 P2
x cos α + y sin α ¿ a cos 2 α respectively, then , is equal to
P2 P1
a) 2 b) 2 c) 2 d) 2
4 sin 4 α 4 cos 4 α 4 cosec 4 α 4 sec 4 α
26. The equation ( x−2 )2 + ( y−1 )2 + ( x+ 2 )2+ ( y−4 )2=5 represents
√ √
a) Circle b) Ellipse c) Line segment d) None of these

27. 1 1 1 1
The value of 2
+ 2 + 2 + 2 is
r1 r2 r3 r
2 2 2 2 2 2 2
a) 0 b) a +b + c c) ∆ d) a +b + c
2 2 2 2
∆ a +b + c ∆
28. The sides of a triangle are 4cm, 5cm and 6cm. the area of the triangle is equal to

a) 15 c m 2 b) 15 √ 7 c m2 c) 4 d) None of these
√ 7 c m2
4 4 15
29. A vertical lamp-post, 6 m high, stands at a distance of 2 m from a wall, 4 m high. A 1.5 m tall man starts to
walk away from the wall on the other side of the wall, in line with the lamp-post the maximum distance to
which the man can walk remaining in the shadow is
a) 5 b) 3 c) 4 m d) None of these
m m
2 2
30. A tower subtends an angle α at a point A in the plane of its base and the angle of depression of the foot of
the tower at a point b feet just above A is β . Then, the height of the tower is
a) b tan α cot β b) b cot α tan β c) b cot α cot β d) b tan α tan β

31. In a ABC , if b=2 ,∠ B=30° , then the area of the circumcircle of ∆ ABC in square unit is

a) π b) 2 π c) 4 π d) 6 π

32. The base of a cliff is circular. From the extremities of a diameter of the base of angle of elevation of the top
of the cliff are 30 ° and 60 ° . If the height of the cliff be 500 m, then the diameter of the base of the cliff is
a) b) 2000 m c) 1000 m d) 2000 m
1000 √ 3 m
√3 √3 √2
33. 2
b −c
2
If R denotes circumradius, then in ∆ ABC , is equal to
2 aR
a) cos (B−C) b) sin(B−C) c) cos B−cos C d) None of these

34. The area between the curve y=1−¿ x∨¿ and the x -axis is equal to

a) 1 sq unit b) 1 sq unit c) 1 sq unit d) 2 sq units


2 3
35. Angles A , B and C of a triangle are in AP with common difference 15 degree, then angle A is equal to

a) 45 ° b) 60 ° c) 75 ° d) 30 °

( )( )
36. r1 r1
In a triangle 1− 1− =2 , then the triangle is
r2 r3
a) Right angled b) Equilateral c) Isosceles d) None of these

Page|3
37. The angle of elevation of the sun, if the length of the shadow of a tower is √ 3 times the height of the pole, is

a) 150 ° b) 30 ° c) 60 ° d) 45 °

38. If the equation 2 x 2+ y 2 −4 x−4 y=0 is transformed to the equation 2 X 2 +Y 2−8 X−8 Y +18=0 by
shifting the origin at a point P without rotating the coordinates axes, then the coordinates of P are
a) (1 , 2) b) (1 ,−2) c) (−1 , 2) d) (−1 ,−2)

39. A vertical pole PS has two marks Q and R such that the portions PQ , PR and PS subtend angles α , β , γ at
a point on the ground distance x from the pole. If PQ=a , PR=b , PS=c and α + β+ γ =180 ° then x 2 is
equal to
a) a b) b c) c d) abc
a+b+ c a+b+ c a+b+ c a+b+ c
40. If in a ∆ ABC , ( s−a )( s−b )=s (s−c), then angle C is equal to

a) 90 ° b) 45 ° c) 30 ° d) 75 °

41. 3
At a point on the ground the angle of elevation of a tower is such that its cotangent is . On walking 32 m
5
2
towards the tower the cotangent of the angle of elevation is . The height of the tower is
5
a) 160 m b) 120 m c) 64 m d) None of these

42. Area of quadrilateral whose vertices are (2, 3), (3, 4), (4, 5) and (5, 6), is equal to

a) 0 b) 4 c) 6 d) None of these

43. If the area of a triangle ABC is ∆ , then a 2 sin 2 B+ b2 sin 2 A is equal to

a) 3 ∆ b) 2 ∆ c) 4 ∆ d) −4 ∆

44. Consider the following statements :


sin A sin( A−B)
1. If in a ∆ ABC , = , then a 2 , b2 , c 2 are in AP
sin C sin( B−C)
2. If exradius r 1 , r 2 and r 3 of a ∆ ABC are in HP, then the sides a , b , c are in AP
Which of these is/are correct?
a) Only (1) b) Only (2) c) Both (1) and (2) d) None of these

45. If the sides of the triangle are p , q , √ p 2+ q2 + pq, then the greatest angle is

a) π b) 5 π c) 2 π d) 7 π
2 4 3 4
46. If x , y , z are perpendicular drawn from the vertices of triangle having sides a , b and c , then the value of
bx cy a z
+ + will be
c a b
2 2 2 2 2 2 2 2 2 2 2 2
a) a +b + c b) a +b + c c) a +b + c d) 2(a +b + c )
2R R 4R R
47. A balloon is observed simultaneously from three points A , B and C on a straight road directly under it.
The angular elevation at B is twice and at C is thrice that of A . If the distance between A and B is 200 m
and the distance between B and C is 100 m, then the height of balloon is given by
a) 50 m b) c) d) None of these
50 √ 3 m 50 √ 2m
48. If the distance of any point P from the points A(a+b , a−b) and B(a−b , a+ b) are equal, then the locus
of P is
Page|4
a) x− y =0 b) a x +by=0 c) bx−ay=0 d) x + y=0

49. The length of altitude through A of the ∆ ABC , where A ≡ (−3 , 0 ) , B≡ ( 4 ,−1 ) ,C ≡ ( 5 , 2 ), is

a) 2 b) 4 c) 11 d) 22
√10 √10 √10 √10
50. Triangle ABC has vertices (0, 0), (11, 60) and (91, 0). If the line y=kx cuts the triangle into two triangles
of equal area, then k is equal to
a) 30 b) 4 c) 7 d) 30
51 7 4 91
51. A pole stands at the centre of a rectangular field and it subtends angles of 15 ° and 45 ° at the mid points of
the side of the field. If the length of its diagonal is 1200 m, then the height of flag staff is
a) 400 m b) 200 m c) d)
300 √ 2+ √3 m 300 √ 2−√ 3 m
52. What is the equation of the locus a point which moves such that 4 times its distance from the x -axis is the
square of its distance from the origin?
a) 2 2 b) 2 2 | | c) 2 2 d) 2 2 ||
x − y −4 y=0 x + y −4 y =0 x + y −4 x=0 x + y −4 x =0
53. A person standing on the bank of a river, observe that the angle of elevation of the top of a tree on the
opposite bank of the river is 60 ° and when he retries 40m a way from the tree the angle of elevation
become 30° . The breadth of the river is
a) 20 m b) 30 m c) 40 m d) 60 m

54. There exist a ∆ ABC satisfying

a) tan A+ tan B+ tanC=0 b) sin A = sin B = sin C


2 3 1

c)
sin A+sin B=−
( √23+1
√2 )
cos A cos B
d)
( a+ b )2=c2 +ab and √ 2 ( sin A +cos A )=√ 3
¿ √ =sin A sin B
3
4
55. From a point a meters above a lake the angle of elevation of a cloud is α and the angle of depression of its
reflection is β . The height of the cloud is
a) a sin(α + β ) b) a sin(α + β ) c) a sin(β−α ) d) None of these
m m
sin( α + β) sin(β−α ) sin(α + β )
56. The orthocentre of the triangle formed by ( 0 , 0 ) , ( 8 , 0 ) ,(4 , 6) is

( 3)
a) 4 , 8 b) (3 , 4) c) (4 , 3) d) (−3 , 4 )

57. The x -coordinate of the incentre of the triangle where the mid point of the sides are (0, 1), (1, 1) and (1,
0), is
a) b) c) d)
2+ √ 2 1+ √ 2 2− √ 2 1− √ 2
58. The locus of the point (x , y ) which is equidistant from the points (a+ b , b−a) and (a−b , a+ b) is

a) ax=by b) ax +by =0 c) bx +ay =0 d) bx−ay=0

59. If the sum of the distances from two perpendicular lines in a plane is 1, then its locus is

a) A square b) A circle

c) A straight line d) Two intersecting lines

60. A tower of x metres high, has a flagstaff at its top. The tower and the flagstaff subtend equal angles at a
Page|5
point distant y metres from the foot of the tower. Then the length of the flagstaff (in meters), is
2 2 2 2 2 2 2 2
a) y (x − y ) b) x( y + x ) c) x( x + y ) d) x( x − y )
(x 2 + y 2) ( y 2−x 2 ) ( x 2− y 2 ) ( x2 + y 2 )
61. A−B+C
In a ∆ ABC , 2 ac sin is equal to
2
a) 2 2 2 b) 2 2 2 c) 2 2 2 d) 2 2 2
a + b −c c +a −b b −a −c c −a −b
62. If P= (1 , 0 ) , Q=(−1 ,0) and R=(2 , 0) are three given points, then the locus of the point S ( x , y ) satisfying
the relation S Q 2+ +S R 2=2 S P2 is
a) A straight line parallel to x -axis b) A circle through the origin

c) A circle with centre at the origin d) A straight line parallel to y -axis

63. If orthocenter and circumcentre of a triangle are respectively (1, 1) and (3, 2), then the coordinates of its
centroid are
a) 7 5
(3 , 3) b) 5 7
(3 , 3) c) (7, 5) d) None of these

64. The locus of the point of intersection of the lines x cotθ + y cosecθ=2 and x cosecθ + y cotθ=6 is

a) A straight line b) Circle c) A hyperbola d) An ellipse

65. In ∆ ABC , if cot A , cot B , cot C be in AP, then a 2 , b2 , c 2 are in

a) HP b) GP c) AP d) None of these

66. The angels of elevation of the cloud at a point 2500 m high from the lake is 15 ° and the angle of depression
of its reflection to the lake is 45° . Then the height of cloud from the foot of lake is
a) b) 2500 m c) d) None of these
2500 √ 3 m 500 √ 3 m
67. ABC is a triangular park with AB= AC=¿100 m. A clock tower is situated at the mid point of BC . The
angle of elevation, if the top of the toper at A and B are cot−1 3.2and cose c−1 2.6respectively. The height of
the tower is
a) 16 m b) 25 m c) 50 m d) None of these

68. In ∆ ABC , b= √ 3 , c=1 and ∠ A=30 °, then the largest angle of the triangle is

a) 60 ° b) 135 ° c) 90 ° d) 120 °

69. In an equilateral triangle, R :r :r 1 is equal to

a) 1:1:1 b) 1:2:3 c) 2:1:3 d) 3:2:4

70. a b c
In a triangle, if r 1=2 r 2=3r 3, then + + is equal to
b c a
a) 75 b) 155 c) 176 d) 191
60 60 60 60
71. In a triangle ABC , a: b :c=4 :5 :6. The ratio of the radius of the circumcircle to that of the incircle is

a) 15 b) 11 c) 16 d) 16
4 5 7 3
72. An aeroplane flying with uniform speed horizontally one kilometer above the ground is observed at an
elevation of 60 ° . After 10 s, if the elevation is observed to be 30 ° , then the speed of the plane (in km/h) is
a) 240 b) c) d) 120
200 √ 3 240 √ 3
√3 √3
Page|6
73. The angle of elevation of the top of a tower standing on a horizontal plane from a point A is α . After
walking a distance a towards the foot of the tower the angle of elevation is found to be β . The height of the
tower is
a) a sin α sin β b) a sin α sin β c) a sin(β−α ) d) a sin(α− β)
sin(β−α ) sin(α− β) sin α sin β sin α sin β
74. If the vertices of a triangle have integral coordinates, the triangle cannot be

a) An equilateral triangle b) A right angled triangle

c) An isosceles triangle d) None of the above

75. In a ∆ ABC , among the following which one is true?

()
a) ( b+ c ) cos A =a sin B+ C
2 2 ( 2) 2
b) ( b+ c ) cos B+ C =a sin A

( 2 ) (2)
c) ( b−c ) cos B−C =a cos A
( 2 )
d) ( b−c ) cos A =a sin B−C
2

The upper ( ) th portion of a vertical pole subtends an angle tan ( ) at a point in the horizontal plane
76. 3 3 −1
4 5
through its foot and at a distance 40 m from the foot. A possible height of the vertical pole is
a) 20 m b) 40 m c) 60 m d) 80 m

77. If C and D are the points of internal and external division of line segment AB in the same ratio, then
AC , AB , AD are in
a) AP b) GP c) HP d) AGP

78. A ladder rests against a vertical wall at angle α to the horizontal. If its foot is pulled away from the wall
through a distance ' a ' so that it slides a distance ' b ' down the wall making an angle β with the horizontal,
then a=¿
a) b tan
α −β
2 ( ) b)
b tan
2
α+ β
( ) c)
b cot
2
α −β
( ) d) None of these

79. The angles A , B and C of a ∆ ABC are in A.P. If AB=6 , BC=7 , then AC=¿

a) 5 b) 7 c) 8 d) None of these

80. The locus of a point whose difference of distance from points (3, 0) and (−3 , 0) is 4, is
2 2 2 2 2 2 2 2
a) x y b) x y c) x y d) x y
=1 − − =1 − =1 − =1
4 5 5 4 2 3 3 2
81. If a ∆ ABC , if a 4 +b 4+ c 4=2 c 2 (a 2+ b2 ), then ∠ C is equal to

a) 60 ° b) 135 ° c) 90 ° d) 75 °

82. Given the points A(0 , 4) and B(0 ,−4) , then the equation of the locus of the point P(x , y) such that,
| AP−BP|=6 , is
2 2 2 2 2 2 2 2
a) x y b) x y c) x y d) y x
+ =1 + =1 − =1 − =1
7 9 9 7 7 9 9 7
83. A C B
If in ∆ ABC , sin sin =sin and 2 s is the perimeter of the triangle, then s is
2 2 2
a) 2 b b) b c) 3 b d) 4 b

84. The angle of depression of a ship from the top of a tower 30 m high is 60 ° .Then the distance of ship from
the base of tower is
Page|7
a) 30 m b) c) d) 10 m
30 √ 3 m 10 √ 3 m
85. At a distance 2 h m from the foot of a tower of height h m the top of the tower and a pole at the top of the
tower subtend equal angles. Height of the pole should be
a) 5 h m b) 4 h m c) 7 h m d) 3 h m
3 3 5 2
86. From the tower 60 m high angles of depression of the top and bottom of a house are α and β respectively.
60 sin( β−α )
If the height of the house is , then x is equal to
x
a) sin α sin β b) cos α cos β c) sin α cos β d) cos α sin β

87. In a triangle, the lengths of the two larger sides are 10 cm and 9 cm respectively. If the angles of the
triangle are in AP, then the length of the third side in cm can be
a) b)
5−√ 6
only 5+ √ 6
only
c) d) Neither
5−√ 6 or 5+ √ 6 5−√ 6 nor 5+ √ 6
88. 2 A 2 B 2C
In ∆ ABC , if sin , sin , sin be in HP. Then, a , b , c will be in
2 2 2
a) AP b) GP c) HP d) None of these

89. The angles of elevation of the top of a tower at the top and the foot of a pole of height 10 m are 30 ° and
60 ° respectively. The height of the tower is
a) 10 m b) 15 m c) 20 m d) None of these

90. If the points ( 1 ,1 ) , (−1 ,−1 ) ,(− √ 3 , √ 3) are the vertices of a triangle, then this triangle is

a) Right angled b) Isosceles c) Equilateral d) None of these

91. The vertices of a family of triangles have integer coordinates. If two of the vertices of all the triangles are
(0, 0) and (6, 8), then the least value of areas of the triangles is
a) 1 b) 3 c) 2 d) 5
2 2
92.
In a ∆ ABC , cot( A
2
+cot
B
2 )( 2 B
a sin +b sin
2
2 A
2 )
is equal to

a) cot C b) c cot C c) cot C d) c cot C


2 2
93. The intercepts on the straight line y=mx by the line y=2 and y=6 is less than 5, then m belongs to

a) ¿− 4 , 4 ¿ b) ¿ 4 , 3 ¿ c) ¿−∞ ,− 4 [ ∪ ] 4 , ∞ ¿ d) ¿ 4 , ∞ ¿
3 3 3 8 3 3 3
94. In ∆ ABC , ( b−c ) sin A+ ( c−a ) sin B+ ( a−b ) sin C is equal to

a) ab+ bc+ ca b) 2 2 2 c) 0 d) None of these


a + b +c
95. The inradius of the triangle whose sides are 3, 5, 6 is

96.
a)
√ 8
7
b)
√8 c)

B
√7
B
d) 7
√ 8
In a ∆ ABC , if the sides are a=3 , b=5 and c=4 , then sin +cos is equal to
2 2
a) b) √ 3+ 1 c) √ 3−1 d) 1
√2 2 2
97. The elevation of an object on a hill is observed from a certain point in the horizontal plane through its
Page|8
base, to be 30 ° . After walking 120 m towards it on level ground the elevation is found to be 60 ° . Then the
height of the object (in metres) is
a) 120 b) c) d) 60
60 √ 3 120 √ 3
98. If the area of the triangle with vertices ( x ,0 ) ,(1 ,1) and (0, 2) is 4 sq unit, then the value of x is

a) −2 b) −4 c) −6 d) 8

99. At a distance 12 metres from the foot A of a tower AB of height 5 metres, a flagstaff BC on top of AB and
the tower subtend the same angle. The, the height of flagstaff is
a) 1440 b) 475 c) 845 d) None of these
metres metres metres
119 119 119
100. A tower 50 m high, stands on top of a mount, from a point on the ground the angles of elevation of the top
and bottom of the tower are found to be 75 ° and 60 ° respectively. the height of the mount is
a) 25 m b) ( ) c) d) ( )
25 √ 3−1 m 25 √ 3 m 25 √ 3+1 m
101. Let AB is divided internally and externally at Pand Q in the same ratio. Then, AP, AB , AQ are in

a) AP b) GP c) HP d) None of these

102. If the sum of the distance of a point P from two perpendicular lines in a plane is 1, then the locus of P is a

a) Rhombus b) Circle c) Straight line d) Pair of straight lines

103. A flagpole stands on a building of height 450 ft and an observer on a level ground is 300 ft from the base of
the building. The angle of elevation of the bottom of the flagpole is 30 ° and the height of the flagpole is
50ft. If θ is the angle of elevation of the top of the flagpole, then tan θ is equal to
a) 4 b) √ 3 c) 9 d) √ 3
3 √3 2 2 5
104.
(
In ∆ ABC , 2 a sin
2C
2
+c sin 2
A
2 )
is equal to

a) a+ b−c b) c +a−b c) b+ c−a d) a+ b+c

105. Orthocenter of triangle with vertices (0, 0), (3, 4) and (4, 0) is

( 4)
a) 3 , 5 b) (3, 12)
( 4)
c) 3 , 3 d) (3, 9)

106. Three vertices of a parallelogram taken in order are (−1 ,−6),(2 ,−5) and (7 , 2). The fourth vertex is

a) (1, 4) b) (4, 1) c) (1, 1) d) (4, 4)

107. If in a ∆ ABC , cos A cos B+sin A sin B sinC=1, then the triangle is

a) Isosceles b) Right angled c) Isosceles right angled d) Equilateral

108. If in a ∆ ABC , the sides AB and AC are perpendicular, then the true equation is

a) tan A+ tan B=0 b) tan B+ tan C=0 c) tan A+ 2 tan C=0 d) tan B tan C=1

109. The points ( 1 ,1 ) ,(−5 ,5) and (13 , λ) lie on the same straight line, if λ is equal to

a) 7 b) −7 c) ± 7 d) 0

110. Circumcentre of triangle whose vertices are (0, 0), (3, 0) and (0, 4) is

Page|9
(2 )
a) 3 , 2
( 2)
b) 2 , 3 c) (0, 0) d) None of these

111. The vertices of a triangle are A(−1,−7), B(5 , 1) and C ( 1 , 4 ). The equation of the bisector of angle ABC ,
is
a) x +7 y−2=0 b) x−7 y −2=0 c) x−7 y +2=0 d) None of these

112. A tower subtends angles α ,2 α ∧3 α respectively at points A , B∧C , all lying on a horizontal line through
AB
the foot of the tower, then is equal to
BC
a) sin 3 α b) 1+2 cos 2 α c) 2 cos 2 α d) sin 2 α
sin 2 α sin α
113. A person standing on the bank of a river finds that the angle of elevation of the top of a tower on the
opposite bank is 45 ° , then which of the following statements is correct?
a) Breadth of the river is twice the height of the tower

b) Breadth of the river and the height of the tower are the same

c) Breadth of the river is half of the height of the tower

d) None of these

114. The angular depression of the top and the foot of the chimney as seen from the top of a second chimney
which is 150 m high and standing on the same level as the first are θ and ϕ respectively. The distance
4 5
between their tops when tanθ= and tan ϕ= is equal to
3 2
a) 50 m b) 100 m c) 15 m d) None of these

115. A round balloon of radius r subtends an angle α at the eye of the observer, While the angle of elevation of
its centre is β . The height of the center of balloon is
a) r cosec α sin β b) r sin α cosec β c) r sin α cosec β d) r cosec α sin β
2 2 2 2
116. In a ∆ ABC , a , c , A are given and b 1 , b2 are two values, if the third side b such that b 2=2 b1, then sin A is
equal to
a) √ 9 a −c
√ c) √ 9 a +c
2 2 2 2 2 2 d) None of these
b) 9 a −c
8 a2 8c
2
8 a2
117. If a , b , c are sides of a triangle, then

a) b) |
√ a+ √b> √ c √ a−√ b|> √c (if c is smallest)
c) d) None of the above
√ a+ √b< √ c
118. ABC Is a triangle with ∠ A=30 ° , BC=10 cm. The area of the circumcircle of the triangle is

a) 100 π sq cm b) 5 sq cm c) 25 sq cm d) 100 π sq cm
3
119. In a ∆ ABC , a :b :c=4 :5 :6 . The ratio of the radius of the circumcircle to that of the incircle is

a) 16 b) 16 c) 11 d) 7
9 7 7 16
120. The incentre of the triangle formed by lines x=0 , y=0and 3 x+ 4 y =12, is at

(2 2)
a) 1 , 1 b) (1 , 1)
( 2)
c) 1 , 1
(2 )
d) 1 , 1

P a g e | 10
121. Given points are A ( 0 , 4 ) and B ( 0 ,−4 ) , the locus of P(x , y) such that | AP−BP|=6 , is

a) 2 2 b) 2 2 c) 2 2 d) None of these
9 x −7 y +63=0 9 x + 7 y −63=0 9 x + 7 y + 63=0
122. The angle of elevation of the top of a tower from a point A due South of the tower is α and from a point B
due East of the tower is β . If AB=d , then the height of the tower is
a) d b) d c) d d) d
√ tan 2
α−tan β 2
√ tan 2 2
α + tan β √ cot 2
α +cot β2
√ cot 2
α−cot2 β
123. Let P be the point (1, 0) and Q be the point on y 2=8 x . The locus of mid point of P Q is

a) 2 b) 2 c) 2 d) 2
x −4 y +2=0 x + 4 y +2=0 y +4 x+2=0 y −4 x +2=0
124. Let A(k ,2) and B(3 , 5) are points. The point (t ,t ) divide AB from A ’s side in the ratio of k , then
k =… , k ∈ R−{0 ,−1}
a) −4 b) −2 c) 4 d) 2

125. If a , b , c the sides of a ∆ ABC are in AP and a is the smallest side, then cos A equals

a) 3 c−4 b b) 3 c−4 b c) 4 c−3 b d) None of these


2c 2b 2c
126. Area of the triangle formed by the lines y=2 x , y=3 x and y=5 is equal to (in square unit)

a) 25 b) 25 c) 5 d) 17
6 12 6 12
127. The angles of depression of the top and the foot of a chimney as seen from the top of a second chimney,
which is 150 m high and standing on the same level as the first are θ and ϕ respectively, then the distance
4 5
between their tops when tanθ= and tan ϕ= , is
3 2
a) 150 m b) c) 150 m d) 100 m
100 √ 3 m
√3
128. If one side of a triangle is double the other and the angles opposite to these sides differ by 60 ° , then the
triangle is
a) Obtuse angled b) Acute angled c) Isosceles d) Right angled

129. If the three points ( 3 q , 0 ) , ( 0 , 3 p ) and( 1 ,1 ) are collinear then which one is true?

a) 1 + 1 =0 b) 1 + 1 =1 c) 1 + 1 =3 d) 1 + 3 =1
p q p q p q p q
130. C
If in a ∆ ABC , a=15 ,b=36 , c=39 , then sin is equal to
2
a) √3 b) 1 c) 1 d) −1
2 2 √2 √2
131. π
In a ∆ ABC , let ∠C= , if r is the inradius and R is the circumradius of the ∆ ABC , then 2(r + R) equals
2
a) c +a b) a+ b+c c) a+ b d) b+ c

132. From the top of a light house 60 m high with its base at the sea level the angle of depression of a boat is
15 ° . The distance of the boat from the foot of light house is

( √ 3+1 ) 60 m ( √ 3−1 ) 60 m
a) √ 3−1 b) √ 3+1 c) √ 3+1 d) None of these
m
√3−1
133. If cos 2 A+ cos2 C=sin2 B , then ∆ ABC is

P a g e | 11
a) Equilateral b) Right angled c) Isosceles d) None of these

134. The sides of triangle are in the ratio 1 : √ 3 :2 , then the angles of the triangle are in ratio

a) 1 :3: 5 b) 2 :3 :1 c) 3 :2 :1 d) 1 :2:3

135. A tower stands at the top of a hill whose height is 3 times the height of the tower. The tower is found to
1
subtend at a point 3 km away on the horizontal through the foot of the hill, an angle θ , where tanθ=¿ ¿.
9
The height of the tower is
a) 12 b) 3 c) 9 ± √ 33 d) None of these
8
136. b √3
Angles A , B and C of a ∆ ABC are in AP. If = , then angle A is equal to
c √2
a) π b) π c) 5 π d) π
6 4 12 2
137. The angle of depression of a boat in a river is 30 ° from the top of a tower, 87 m high and the speed of the
boat is 5.8 km/h. The time taken by the boat to reach at the base of the tower is
a) 9 min b) 9 √ 3 min
c) 25 min d) 15 min
10
138. If the centroid of the triangle formed by the points ( a , b ) ,(b , c) and (c , a) is at the origin, then
3 3 3
a + b +c =¿
a) 0 b) abc c) 3 abc d) −3 abc

139. The sides of a ∆ ABC are BC=5 , CA=4 and AB=3. If A is at the origin and the bisector of the internal
angle A meets BC in D(12/7 , 12/7), then the coordinates of the incentre, are
a) (2 , 2) b) (2 , 3) c) (3 , 2) d) (1 , 1)

140. If a , b and c are the sides of a triangle such that a 4 +b 4 +c 4 =2 c2 ( a2 +b2 ), then the angles opposite to the
side C is
a) 45 ° ∨90 ° b) 30 °∨135 ° c) 45 ° ∨135° d) 60 °∨120 °

141. In radius of a circle which is inscribed in a isosceles triangle one of whose angle is 2 π /3, is √ 3, then area of
triangle is
a) b) c) d) None of these
4 √3 12−7 √ 3 12+7 √ 3
142. A triangular park is enclosed on two sides by a fence and on the third side by a straight river bank. The
two sides having fence are of same length x . The maximum area enclosed by the park is

143.
a)
√ x3
8
A 5
2
C 2
b) 1 x 2 c)
πx
2 d) 3 x 2
2

In a ∆ ABC , if tan = , tan = , then


2 6 2 5
a) a , b , c are in AP b) a , b , c are in GP c) b . a . c are in AP d) a , b , c are in AP

144. The vertices P , Q, R of a triangle are (2, 1), (5, 2) and (3, 4) respectively. Then, the circumcentre is

(4
a) 13 ,− 9
4) (
b) −13 , 9
4 4) (
c) −13 ,− 9
4 4) ( 4 4)
d) 13 , 9

145. In a ∆ ABC , ( a+b+ c )( b +c−a )=kbc , if

a) k < 0 b) k > 6 c) 0< k <4 d) k > 4

P a g e | 12
146. If A(6 ,−3), B(−3 ,5) ,C (4 ,−2), P ( α , β ) , then the ratio of the areas of the triangles PBC , ABC is

a) ¿ α + β∨¿ b) ¿ α −β∨¿ c) ¿ α + β+ 2∨¿ d) ¿ α + β−2∨¿

147. ABC is a triangular park with AB= AC=100 m. A clock tower is situated at the mid point of BC . The
angles of elevation of the top of the tower at A and B are cot−1 3.2 and cose c−1 2.6 respectively. The
height of the tower is
a) 50 m b) 25 m c) 40 m d) None of these

148. In a ∆ ABC , a cot A+b cot B+c cot C is equal to

a) r + R b) r −R c) 2(r + R) d) 2(r −R)

149. If (1 , a),(2 ,b) , and ( 3 , c ) ; a , b , c ∈ R are the vertices of a triangle, its centroid can

a) Not be on x -axis b) Not be on y -axis c) Be on (0, 0) d) None of these

150. The pair of lines √ 3 x2 −4 xy + √ 3 y 2=0 are rotated about the origin by π /6 in the anti-clockwise sense.
The equation of the pair in the new position is
a) b) c) d)
√ 3 y −xy=0
2 2
√ 3 x −xy=0 2
x − y =0
2
√ 3 x2 + xy =0
151. 2
In triangle ABC , a=2 , b=3 and sin A= then B is equal to
3
a) 30 ° b) 60 ° c) 90 ° d) 120 °

152. If the sides of a right angle triangle form an AP, the 'sin' of the acute angles are

(5 5)
a) 3 , 4 b)
( √3 ,
1
√3 ) c)
(√ √ √ √ )
5−1
2
,
5−1
2
d)
(√ √ √ √ )
3−1
2
,
3−1
2
153. In a ∆ ABC , 2 a2+ 4 b2+ c 2=4 ab+2 ac , then cos B is equal to

a) 0 b) 1 c) 3 d) 7
8 8 8
154. The line joining A ( b cos α , b sin α )and B ( a cos β , a sin β ) is produced to the point M ( x , y )so that
AM : MB=b :a , then
x cos
2( )
α +β
+ y sin
α+β
2( ) is

a) −1 b) 0 c) 1 d) 2 2
a +b
155. A house of height 100 m subtends a right angle at the window of an opposite house. If the height of the
window be 64m, then the distance between the two houses is
a) 48 m b) 36 m c) 54 m d) 72 m

156. A vertical tower stands on a declivity which is inclined at 15 ° to the horizon. From the foot of the tower a
man ascends the declivity for 80 ft and them, finds that the tower subtends an angle of 30 ° . The height of
tower is
a) ( ) b) ( ) c) ( ) d) None of these
20 √ 6−√2 ft 40 √6−√ 2 ft 40 √6+ √2 ft
157. (0 ,−1) And (0, 3) are two opposite vertices of a square. The other two vertices are

a) ( 0 , 1 ) ,(0 ,−3) b) ( 3 ,−1 ) ,(0 ,0) c) ( 2 , 1 ) ,(−2 ,1) d) ( 2 , 2 ) , (1 ,1)

158. The points (1, 3) and (5, 1) are two opposite vertices of a rectangle. The other two vertices lie on the line
y=2 x +c ,are

P a g e | 13
a) (2, 0) and (4, 4) b) (2, 0) and (−4 ,−4) c) (2, 0) and (−4 , 4) d) (−2 , 0) and (4, 4)

159. If in a ∆ ABC , r 3=r 1 +r 2 +r , then ∠ A+∠ B is equal to

a) 120 ° b) 100 ° c) 90 ° d) 80 °

160. In a triangle ABC ,if a=3 , b=4 , c=5, then the distance between its incentre and circumcentre is

a) 1 b) √ 3 c) 3 d) √ 5
2 2 2 2
161. One side of length 3 a of triangle of area a square unit lies on the line x=a . Then, one of the lines on which
2

the third vertex lies, is


a) 2 b) 2 c) x=−a d) x= a
x=−a x=a
3
162. In a ∆ ABC , if D is the middle point BC and AD is perpendicular to AC , then cos B is equal to

b) −b
2 2 2 2
a) 2b c) b +c d) c + a
a a ca ca
163. The angle of depression of a point situated at a distance of 70 metres from the base of a tower is 45 ° . The
height of the tower is
a) 70 m b) c) 70 d) 35 m
70 √ 2m m
√2
164. The sides of a triangle are three consecutive natural numbers and its largest angle is twice the smallest
one. Then, the sides of the triangle are
a) 1, 2, 3 b) 2, 3, 4 c) 3, 4, 5 d) 4, 5, 6

165. Consider the following statements :


2 2
b −c
1. =2 R
a sin(B−C)
2. a sin ( B−C ) +b sin ( C− A )+ c sin ( A−B )=0
Which of these is/are correct?
a) Only (1) b) Only (2) c) Both (1) and (2) d) None of these

166. The four distinct point ( 0 , 0 ) , ( 2 ,0 ) , ( 0 ,−2 )and ( k ,−2 ) are concyclic, if k is equal to

a) −2 b) 2 c) 1 d) 0

167. If origin is shifted to (7 ,−4) , then point (4, 5) shifted to

a) (−3 , 9) b) (3, 9) c) (11, 1) d) None of these

168.
In ∆ ABC ,(a+b+ c) tan ( A
2
+ tan
B
2 )
is equal to

a) 2 c cot C b) 2 a cot A c) 2 b cot B d) tan C


2 2 2 2
169. 2 2 1 8
a
In a ∆ ABC , sides a , b , c are in AP and + + = , then the maximum value of
1! 9 ! 3 ! 7 ! 5 ! 5 ! (2 b)!
tan A tan B is equal to
a) 1 b) 1 c) 1 d) 1
2 3 4 4
170. If the angle of elevation of two towers from the middle point of the line joining their feet be 60 ° and 30 °
respectively, then the ratio of their heights is

P a g e | 14
a) 2 :1 b) c) 3 :1 d)
1 : √2 1 : √3
171. 1 1
In a ∆ ABC , ∠ C=60 °then + is equal to
a+c b+c
a) 1 b) 2 c) 3 d) None of these
a+b+ c a+b+ c a+b+ c
172. In ∆ ABC , if ( a+b +c ) ( a−b+c )=3 ac , then

a) ∠ B=60 ° b) ∠ B=30 ° c) ∠ C=60 ° d) ∠ A+∠ C=90°

173. If a 2 , b2 , c 2 are in AP, then which of the following are also in AP?

a) sin A , sin B , sin C b) tan A , tan B , tan C c) cot A , cot B ,cot C d) None of these

174. ab
In a triangle ABC , if sin A sin B= 2 , then the triangle is
c
a) Equilateral b) Isosceles c) Right angled d) Obtuse angled

175. The perimeter of a ∆ ABC is 6 times the arithmetic mean of the sine ratios of its angles. If a=1, then A is
equal to
a) π b) π c) π d) 2 π
6 3 2 3
176. The centriod of the triangle ABC , where A ≡ ( 2 , 3 ) , B ≡ ( 8 ,10 ) and C ≡(5 , 5) is

a) (5, 6) b) (6, 5) c) (6, 6) d) (15, 18)

177. The angle of elevation of the top of the tower observed from each of the tree point A , B ,C on the ground
forming a triangle is the same angle α . If R is the circumaradias of the triangle ABC , then the height of the
tower is
a) R sin α b) R cos α c) R cot α d) R tan α

178. The angle of elevation of the top of a hill from a point is α . After walking b metres towards the top up a
slope inclined at an angle β to the horizon, the angle of elevation of the top becomes γ . Then, the height of
the hill is
a) b sin α sin (γ −β) b) b sin α sin (γ −α ) c) b sin(γ −β) d) sin(γ −β)
sin(γ −α ) sin(γ −β) sin( γ−α ) b sin α sin (γ −α )
179. The area of the ∆ ABC , in which a=1 , b=2 , ∠ C=60 °, is

a) 4 sq unit b) 1 sq unit c) √3 sq unit d)


2 2
√ 3 sq units
180. If t 1 , t 2 and t 3 are distinct points ( t ,2 a t + a t 3 ) ,(t , 2 a t +a t 3) and t 3 , 2 a t 3+ a t 33 ¿ are collinear, if
1 1 1 2 2 2

a) t t t =1 b) t + t +t =t t t c) t + t +t =0 d) t + t +t =−1
1 2 3 1 2 3 1 2 3 1 2 3 1 2 3

181. If A and B are two points having coordinates (3, 4) and (5 ,−2) respectively and P is a point such that
PA=PB and area of triangle PAB=10 sq unit, then the coordinates of P are
a) (7, 4) and (13, 2) b) (7, 2) and (1, 0) c) (2, 7) and (4, 13) d) None of these

182. π R
In ∆ ABC , ∠ A= , b=4 , c=3, then the value of is equal to
2 r
a) 5 b) 7 c) 9 d) 35
2 2 2 24
183. In the angles A , B and 𝐶 of a triangular are in the arithmetic progression and if a , b and c denotes the
lengths of the sides opposite to A , B and 𝐶 respectively, then the value of the expression

P a g e | 15
a c
sin 2 C+ sin 2 A is
c a
a) 1 b) √ 3 c) 1 d)
2 2
√3
184. Two sides of a triangle are given by the roots of the equation x 2−5 x+ 6=0 and the angle between the
π
sides is . Then, the perimeter of the triangle is
3
a) b) c) d)
5+ √ 2 5+ √ 3 5+ √ 5 5+ √ 7
185. In a triangle ABC ,if ∠ A=60 ° , a=5 , b=4 , then c is a root of the equation

a) 2 b) 2 c) 2 d) 2
c −5 c−9=0 c −4 c−9=0 c −10 c +25=0 c −5 c−41=0
186. The angle of elevation of the top of vertical tower from a point A on the horizontal ground is found to be
π π
. From A , a man walks 10 m up a path sloping at a angle . After this the slope becomes steeper and
4 6
after walking up another 10 m, the man reaches the top of the tower. Distance of A from the foot of the
tower is
a) b) 5 ( 1+ √ 3 ) m c) d) 5 ( √ 3−1 ) m
5 ( 1+ √ 3 ) m 5 ( √ 3−1 ) m
2 2
187. If the distance between the points (a cos θ , a sin θ) and (a cos ϕ , a sin ϕ) is 2 a, then θ is equal to

a) 2 nπ ± π + ϕ , n ∈ Z b) nπ + π + ϕ , n ∈ Z
2
c) nπ −ϕ , n∈ Z d) 2 nπ +ϕ , n ∈ Z

188. If A ( 0 , 0 ) , B ( 12 , 0 ) ,C ( 12, 2 ) , D(6 , 7) and E(0 ,5) are the vertices of the pendagon ABCDE , then its area
in square units, is
a) 58 b) 60 c) 61 d) 63

189. A flag is standing vertically on a tower of height b . On a point at a distance a from the foot of the tower, the
flag and the tower subtend equal angles. The height of the flag is
2 2 2 2 2 2 2 2
a) b . a +b b) a . a −b c) b . a −b d) a . a +b
2 2 2 2 2 2 2 2
a −b a +b a +b a −b
190. A kite is flying at an inclination of 60 ° with the horizontal. If the length of the thread is 120 m, then the
height of the kite is
a) b) 60 m c) 60 m d) 120 m
60 √ 3 m
√3
191. a cos A+ b cos B+c cos C
is equal to
a+b+ c
a) 1/r b) r / R c) R/r d) 1/ R

192. AB is a vertical pole. The end A is on the level ground. C is the middle point of AB. P is a point on the
level ground. The portion BC subtends an angle β at P . If AP=n AB, then tan β =¿ ¿
a) n b) n c) n d) None of these
2 2 2
2n +1 n −1 n +1
193. If P(3 , 7) is a point on the line joining A(1 , 1) and B ( 6 ,16 ) , then the harmonic conjugate Q of point P
has the coordinates
a) (9 , 29) b) (−9 , 29) c) (9 ,−29) d) (−9 ,−29)

194. The angles of a triangle are in the ratio 3:5:10. Then, the ratio of the smallest side to the greatest side is

P a g e | 16
a) 1 :sin 10° b) 1 :2sin 10 ° c) 1 :cos 10 ° d) 1 :2 cos 10°

| |
195. 1 a b
In ∆ ABC , if 1 c a =0, then
1 b c
2 2 2
sin A +sin B+sin C is equal to
a) 4 b) 9 c) d) 1
3 √3
9 4
196. From a station A due West of a tower the angle of elevation of the top of the tower is seen to be 45 ° . From
a station B, 10 m from A and in the direction 45 ° South of East of angle of elevation is 30 ° , the height of
tower is
a) 5 2( 5+1)m b) 5( √ 5+1) m c) 5 √ 2( √ 5+1) m d) None of these
√ √
2 2
197. A straight line with negative slope passing through the point (1, 4) meets the coordinate axes at A and B.
The minimum value of OA +OB is equal to
a) 5 b) 6 c) 9 d) 8

198. 1°
An observer finds that the elevation of the top of a tower is 22 and after walking 150 metres towards
2

the foot of the tower he finds that the elevation of the top has increased to 67 . The height of the tower
2
in metres is
a) 50 b) 75 c) 125 d) 175

199. In an isosceles ∆ ABC , AB=AC . If vertical angle A is 20 ° , then a 3+ b3 is equal to

a) 2 b) 2 c) 2 d) abc
3a b 3b c 3c a
200. In a ∆ ABC , a ( cos 2 B+cos 2 C ) +cos A (c cos C+ b cos B) is equal to

a) a b) b c) c d) a+ b+c

201. ABC Is a triangle with vertices A (−1 , 4 ) , B(6 ,−2) andC (−2 , 4). D , E And F are the points which divide
each AB , BC and CA respectively in the ratio 3:1 internally. Then, the centroid of the triangle ¿ is
a) (3, 6) b) (1, 2) c) (4, 8) d) (−3 , 6)

202. 1 1 3
If in a ∆ ABC , + = , then the value of the angle C is
a+c b+ c a+ b+c
a) 60 ° b) 30 ° c) 45 ° d) None of these

203. A tower subtends an angle of 30 ° at a point distance d from the foot of the tower and on the same level as
the foot of the tower. At a second point, h vertically above the first, the angle of depression of the foot of
the tower is 60 ° . The height of the tower is
a) h b) h c) 3 h d) 3 h
3 3d d
204. Points D,E are taken on the side BC of the∆ ABC , such that BD=DE=EC . If
sin ( x+ y ) sin( y+ z)
∠ BAD=x , ∠ DAE= y ,∠ EAC =z , then the value of is equal to
sin x sin z
a) 1 b) 2 c) 4 d) None of these

205. sin A :sin C=sin( A−B):sin (B−C), then a 2 , b2 , c 2 are in

P a g e | 17
a) AP b) GP c) HP d) None of these

206. The point on the line 3 x+ 4 y =5, which is equidistant from (1, 2) and (3, 4) is

a) (7 ,−4) b) (15 ,−10)


(7 7)
c) 1 , 8
( 4)
d) 0 , 5

207. If A and B are two fixed points, then the locus of a point which moves in such a way that the angle, APB is
a right angle is
a) A circle b) An ellipse c) A parabola d) None of these

208.
If in a ∆ ABC , a cos
2
( C2 )+c cos ( A2 )= 32b , then the sides a , b and c
2

a) Are in AP b) Are in GP c) Are in HP d) Satisfy a+ b=c

209. For a regular polygon, let r and r be the radii of the inscribed and the circumscribed circles. A false
statement among the following is
a) There is regular polygon with r = 1 b) There is a regular polygon r = 1
R 2 R √2
c) There is a regular polygon with r = 2 d) There is a regular polygon with r = √ 3
R 3 R 2
210. In a triangle vertex angles are A , B ,C and side BC are given. The area of ∆ ABC is

a) s ( s−a )( s−b ) (s−c)


2
b) b sin C sin A
2 sin B
2
c) ab sin C d) 1 a sin B sin C
.
2 sin A
211. A flag staff in the centre of a rectangular field whose diagonal is 1200 m and subtends angle 15 ° and 45 °
at the mid point of the sides of the field. The height of the flag staff is
a) 200 m b) c) d) 400 m
300 √ 2+ √ 3 m 300 √ 2−√ 3 m
212. On the level ground the angle of elevation of the top of a tower is 30 ° . On moving 20 m nearer the tower,
the angle of elevation is found to be 60 ° . The height of the tower is
a) 10 m b) 20 m c) d) None of these
10 √ 3 m
213. The angle of elevation of the top of a tower at any point on the ground is 30 ° and moving 20 metres
towards the tower it becomes 60 ° . The height of the tower is
a) 10 m b) c) 10 d) None of these
10 √ 3 m m
√3
214. a+b
If angles A , B and C are in AP, then is equal to
c
a) 2 sin A−C b) 2 cos A−C c) cos A−C d) sin A−C
2 2 2 2
215. If in a ∆ ABC , the altitude from the vertices A , B ,C on opposite sides are in HP, then sin A , sin B , sin C
are in
a) GP b) Arithmetic-Geometric Progression

c) AP d) HP

216. The area of an equilateral triangle that can be inscribed in the circle
2 2
x + y −4 x−6 y−12=0, is
a) 25 √ 3 sq units b) 35 √ 3 sq units c) 55 √ 3 sq units d) 75 √ 3 sq units
4 4 4 4

P a g e | 18
217. The area of a triangle is 5 and its two vertices are A(2, 1) and B(3 ,−2). The third vertex lies on y=x +3 .
Then, third vertex is
a) 7 13
(2 , 2 ) b) 5 5
(2 , 2) c) −3 3
( 2 , 2) d) (0, 0)

218. The incentre of the triangle with vertices ( 1 , √ 3 ) ,(0 , 0) and (2, 0) is

( 2)
a) 1 , √ 3
( 3 √3 )
b) 2 , 1
(3 2 )
c) 2 , √ 3
( √3 )
d) 1 , 1

219.
The area (in square unit) of the triangle formed by the points with polar coordinates (1, 0), 2 , ( π3 ) and
(3 , 23π ) is
a) 11 √ 3 b) 5 √ 3 c) 5 d) 11
4 4 4 4
220. The points P is equidistant from A(1 , 3), B(−3 , 5) and C (5 ,−1), then PA is equal to

a) 5 b) c) 25 d)
5 √5 5 √ 10
221. A rod of length l slides with its ends on two perpendicular lines. The locus of a point which divides it in the
ratio 1 ∶ 2, is
a) 2 2 2 b) 2 2 2 c) 2 2 2 d) 2 2 2
36 x + 9 y =4 l 36 x + 9 y =l 9 x + 36 y =4 l 9 x −36 y =4 l
222. The circumradius of the triangle whose sides are 13, 12 and 5, is

a) 15 b) 13 c) 15 d) 6
2 2
223. Locus of centroid of the triangle whose vertices are ( a cos t , a sin t ) ,(b sin t ,−b cos t) and (1, 0), where t is
a parameter, is
a) b)
( 3 x−1 ) + ( 3 y )2=a 2−b2
2
( 3 x−1 )2 + ( 3 y )2=a 2+b 2
c) d)
( 3 x+ 1 )2+ (3 y )2=a2 +b2 ( 3 x+ 1 )2+ (3 y )2=a2−b 2
224. AB is a vertical pole with B at the ground level and A at the top. A man finds that the angle of elevation of
the point A from a certain point C on the ground is 60 ° . He moves away from the pole along the line BC to
a point D such that CD =7m. From D the angle of elevation of the point A is 45° . Then the height of the
pole is
a) 7 √ 3
2 ( 1
)
√ 3+1
m b) 7 √ 3
2 √ 3−1
1
m
( ) c) 7 √ 3 ( 3+1 ) m
2
√ d) 7 √ 3 ( 3−1 ) m
2

225. If C is the reflection of A(2, 4 ) in x -aixs and B is the reflection of C in y -axis, then ¿ AB∨¿ is

a) 20 b) c) d) 4
2 √5 4 √5
226. ABC is an isosceles triangle if the coordinates of the base are B (1 , 3 ) and C (−2 ,7) , the coordinates of
vertex A can be
a) (1 , 6)
(2 )
b) −1 , 5
(6 )
c) 5 ,6
(
d) −8 , 1
8 )
227.
Point ( 12 ,− 134 ) divides the line joining the points (3 ,−5) and (−7 , 2) in the ratio of
a) 1 :3 internally b) 3 :1 internally c) 1 :3 externally d) 3 :1 externally

P a g e | 19
228.
( 32 ) , B(−5 , 0) is
The orthocenter of the triangle with vertices O ( 0 , 0 ) , A 0 ,

(2 4)
a) 5 , 3
( 2 4)
b) −5 , 3
( 2)
c) −5 , 3 d) (0 , 0)

229. Area of triangle formed by the lines x + y=3 and angle bisectors of the pair of straight lines x 2− y 2 +2 y=1
is
a) 2 sq units b) 4 sq units c) 6 sq units d) 8 sq units

230. ABCD is a rectangular field. A vertical lamp post of height12m stands at the corner A . If the angle of
elevation of its top from B is 60 ° and from C is 45 ° , then the area of the field is
a) b) c) 48 sq m d)
48 √ 2 sq m 48 √ 3 sq m 12 √ 2 sq m
231. If two adjacent sides of a cylinder quadrilateral are 2 and 5 and the angle between them is 60 ° . If the third
side is 3, then the remaining fourth side is
a) 2 b) 3 c) 4 d) 5

232. In ∆ ABC , 2 R 2 sin A sin B sin C is equal to

a) 2 b) ab+ bc+ ca c) ∆ d) None of these


s
233. If in a ∆ ABC , the altitudes from the vertices A , B ,C on opposite sides are in HP, then sin A , sin B , sin C
are in
a) HP b) Arithmetico-Geometric Progression

c) AP d) GP

234. The middle point of the line segment joining (3 ,−1) and (1, 1) is shifted by two units (in the sense of
increasing y ) perpendicular to the line segment. Then the coordinates of the point in the new position are
a) (2− 2 , 2) b) (2 , 2− 3) c) (2+ 2 , 3) d) None of these
√ √ √ √
235. If area of triangle with vertices (0, 0), (0, 6) and (α , β ) is 15 sq unit, then

a) α =± 5 , β=5 b) α =± 10 , β=5

c) α =± 5 , β=2 d) α =± 5 , β can take any real value

236. Area of quadrilateral whose vertices are (2, 3), (3, 4), (4, 5) and (5, 6) is equal to

a) 0 b) 4 c) 6 d) None of the above

237. If the distance between (2, 3) and (−5 , 2) is equal to the distance between (x ,2) and (1, 3), then the
values of x are
a) −6 , 8 b) 6, 8 c) −8 , 6 d) −7 , 7

238. A circle is inscribed in a equilateral triangle of side a . The area of the circle is

a) 2 b) 2 c) 2 d) None of these
3 π a sq units 2 a sq units a sq units
239. The x -axis, y -axis and a line passing through the point A(6 , 0) form a triangle ABC . If ∠ A=30 ° then the
area of the triangle, in sq units is
a) b) c) d)
6 √3 12 √ 3 4 √3 8 √3
240. cos A cos B cos C
In a ∆ ABC , if = = and the side a=2, then area of the triangle is
a b c

P a g e | 20
a) 1 sq unit b) 2 sq unit c) √3 sq unit d)
2
√ 3 sq unit
241. π
In an ambiguous case of solving a triangle when a=√ 5 , b=2 , ∠ A= , the two possible values of third
6
side are c 1 and c 2, then
a) |c −c |=2 √ 6 b) |c −c |=4 √ 6 c) |c −c |=4 d) |c −c |=6
1 2 1 2 1 2 1 2

242. 2 2 C 2 2C
In ∆ ABC , ( a−b ) cos + ( a+b ) sin is equal to
2 2
a) 2 b) 2 c) 2 d) None of these
a b c
243. In ∆ ABC , with usual notation, observe the two statements given below
2
I. r r 1 r 2 r 3 =∆
2
II. r 1 r 2 +r 2 r 3+ r 3 r 1=s
Which of the following is correct?
a) Both I and III are true b) I is true, II is false c) I is false, II is true d) Both I and II are false

244. If the angles of a triangle be in the ratio 1:2:7, then the ratio of its greatest side to the least side is

a) 1:2 b) 2:1 c) ( √ 5+1 ) :( √5−1) d) (


√ 5−1 ) :( √5+ 1)
245. From the top of a cliff 300 metres high, the top of a tower was observed at an angle of depression 30 ° and
from the foot of the tower the top of the cliff was observed at an angle of elevation 45 ° , the height of the
tower is
a) ( ) b) ( ) c) ( ) d) None of these
50 3− √ 3 m 200 3−√ 3 m 100 3− √3 m
246. If x 1 , x 2 , x 3 and y 1 , y 2 , y 3 are both in GP with the same common ratio, then the points
(x 1 , y 1 ), (x2 , y 2) ,(x 3 , y 3 )
a) Lie on a straight line b) Lie on an ellipse

c) One vertices of a triangle d) Lie on a circle

247. The coordinates axes are rotated through an angle135 ° . If the coordinates of a point P in the new system
are known to be (4 ,−3), then the coordinates of P in the original system are
a)
( √12 , √72 ) b)
( √12 ,− √72 ) ( √2
c) −1 ,− 7
√2 ) ( √ 2 √2 )
d) −1 , 7

248. The coordinate axes rotated though an angle 135∘. If the coordinates of a point P in the new system are
known to be (4 ,−3), then the coordinates of P in the original system are
a)
( √12 , √72 ) b)
( √12 ,− √72 ) ( √2
c) −1 ,− 7
√2 ) ( √ 2 √2 )
d) −1 , 7

249. When the elevation of sun changes from 45 ° to 30 ° the shadow of a tower increases by 60 m the height of
the tower is
a) b) 30( 2+1)m c) 30( 3−1)m d) 30( 3+1)m
30 √ 3 m √ √ √
250. The angle of elevation of the top of a tower from a point A tan−1 6 and that from B due West of it, is
−1 2
tan 7.5 . If h is the height of the tower, then AB=λ h , where λ is equal to
a) 21 b) 42 c) 41 d) None of these
700 1300 900
251. 2 B
In a triangle, if r 1 +r 3=kco s , then k is equal to
2

P a g e | 21
a) R b) 2 R c) 3 R d) 4 R

252. AB is a vertical pole and C is its middle point. The end A is on the level ground and P is any point on the
level ground other than A the portion CB subtends an angle β at P . If AP: AB=2 :1 , then β=¿
a) tan−1 4 b) tan−1 1 c) tan−1 5 d) tan−1 2
9 9 9 9
253.
(
If the points ( x +1 ,2 ) , ( 1 , x +2 ) ,
1
,
2
)
x+ 1 x +1
are collinear, then x is

a) 4 b) 5 c) −4 d) None of these

254. If in a ∆ ABC , CD is the angular bisector of the ∠ ACB, then CD is equal to

a) a+b cos C b) a+b cos C c) 2ab cos C d) None of these


2ab 2 ab 2 a+b 2
255. A tower stands at the center of a circuit park. A and B are two points on the boundry of the park such that
AB¿ ) subtends an angles of 60 ° at the foot of the tower and the angle of elevation of the top of the tower
from A or B is 30 ° . The height of the tower is
a) 2 a b) c) a d)
2 a √3 √3
√3 √3
256. Consider three points
P=(−sin ( β−α ) ,−cos β)
Q=(cos ( β−α ) , sin β)
And R=¿,
π
Where0< α , β , θ< . Then,
4
a) P lies on the line segment R Q b) Q lies on the line segment PR

c) R lies on the line segment Q P d) P , Q, R are non-collinear

257. In ∆ ABC , if 8 R2=a2 +b2 +c 2, then the triangle is

a) Right angled b) Equilateral c) Acute angled d) Obtuse angled

258. A triangle with vertices ( 4 , 0 ) , (−1 ,−1 ) ,(3 ,5) is

a) Isosceles and right angled b) Isosceles but not right angled

c) Right angled but not isosceles d) Neither right angled nor isosceles

259. In a ∆ ABC , the correct formulae among the following are


A B C
III. r =4 R sin sin sin
2 2 2
A
IV. r 1= ( s−a ) tan
2

V. r 3=
s−c
a) Only I, II b) Only II, III c) Only I, III d) I, II, III

260. The area (in square unit) of the triangle formed by the lines x=0 , y=0 and 3 x+ 4 y =12, is

a) 3 b) 4 c) 6 d) 12

261. Three distinct points A , B and C given in the two dimensional coordinate plane such that the ratio of the

P a g e | 22
1
distance of any one of them from the point (1, 0) to the distance from the point (−1 , 0) is equal to . Then ,
3
the circumcentre of the triangle ABC is at the point

(4 )
a) 5 , 0
(2 )
b) 5 , 0
(3 )
c) 5 , 0 d) (0, 0)

262. One possible condition for the three points ( a , b ) ,(b , a) and (a 2 ,−b2 ) to be collinear, is

a) a−b=2 b) a+ b=2 c) a=1+b d) a=1−b

263. Three vertical towers standing at A , B ,C subtends the angle θ A ,θ B ,θ C respectively at the circumcentre of
the ∆ ABC , then tanθ A , tan θ Band tanθ C are in
a) AP b) GP c) HP d) None of these

264. The area of the triangle whose sides are 6 , 5 , √ 13 (in square unit) is

a) b) 9 c) d) 11
5 √2 6 √2
265. If points A ( x 1 , y 1 ) , B (x 2 , y 2) and C (x 3 , y 3 ) are such that x 1 , x 2 , x 3 and y 1 , y 2 , y 3are in AP, then

a) A , B and C are concyclic points b) A , B and C are collinear points

c) A , B and C are vertices of an equilateral triangle d) None of the above

266. Two points P ( a , 0 ) and Q(−a , 0) are given, R is a variable point on one side of the line PQ such that
∠ RPQ−∠ RQP is 2 α , then
a) Locus of R is 2 2 2 b) Locus of R is 2 2 2
x − y +2 xy cot 2 α −a =0 x + y + 2 xy cot α −a =0
c) Locus of R is a hyperbola, if α = π d) Locus of R is a circle, if α = π
4 4
267. π π
In a∆ ABC , medians, AD and BE are drawn. If AD=4 , ∠ DAB= ∧∠ ABE= , then the area of the
6 3
∆ ABC is
a) 8 sq units b) 16 sq units c) 32 sq units d) 64 sq units
3 3 3 √3 3
268. Point Q is symmetric to P(4 ,−1) with respect to the bisector of the first quadrant. The length of P Q is

a) b) c) d)
3 √2 5 √2 7 √2 9 √2
269. The area of triangle formed by the points ( a , b+ c ) , ( b , c +a ) , (c ,a+ b) is equal to

a) abc b) 2 2 2 c) ab+ bc+ ca d) 0


a + b +c
270. π
Let 0 ≤ θ ≤ and x=X cos θ+Y sin θ , y= X sinθ−Y cos θ such that x 2+ 4 xy + y 2=a X 2 +b Y 2 , where a , b
2
are constants, then
a) a=−1 , b=3 , θ= π b) a=1 , b=−3 , 2= π c) a=3 , b=−1, θ= π d) a=3 , b=−1, θ= π
4 3 4 3
271. If a point P(4 , 3) is shifted by a distance √ 2 unit parallel to the line y=x , then coordinates of p in new
position are
a) (5, 4) b) (5+ 2 , 4+ 2) c) (5− 2 , 4− 2) d) None of these
√ √ √ √
272. From the top of a cliff 50m high, the angles of depression of the top and bottom of a tower are observed to
be 30° and45 ° . The height of tower is

P a g e | 23
( )
a) 50m b) c) d) 50 1− √ 3 m
50 √ 3 m 50 ( √ 3−1 ) m
3
273. The vertex of an equilateral triangle is (2 ,−1) and the equation of its base is x +2 y=1, the length of its
sides is
a) 2 b) 4 c) 1 d) 4
√15 3 √3 √5 √15
274. If the elevation of the sun is 30 ° , then the length of the shadow cast by a tower of 150 ft. height is

a) b) c) d) None of these
75 √ 3 ft . 200 √ 3 ft . 150 √ 3 ft .
275. The area of the triangle formed by the points (2, 2), (5, 5), (6, 7) is equal to (in square unit)

a) 9 b) 5 c) 10 d) 3
2 2
276. The orthocenter of the ∆ OAB , where O is the origin, A(6 , 0) and B(3 , 3 √ 3) is

a) (9 /2, 3/2) b) (3 , 3) c) ( 3 , 3) d) (3 ,− 3)
√ √ √ √
277. In a ∆ ABC , cos A+cos B+ cos C is equal to

a) 1+ r b) 1− r c) 1− R d) 1+ R
R R r r
278. If in a ∆ PQR ,sin P , sin Q ,sin R are in AP, then

a) The altitudes are in AP b) The altitudes are in HP

c) The medians are in GP d) The medians are in AP

279. The angle of elevation of an object on a hill from a point on the ground is 30 ° . After walking 120 metres
the elevation of the object is 60 ° . The height of the hill is
a) 120 m b) c) d) 60 m
60 √ 3 m 120 √ 3 m
280. If the median AD of ∆ ABC , makes an angle θ with side AB, then sin( A−θ) is equal to

(c )
a) b cosec θ
(c )
b) b sin θ
(b)
c) c sin θ
(b)
d) c cosec θ

281. The angle of elevation of a cliff at a point A on the ground and a point B , 100 m vertically at A are α and β
respectively. The height of the cliff is
a) 100 cot α b) 100 cot β c) 100 cot β d) 100 cot β
cot α −cot β cot α −cot β cot β−cot α cot β+ cot α
282. A quadrilateral ABCD in which AB=a , BC=b ,CD =c and DA=d is such that one circle can be
inscribed in it and another circle can be circumscribed about it, then cos A is equal to
a) ad +bc b) ad−bc c) ac +bd d) ac−bd
ad−dc ad +bc ac−bd ac +bd
283.
If λ be the perimeter of the ∆ ABC , then b cos ( )
2 C
2
+ c cos 2
B
2 ()
is equal to

a) λ b) 2 λ c) λ d) None of these
2
284. If in the ∆ ABC , ∠ B=45 ° , then a +b +c is equal to
4 4 4

a) 2 2 2 b) 2 2 2
2 a (b + c ) 2 c (a +b )
c) 2 2 2 d) 2 2 2 2 2 2
2 b (a + c ) 2( a b + b c + c a )

P a g e | 24
285. The ratio in which the line x + y=4 divides the line joining the points (1 ,−1) and (5, 7) is

a) 1 :2 b) 2 :1 c) 1 :3 d) 3 :1

286. The coordinates of the orthocenter of the triangle formed by (0, 0), (8, 0), (4, 6) is

a) (4, 0) b) (6, 3) c) (6, 0) d) None of these

287. If ∆=a 2−( b−c )2, where ∆ is the area of ∆ ABC , then tan A is equal to

a) 15 b) 8 c) 8 d) 1
16 15 17 2
288. b +c
In ∆ ABC , is equal to
a
1 1 1 1
cos (B−C) sin (B−C) cos (B+C) cos (B+C)
a) 2 b) 2 c) 2 d) 2
1 1 1 1
sin A cos A sin A cos A
2 2 2 2
289. In a cubical hall ABCDPQRS with each side 10 m, G is the centre of the wall BCRQ and T is the mid point
of the side AB. The angle of elevation of G at the point T is
a) b) c) d) None of these
sin (1/ √ 3) cos (1/ √ 3) cot (1/ √ 3)
−1 −1 −1

290. If p1 , p2 , p 3 are respectively the perpendicular from the vertices of a triangle to the opposite sides, then
p1 , p2 , p 3 is equal to
2 2 2 2 2 2
a) 2 2 2 b) 2 2 2 c) 4 a b c d) a b c
a b c 2a b c 2 2
R 8R
291. An observer standing on a 300 m high tower observes two boats in the same direction their angles of
depression are 60 ° and 30 ° respectively. The distance between boats is
a) 173.2 m b) 346.4 m c) 25 m d) 72 m

292. If the length of the sides of a triangle are 3, 4 and 5 unit, then R is

a) 3.5 b) 3.0 c) 2.0 d) 2.5

( )( )
293. r1 r1
In a triangle 1− 1− =2 , then the triangle is
r2 r3
a) Right angled b) Isosceles c) Equilateral d) None of these

294. A +B
If in a ∆ ABC , a tan A+ b tan B=( a+b ) tan , then
2
a) A=B b) A=−B c) A=2 B d) B=2 A

295. The coordinates of the circumcentre of the triangle with vertices (8 , 6),(8 ,−2) and
(2 ,−2) are

( )
a) 6 , 2
3
b) (8 , 2) c) (5 ,−2) d) (5 , 2)

296. Points D , E are taken on the side BC of a ∆ ABC such that BD=DE=EC . If
sin ( x+ y ) sin( y+ z)
∠ BAD=x , ∠ DAE= y ,∠ EAC =z , then the value of is equal to
sin x sin z
a) 1 b) 2 c) 4 d) None of these

297. An observer on the top of tree, finds the angle of depression of a car moving towards the tree to be 30 ° .
After 3 min this angle becomes 60 ° . After how much more time, the car will reach the tree?
P a g e | 25
a) 4 min b) 4.5 min c) 1.5 min d) 2 min

298. If PQ be a vertical tower subtending angles α , β∧γ at the points A , B∧C respectively on the line in the
horizontal plane through the foot D of tower and on the same side of it, then
BC cot α −CA cot α + AB cot γ is equal to
a) 0 b) 1 c) 2 d) None of these

299. The equation of the three sides of a triangle are x=2 , y +1=0 and x +2 y=4 . The coordinates of the
circumcentre of the triangles are
a) (4, 0) b) (2 ,−1) c) (0, 4) d) (−1 , 2)

300. On one bank of river there is a tree. On another bank, an observer makes an angle of elevation of 60 ° at the
top of the tree. The angle of elevation of the top of the tree at a distance 20 m away the bank is 30 ° . The
width of the river is
a) 20 m b) 10 m c) 5 m d) 1 m

301. Consider the following statements :


1. In ∆ ABC , a= √ 3+1 , ∠ B=30 ° ,∠ C=45 °, then c is equal to
2. In a triangle, if a 2+ b2 +c 2=8 R2, then the triangle is right angled
7
3. In a ∆ ABC , a=2, b=3 , c=4, then cos A=
8
Which of the statements given above is/are correct?
a) Only (1) b) Only (2) c) Only (3) d) All of these

302. If A is the area and 2 s the sum of three sides of a triangle, then

a) A ≤ s
2
b) s
2
c) A> s
2
d) None of these
A≤
3 √3 2 √3
303. A flag staff of 5 m high stands on a building of 25 m high. At an observer at a height of 30 m, the flag staff
and the building subtend equal angles. the distance of the observer from the top of the flag staff is

√ √
a) 5 √ 3 b) 3 c) 2 d) None of these
m 5 m 5 m
2 2 3
304. A tower of height b subtends an angle at a point O on the level of the foot of the tower and at a distance
' a ' from the foot of the tower. If the pole mounted on the tower also subtends an equal angle at O , the
height of the pole is

( ) ( ) ( ) ( )
2 2 2 2 2 2 2 2
a) b a −b b) b a +b c) a a −b d) a a +b
2 2 2 2 2 2 2 2
a +b a −b a +b a −b
305. If the angles of a triangle are in the ratio 4 :1:1 , then the ratio of the longest side to the perimeter is

a) 3 :(2+ 3) b) 1 :6 c) 1 :(2+ 3) d) 2 :3
√ √ √
306. If two angles of a triangle are 45 ° and tan−1 (2), then the third angle is

a) 60 ° b) 75 ° c) −1 d) 90 °
tan 3
307. The angle of elevation of top of a tower form a point on the ground is 30 ° and it is 60 ° when it is viewed
from a point located 40m away the initial point towards the tower. The height of the tower is
a) b) √ 3 m c) −√ 3 m d)
−20 √ 3 m 20 √ 3 m
20 20
308. The orthocentre of the triangle formed by the points (0, 0), (4, 0) and (3, 4) is

P a g e | 26
a) (2 , 0)
(2 )
b) 3 , 2
(4 )
c) 3 , 3
( 4)
d) 3 , 3

309. In order to remove xy -term from the equation 5 x 2+ 4 √ 3 xy+ 9 y 2−8=0 the coordinate axes must be
rotated through an angle
a) π /6 b) π /4 c) π /3 d) π /2

310. If C is a point on the line segment joining A(−3 , 4) and B(2 , 1) such that AC=2 BC , then the coordinate
of C is
a) 1
( 3 , 2) b)
(2 , 3 )
1 c) (2, 7) d) (7, 2)

311. The image of the centre of the circle x 2+ y 2=a2 with respect to the mirror image x + y=1, is

( )
a) 1 b) ( 2 , 2) c) ( 2 ,2 2) d) None of these
, √2 √ √ √ √
√2
312. In a ∆ ABC , cosec A (sin B cos C+ cos B sinC ) is equal to

a) c b) a c) 1 d) c
a c ab
313. If A ( 3 , 5 ) , B (−5 ,−4 ) , C(7 ,10) are the vertices of a parallelogram, taken in the order, then the
coordinates of the fourth vertex are
a) (10, 19) b) (15, 19) c) (19, 10) d) (19, 15)

314. Two pillars of equal height stand on either side of a road-way which is 60 m wide. At a point in the road-
way between the pillars, the elevation of the top of pillars are 60 ° and 30° .The height of the pillars is
a) b) 15 c) 15 m d) 20 m
15 √ 3 m m
√3
315. From the top of a cliff of height a , the angle of depression of the foot of a certain tower is found to be
double the angle of elevation of the top of the tower of height h . If θ be the angle of elevation, then its value
is
a)
cos−1
2h
a √ b)
sin−1
√ 2h
a
c)

sin−1
a
2−h
316. The points A ( 2 a , 4 a ) , B(2 a , 6 a) and C ( 2 a+ √ 3 a ,5 a ) , a> 0 are the vertices of
d)

tan−1 3−
2h
a

a) An isosceles triangle b) A right angled triangle

c) An acute angled triangle d) None of the above

317. If a vertex of a triangle is (1, 1) and the mid points of two sides through the vertex are (−1 , 2 ) and (3, 2),
then the centroid of the triangle is

( 3)
a) 1 , 7
(3 3)
b) 1 , 7
( 3 3)
c) −1 , 7
(
d) −1 , 7
3 )
318. P , Q, R and S are the points on the line joining the points P ( a , x ) and T ( b , y ) such that

PQ=QR=RS=ST , then ( 5 a+38 b , 5 x+83 y ) is the mid point of


a) PQ b) QR c) RS d) ST

319. Orthocenter of the triangle formed by the lines x + y=1 and xy=0 is

a) (0, 0) b) (0, 1) c) (1, 0) d) (−1 , 1)

320. The circumcentre of the triangle with vertices (0, 30), (4, 0) and (30, 0) is

P a g e | 27
a) (10, 10) b) (10, 12) c) (12, 12) d) (17, 17)

321. A vertical pole (more than 100 m high) consists of two portions, the lower being-one third of the whole, if
−1 1
the upper portion subtends and angle tan at a point in a horizontal plane through the foot of the pole
2
and distance 40 ft from it, then the height of the pole is
a) 100 ft b) 120 ft c) 150 ft d) None of these

322. If a point P ( 4 ,3 ) is rotated through an angle 45 ∘ in anti-clockwise direction about origin, then coordinates
of P in new position are
a)
( √12 , √72 ) ( √2
b) −7 ,− 1
√2 ) ( √ 2 √2 )
c) −1 , 7 d)
( √12 ,− √72 )
323. The horizontal distance between two towards is 60m and the angle of depression of the top of the first
tower as seen from the top of the second is 30 ° . If the height of the second tower be 150m, then the height
of the first tower is
a) 90 m b) ¿ c) ¿ d) None of the above

324. In a ∆ ABC , if sin A+sin B+sin C ¿ ( sin A+ sin B−sin C ) =3 sin A sin B , then the angle C is equal to

a) π b) π c) π d) π
2 3 4 6
325. From the bottom of a pole of height h the angle of elevation of the top of a tower is α and the pole subtends
an angle β at the top of the tower. The height of the tower is
a) h tan (α −β) b) h cot( α−β ) c) cot(α−β ) d) None of these
tan ( α −β )−tan α cot ( α −β )−cot α cot ( α −β )−cot α
326.
A variable line through the point , ( )
1 1
5 5
cuts the coordinate axes in the points

A and B. If the point P divides AB internally in the ratio 3:1, then the locus of P is
a) 3 y + x=20 xy b) y +3 x=20 xy c) x + y=20 xy d) 3 x+ 3 y=20 xy

327. If the points (1, 2) and (3, 4) were to be on the same side of the line 3 x−5 y +a=0 ,then

a) 1<a< 6 b) 7< a<11 c) a> 11 d) a< 7∨a>11

328. The area of the region bounded by the lines y=| x−2|, x=1 , x=3 and the x -axis is

a) 1 b) 2 c) 3 d) 4

329. A B
cot cot −1
2 2
In triangle ABC , the value of is
A B
cot cot
2 2
a) a b) c c) 2a 2c
d)
a+b+ c a+b+ c a+b+ c a+b+ c
330. If the coordinates of the centroid and a vertex of an equilateral triangle are (1 , 1) and (1 , 2) respectively,
then the coordinates of another vertex, are

( 2) ( 2) ( 2)
a) 2−√ 3 ,− 1 b) 2+3 √ 3 ,− 1 c) 2+ √ 3 , 1 d) None of these
2 2 2
331. In any triangle ABC , c sin 2 B+b sin 2 C is equal to
2 2

a) ∆ b) ∆ c) 2 ∆ d) 4 ∆
2

P a g e | 28
332. A house of height 100 m subtends a right angle at the window of an opposite house. If the height of the
window be 64 m, then the distance between the two houses is
a) 48 m b) 36 m c) 54 m d) 72 m

333. The angle of elevation of the top of a vertical pole when observed from each vertex of a regular hexagon is
π
. If the area of the circle circumscribing the hexagon be A metre 2 , then the area of the hexagon is
3
a) 3 √ 3 A m2 b) √ 3 A m 2 c) 3 √ 3 A m2 d) 3 √ 3 A m2
8 π 4π 2π
334. The area of the segment of a circle of radius a subtending an angle of 2 α at the centre is

(
a) a 2 α + 1 sin 2 α
2 ) b) 1 a2 sin 2 α
2 (
c) a 2 α + 1 sin 2 α
2 ) d) 2
a α
335. A man of height 6 ft. observes the top of a tower and the foot of the tower at angles of 45 ° and 30 ° of
elevation and depression respectively. The height of the tower is
a) 13.79 m b) 14.59 m c) 14.29 m d) None of these

336. If the sides of the triangles are 5 k , 6 k , 5 k and radius of incircle is 6, the value of k is equal to

a) 4 b) 5 c) 6 d) 7

337. At the foot of the mountain the elevation of its summit is 45 ° , after ascending 100 m towards the
mountain up a slope of 30 ° inclination, the elevation is found to be 60 ° . The height of the mountain is
a) √ 3+ 1 m b) √ 3−1 m c) √ 3+ 1 m
d) None of these
2 2 2 √3
338. At each end of a horizontal line of length 2 a, the angular elevation of the peak of a vertical tower is θ and
that at its middle point it is ϕ . The height of the peak is
a) a sin θ sin ϕ b) a sin θ sin ϕ
√sin (θ+ ϕ)sin(ϕ−θ)
c) a cos θ cos ϕ d) None of the above
√ cos(ϕ+θ)cos (ϕ−θ)
339. cos A cos B cos C 1
In a triangle ABC , = = . if a= , then the area of the triangle (in square unit) is
a b c √6
a) 1/24 b) c) 1 d) 1
√ 3/24 8 √3
340. A ladder leavs again a wall at an angle α to the horizontal. Its foot is pulled away through a distance a 1 so
that it slides a distance b 1down the wall and rests inclined at angle β with the horizontal. It foot is further
pulled aways through a 2, so that it slides a further distance b 2 down the wall and is now, inclined at an
angle γ . If a 1 a 2=b1 b2, then
a) α + β+ γ is greater than π b) α + β+ γ is equal to π

c) α + β+ γ is less than π d) Nothing can be said about α + β+ γ

341. ( a+b+ c )( b +c−a ) ( c+ a−b ) (a+ b−c)


In a ∆ ABC , 2 2 equals
4b c
a) 2 b) 2 c) 2 d) 2
cos A cos B sin A sin B
342. The locus of a point P which moves such that 2 PA=3 PB , where A(0 ,0) and B(4 ,−3) are points, is

a) 2 2 b) 2 2
5 x −5 y −72 x +54 y+225=0 5 x +5 y −72 x +54 y +225=0

P a g e | 29
c) 2 2 d) 2 2
5 x +5 y +72 x−54 y +225=0 5 x +5 y −72 x−54 y−225=0
343. The incentre of the triangle formed by (0, 0), (5, 12), (16, 12) is

a) (7, 9) b) (9, 7) c) (−9 , 7) d) (−7 , 9)

344. A C 1
In ∆ ABC , if tan tan = , then a , b , c are in
2 2 2
a) AP b) GP c) HP d) None of these

345. 2 A 2 B 2C
In ∆ ABC , if sin , sin , sin be in HP, then a , b , c will be in
2 2 2
a) AP b) GP c) HP d) None of these

346. If O is the origin and P(2 , 3) and Q(4 , 5) are two points, then OP . OQ cos ∠ POQ=¿ ¿

a) 8 b) 15 c) 22 d) 23

347. sin 3 B
If in a ∆ ABC , 2b 2=a 2+ c2 , then is equal to
sin B

( ) ( 2 ca )
2 2 2 2 2 2 2 2 2 2
a) c −a b) c −a c) c −a d) c −a
2 ca ca ca
348. A spherical balloon of radius r subtends an angle α at the eye of an observer. If the angle of elevation of the
centre of the balloon be β , then height of the centre of the balloon is
a) r cosec α sin β
(2) b) r cosec α sin β
(2 ) (2)
c) r sin α cosec β
(2 )
d) r sin α cosec β

349. A point P(2 , 4) transtates to the point Q along the parallel to the positive direction of x -axis by 2 units. If
O be the origin, then ∠ OPQ is

√ (√ 400 )
d) None of these
a) −1 399
sin
400
b) cos−1 1
( 20 )
c) −1 399
−sin

350. From the top of a hill h meters high, the angles of depressions of the top and the bottom of a pillar are α
and β respectively. The height (in meters) of the pillar is
a) h ¿ ¿ b) h ¿ ¿ c) h ¿ ¿ d) h ¿ ¿

351. If G(1 , 4) is the centroid of triangle ABC having its two vertices A and B at (4 ,−3) and (−9 , 7)
respectively, then area of the triangle ABC in square units, is
a) 138 b) 319 c) 183 d) 381
2 2 2 2
352. If a=2 √ 2 , b=6 , A=45 °, then

a) No triangle is possible b) One triangle is possible

c) Two triangles are possible d) Either no triangle or two triangles are possible

353. Two poles of equal height stand on either side of a 100 m wide road. At a point between the poles the
angles of elevation of the topes of the poles are 30 ° and 60 ° . The height of each pole is
a) 25 m b) c) 100 d) None of these
25 √ 3 m m
√3
354. Area (in sq unit) enclosed by y=1 , 2 x+ y =2 and x + y=2 is

a) 1 sq unit b) 1 sq unit c) 1 sq unit d) 2 sq units


2 4

P a g e | 30
355. The feet of the perpendicular drawn from P to the sides of a ∆ ABC are collinear, then P is

a) Circumcentre of ∆ ABC b) Lies on the circumcircle of ∆ ABC

c) Excentre of ∆ ABC d) None of the above

356. Let O ( 0 , 0 ) , P ( 3 , 4 ) ,Q ( 6 , 0 ) be the vertices of the triangleOP Q . The point R inside the triangle OP Q is
such that the triangles OPR , P Q R ,O Q R are of equal area. The coordinates of R are

(3 )
a) 4 , 3
( 3)
b) 3 , 2
( 3)
c) 3 , 4
(3 3)
d) 4 , 2

357. A tree is broken by wind, its upper part touches the ground at a point 10 metres from the foot of the tree
and makes an angle of 45 ° with the ground. The entire length of the tree is
a) 15 metres b) 20 metres c)
10 ( 1+ √ 2 ) metres (
d) 10 1+ √ 3 metres
2 )
358. Without change of axes the origin is shifted to ( h , k ) , then from the equation
2 2
x + y −4 x+6 y−7=0 the terms containing linear powers are missing. The point (h , k ) is
a) (3 , 2) b) (−3 , 2) c) (2 ,−3) d) (−2 ,−3)

359. The horizontal distance between two towers is 60 m and the angle of depression of the top of the first
tower as seen from the top of the second is 30 ° . If the height of the second tower be 150 m, then the height
of the first tower is
a) (150−60 3)m b) 90 m c) (150−20 3)m d) None of these
√ √
360. x y x y
Let a> 0 , b>0 . The sum of the distance of the point (a ,b) from the lines + =1and + =1is
a b b a
a) a+b b) c) 2ab d)
2 √ ab a+b √ a2 +b 2
361. 1 1 3
In ∆ ABC , if + = , then C is equal to
b+c c+ a a+b+ c
a) 90 ° b) 60 ° c) 45 ° d) 30 °

362. If in a ∆ ABC , 4 sin A=4 sin B=3 sin C , then cos C is equal to

a) 1/3 b) 1/9 c) 1/27 d) 1/18

363. In a ∆ ABC , ( b+c ) ( bc ) cos A + ( a+ c )( ac ) cos B+ ( a+ b ) ( ab ) cos C is

a) 2 2 2 b) 3 3 3
a + b +c a + b +c
c) ( 2 2 2 d) ( a+ b+c ) (ab+bc +ca)
a+ b+c ) (a + b +c )
364. If a flag staff of 6 m high placed on the top of a tower throws a shadow of 2 √ 3 m along the ground, then the
angle (in degrees) that the sun makes with the ground is
a) 60 ° b) 80 ° c) 75 ° d) None of these

365. If the angles of a ∆ ABC be in AP, then

a) 2 2 2 b) 2 2 2 c) 2 2 2 d) 2 2 2
c =a + b −ab b =a +c −ac a =b +c −ac b =a +c
366. The sides of a triangle are respectively 7 cm, 4 √ 3 cm and √ 13 cm, then the smallest angle of the triangle is

a) π b) π c) π d) π
6 3 4 5
367. The straight lines x + y=0 , 3 x+ y−4=0 and x +3 y−4=0 form a triangle which is

P a g e | 31
a) Right angled b) Equilateral c) Isosceles d) None of these

368. A tower of x metres height has flag staff at its top. The tower and the flag staff subtend equal angles at a
point distant y metres from the foot of the tower. Then, the length of the flag staff in metres is

( ) ( ) ( ) ( )
2 2 2 2 2 2 2 2
a) y x − y b) x x + y c) x x + y d) x x − y
2 2 2 2 2 2 2 2
x +y y −x x −y x +y

| |
369. x1 y1 1
If x 2 y 2 1 =0 , then the points (x 1 , y 1 ), (x2 , y 2) and (x 3 , y 3 ) are
x3 y3 1
a) Vertices of an equilateral triangle b) Vertices of a right angled triangle

c) Vertices of an isosceles triangle d) None of the above

370. If two angles of ∆ ABC are 45 ° and 60 ° , then the ratio of the smallest and the greatest sides are

a) ( b) c) d)
√ 3−1 ) :1 √ 3 : √2 1 : √3 √ 3 :1
371. An aeroplane flying horizontally 1 km above the ground is observed at an elevation of 60 ° and after 10 s
the elevation is observed to be 30 ° . The uniform speed of the aeroplane (in km/h) is
a) 240 b) c) d) None of these
240 √ 3 60 √ 3
372. The angle of elevation of the top of a tower from the top and bottom of a building of height ' a ' are 30 ° and
45 ° respectively. If the tower and the building stand at the same level, the height of the tower is
a) a(3+ √ 3) b) a ( 3+1) c) d) a ( 3−1)
√ a √3 √
2
373. sin 3 B
If in a ∆ ABC , 2b 2=a 2+ c2 , then is equal to
sin B

( ) ( )
2 2 2 2 2 2 2 2 2 2
a) c −a b) c −a c) c −a d) c −a
2 ca ca ca 2 ca
374. The angle of elevation of a cloud from a point h mt. above is θ ° and the angle of depression of its reflection
in the lake is ϕ . Then, the height is
a) h sin(ϕ−θ) b) h sin(ϕ+θ) c) h sin(θ+ ϕ) d) None of these
sin(ϕ+θ) sin(ϕ−θ) sin(θ−ϕ)
375. π π sin ∠BAD
In a∆ ABC , ∠ B= ∧∠ C= . If D divides BC internally in ratio 1:3, then the value of is
3 4 sin ∠CAD
a) 1
√3
b) 1
√6
c) 2
3 √ d) 1
3
376. Let equation of the side BC of a ∆ ABC be x + y +2=0. If coordinates of its orthocentre and circumcentre
are (1, 1) and (2, 0) respectively, then radius of the circumcircle of ∆ ABC is
a) 3 b) c) d) None of these
√ 10 2 √2
377. In a ∆ ABC , if b+ c=2 a and ∠ A=60 °, then ∆ ABC is

a) Equilateral b) Right angled c) Isosceles d) Scalene

378. The orthocenter of the triangle with vertices (−2 ,−6 ) ,(−2 , 4) and (1, 3) is

a) (3, 1) b) (1, 1/3) c) (1,3) d) None of these

379. If a> 0 , b>0 the maximum area of the triangle formed by the points O ( 0 , 0 ) , A (a cos θ , b sin θ) and
B(a cos θ ,−b sin θ) is (in sq unit)

P a g e | 32
a) ab whenθ= π b) 3 ab whenθ= π c) ab whenθ= −π d) 2 2
a b
2 4 2 4 2 4
380. If the coordinates of orthocentre O ' and centroid G of a ∆ ABC are (0 , 1) and (2 , 3) respectively, then the
coordinates of the circumcentre are
a) (3 , 2) b) (1 , 0) c) (4 , 3) d) (3 , 4)

381. The point P is equidistant from A(1 , 3), B(−3 , 5) and C (5 ,−1), then PA is equal to

a) 5 b) c) 25 d)
5 √5 5 √ 10
382. The coordinates of the incentre of the triangle having sides
3 x−4 y=0 , 5 x+12 y=0
and y−15=0 are
a) −1 , 8 b) 1 ,−8 c) 2, 6 d) None of these

383. The mid point of the line joining the points (−10 , 8) and (−6 , 12) divides the line joining the points
(4 ,−2) and (−2 , 4) in the ratio
a) 1 :2 internally b) 1 :2 externally c) 2 :1 internally d) 2 :1 externally

384. The centre of circle inscribed in square formed by the lines x 2−8 x +12=0 and y 2−14 y+ 45=0, is

a) (4, 7) b) (7, 4) c) (9, 4) d) (4, 9)

385. 1 1 1
If A , A 1 , A 2 , A3 be the areas of the incircle and excircles, then + + is equal to
√ A1 √ A2 √ A3
a) 1 b) 2 c) 3 d) 4
√A √A √A √A
386. 2 2 2 2 π
If x=X cos θ−Y sin θ , y=X sin θ+Y cos θ and x + 4 xy + y = A X + B Y , 0 ≤θ ≤ , then
2
a) θ= π b) θ= π c) A=−6 d) B=1
6 4
387. If P ( 1 ,0 ) , Q=(−1 , 0) and R=(2 , 0) are three given points, then the locus of a point S satisfying the
relation S Q 2+ S R 2=2 S P2 is
a) A straight line parallel to x -axis b) A circle through origin

c) A circle with centre at the origin d) A straight line parallel to y -axis

388. The triangle formed by x 2−3 y 2=0 and x=4 is

a) Isosceles b) Equilateral c) Right angled d) None of these

389. The angle of elevation of the top of an incomplete vertical pillar at a horizontal distance of 100 m from its
base is 45 ° . If the angle of elevation of the top of the complete pillar at the same point is to be 60 ° , then
the height of the incomplete pillar is to be increased by
a) b) 100 m c) ( ) d) ( )
50 √ 2m 100 √ 3−1 m 100 √ 3+1 m
390. If O ( 0 , 0 ) , A (4 ,0) and B(0 , 3) are the vertices of a triangle OAB , then the coordinates of the excentre
opposite to the vertex O(0 , 0) are
a) (12 , 12) b) (6 , 6) c) (3 , 3) d) None of these

391. A tower subtends an angle α at a point A in the plane of its base and angle of depression of the foot of the
tower at a point l metres just above A is β . The height of the tower is
a) l tan β cot α b) l tan α cot β c) l tan α tan β d) l cot α cot β

P a g e | 33
392. Observe the following statements
C
2 2 B
I .∈∆ ABC b cos +c cos =s
2 2
A b+c
II . ∆ ABC , cot = ⇒ B=90 °
2 2
Which of the following is correct?
a) Both I and II are true

b) I is true, II is false

c) I is false, II is false

d) Both I and II are false

393. If A and B are two points on one bank of a straight river and C , D are two other points on the other bank
of river. If direction from A to B is same as that from C to D and AB=a , ∠ CAD=α ,∠ DAB=β ,
∠ CBA=γ , then CD is equal to
a) a sin β sin γ b) a sin α sin γ c) a sin α sin β d) None of these
sin α sin(α + β + γ ) sin β sin(α + β + γ ) sin γ sin (α + β+ γ )
394. ABCD is a square plot. The angle of elevation of the top of a pole standing at D from A or C is 30 ° and
that from B is θ , then tanθ is equal to
a) b) c) d)
√6 1/ √ 6 √ 3/2 √ 2/3
395. In a ∆ ABC , if ( √ 3−1 ) a=2 b , A=3 B, then ∠ C is

a) 60 ° b) 120 ° c) 30 ° d) 45 °

396. The angle of elevation of an object from a point on the level ground is α . Moving d meters on the ground
towards the object, the angle of elevation is found to be β . Then the height (in meters) of the object is
a) d tan α b) d cot β d
c) d) d
cot α + cot β cot α −cot β
397. In a ∆ PQR as shown in figure given that x : y : z=2 :3 :6 , then the value of ∠ QPR is

a) π b) π c) π d) None of these
6 4 3
398. If by shifting the origin at (1 , 1) the coordinates of a point P become ( cos θ ,cos ϕ ) , then the original
coordinates of P were
a) 2 2
(2 cos θ/2 , 2 cos ϕ /2)
b) 2 2
(2 sin θ/ 2 ,2 sin ϕ / 2)
c) (2 cos θ /2 , 2 cos ϕ /2)

d) (2 sin θ/2 , 2sin ϕ /2)

399. In a ∆ ABC , if r 1=2 r 2=3 r 3 , then

a) a = 4 b) a = 5 c) a+ b−2 c=0 d) 2 a = b+c


b 5 b 4

P a g e | 34
400. A C
sin sin
2 2
In a ∆ ABC , if a , b , c are in AP, then the value of is
B
sin
2
a) 1 b) 1 c) 2 d) −1
2
401. The top of a hill observed from the top and bottom of a building h is at angles of elevation p and q
respectively. The height of hill is
a) h cot q b) h cot p c) h tan p d) None of these
cot q−cot p cot p−cot q tan p−tan q
402. 3
In a triangle, if b=20 , c=21 and sin A= , then a is equal to
5
a) 12 b) 13 c) 14 d) 15

403. In ∆ ABC , a=2, b = 4 and ∠ C=60 ° , then ∠ A and ∠ B are equal to

a) 90 ° , 30° b) 60 ° ,60 ° c) 30 ° , 90° d) 60 ° , 45 °

404. If in an equilateral triangle R=√ 3 cm, then the length of each side of the triangle is

a) 1 cm b) 2 cm c) 3 cm d) None of these

405. b−c cos A


In a ∆ ABC , is equal to
c−b cos A
a) sin B b) cos C c) cos B d) None of these
sin C cos B cos C
406. s (s−a) ( s−b ) (s−c )
In a ∆ ABC , if 2 s=a+b+ c, then the value of the − is equal to
bc bc
a) sin A b) cos A c) tan A d) None of these

407. The centroid of a triangle is (2, 7) and two of its vertices are (4, 8) and (−2 , 6). The third vertex is

a) (0, 0) b) (4, 7) c) (7, 4) d) (7, 7)

408. Each side of a square subtends an angle of 60 ° at the top of a tower h metres high standing in the centre of
the square. If a is the length of each side of the square, then
a) 2 2 b) 2 2 c) 2 2 d) 2 2
2 a =h 2 h =a 3 a =2h 2 h =3 a
409. 2 2 C 2 2C
In ∆ ABC , ( a−b ) cos + ( a+b ) sin is equal to
2 2
a) 2 b) 2 c) 2 d) None of these
a b c
410.
If the area of a ∆ ABC is given by ∆=a 2−( b−c )2, then tan ( A2 ) is equal to
a) −1 b) 0 c) 1 d) 1
4 2
411. If O ' (4 , 8/3) is the orthocentre of the triangle ABC the coordinates of whose vertices are
O ( 0 , 0 ) , A (8 , 0) and B ( 4 , 6 ) , then the coordinates of the orthocentre of ∆ O ' AB are
a) (0 , 0) b) (8 , 0) c) (4, 6) d) None of these

412. In a ∆ ABC , if b 2+ c2 =3 a2, then cot B+cot C−cot A is equal to

P a g e | 35
a) 1 b) ab c) 0 d) ac
4∆ 4∆
413. From the top of a tower, the angle of depression of a point on the ground is 60 ° .If the distance of this point
1
from the tower is m, then the height of the tower is
√3+ 1
a) 4 √ 3 m b) √ 3+ 3 m c) 3−√ 3 m d) √ 3 m
2 2 2 2
414. If in a ∆ ABC , a=6 cm, b=8 cm, c=10 cm, then the value of sin 2 A is

a) 6 /25 b) 8 /25 c) 10/25 d) 24 /25

415. 1+cos ( A−B ) cos C


In ∆ ABC , is equal to
1+cos ( A−C)cos B
2 2 2 2
a) a−b b) a+b c) a −b d) a +b
a+b a+ c 2
a −c
2 2
a +c
2

416. The base of a cliff is circular. From the extremities of a diameter of the base angles of elevation of the top
of the cliff are 30 ° and 60 ° . If the height of the cliff be 500 m, then the diameter of the base of the cliff is
a) 2000 m b) 1000 m c) 2000 m d)
1000 √ 3 m
√3 √3 √2
417. The sides BC , CA and AB of a triangle ABC are of lengths a , b and c respectively. If D is the mid point of
BC and AD is perpendicular to AC , then the value of cos A cos C is
2 2 2 2 2 2 2 2
a) 3(a −c ) b) 2(a −c ) c) (a −c ) d) 2(c −a )
2 ac 3 bc 3 ac 3 ac
418. The mid points of the sides of a triangle are D ( 6 , 1 ) , E(3 ,5) and F (−1 ,−2), then the vertex opposite to
D is
a) (−4 , 2) b) (−4 , 5) c) (2, 5) d) (10, 8)

419. The locus of a points which moves such that the sum of the squares of its distance from three vertices of
the triangle is constant is a/an
a) Circle b) Straight line c) Ellipse d) None of the above

420. The angles A , B and C of a ∆ ABC are in AP. if b :c =√ 3 : √ 2, then the angle A is

a) 30 ° b) 15 ° c) 75 ° d) 45 °

421. If the three points (0, 1), (0 ,−1) and (x , 0) are vertices of an equilateral triangle, then the values of x are

a) b) c) d)
√ 3 , √2 √ 3 ,−√ 3 −√ 5 , √ 3 √ 2 ,−√ 2
422. π
The sides of a triangle are sin α cos α and √ 1+sin α cos α for some0< α < . Then, the greatest angle of the
2
triangle is
a) 60 ° b) 90 ° c) 120 ° d) 150 °

423. From an aeroplane flying, vertically above a horizontal road, the angles of depression of two consecutive
stones on the same side of the aeroplane are observed to be 30 ° and 60 ° respectively. The height at which
the aeroplane is flying in km, is
a) 4 b) √ 3 c) 2 d) 2
√3 2 √3
424. A flag staff 20 m long standing on a wall 10 m high subtends an angle whose tangent is 0.5 at a point on the
ground. If θ is the angle subtended by the wall at this point, then

P a g e | 36
a) tanθ=1 b) tanθ=3 c) tanθ= 1 d) None of these
2
425. The angle of elevation of the top of a vertical tower from two points distance a and b from the base and in
the same line with it, are complimentary. If θ is the angle subtended at the top of the tower by the line
joining these points then sin θ=¿
a) a−b b) a+b c) a−b d) None of these
√2(a+b) a−b a+b
426. In a triangle with one angle if 120 ° , the length of the sides forms an AP. If the length of the greatest sides is
7 cm, then area of triangle is
a) 3 √ 15 c m 2 b) 15 √ 3 c m 2 c) 15 c m 2 d) 3 √ 3 c m 2
4 4 4 4
427. b +c c+ a a+b
If in a ∆ ABC , = = , then cos A is equal to
11 12 13
a) 1 b) 5 c) 19 d) None of these
5 7 35
428. If two vertices of an equilateral triangle are (0, 0) and (3 , 3 √ 3), then the third vertex lies at

a) (3 ,−3) b) (−3 , 3) c) (−3 , 3 3) d) None of these



429. sin ∠BAD
In an isosceles right angled ∆ ABC , ∠ B=90 ° , AD is the median, then is
sin ∠CAD
a) 1 b) c) 1 d) None of these
√2
√2
430. If P ( 1 ,2 ) , Q ( 4 , 6 ) , R(5 , 7) and S(a , b) are the vertices of a parallelogram P Q RS , then

a) a=2 , b=4 b) a=b , b=4 c) a=2 , b=3 d) a=3 , b=5

431. Angles of a triangle are in the ratio4 :1:1 . The ratio between its greatest side and perimeter is

a) 3 b) 1 c) √3 d) 2
2+ √ 3 2+ √ 3 √3+ 2 2+ √ 3
432. From the top of a cliff h metres above sea level an observer notices that angles of depression of an object
A and its image B are complementary. If the angle of depression at A is θ . The height of A above sea level
is
a) h sin θ b) h cos θ c) h sin 2 θ d) h cos 2 θ

433. The radius of the incircle of triangle when sides are 18, 24 and 30 cm is

a) 2 cm b) 4 cm c) 6 cm d) 9 cm

434. The points (1, 3) and (5, 1) are the opposite vertices of a rectangle. The other two vertices lie on the line
y=2 x +c , then the value of c will be
a) 4 b) −4 c) 2 d) −2

435. If the vertices of a triangle at O ( 0 , 0 ) , A (a , 0) and B ( 0 , a ) . Then, the distance between its circumcentre
and orthocentre is
a) a b) a c) d) a
√2 a
2 √2 4
436. The straight lines x= y , x−2 y=3 and x +2 y=−3 form a triangle, which is

a) Isosceles b) Equilateral c) Right angled d) None of these

P a g e | 37
437. The vertices of a triangle are (6, 0), (0, 6) and (6, 6). The distance between its circumcentre and centroid is

a) 2 b) c) 1 d)
√2 2 √2
438. 3
In a ∆ ABC , a=5 ,b=7 and sin A= , then the number of possible triangles are
4
a) 1 b) 0 c) 2 d) Infinite

439. The points (k , 2−2k ),(−k +1 ,2 k ),(−4−k , 6−2 k ) are collinear, then k is equal to

a) 2 , 3 b) 1 , 0 c) 1 , 1 d) 1 , 2
2
440. If the vertices P , Q, R of a ∆ PQR are rational points, which of the following points of the ∆ PQR is (are)
always rational points?
(A rational point is a point both of whose coordinates are rational numbers)
a) Centroid b) Incentre c) Circumcentre d) Orthocentre

441. Which of the following pieces of data does not uniquely determine acute angled ∆ ABC ( R=¿
circumradius)?
a) a , sin A , sin B b) a , b , c c) a , sin B , R d) a , sin A , R

442. A−B+ C
If a ∆ ABC , 2 ca sin is equal to
2
a) 2 2 2 b) 2 2 2 c) 2 2 2 d) 2 2 2
a + b −c c +a −b b −c −a c −a −b
443. In any ∆ ABC under usual notation, a (b cos C−c cos B) is equal to
2 2 2 2
a) 2 2 b) 2 2 c) b −c d) c −b
b −c c −b
2 2
444. In ∆ ABC , G is he centroid, D is the mid point of BC . If A=( 2 ,3 )and G(7 , 5), then the point D is

(2 )
a) 9 , 4
(2 )
b) 19 , 6
(2 2)
c) 11 , 11
( 2)
d) 8 , 13

445. The transformed equation of 3 x 2+3 y 2 +2 xy=2 , when the coordinate axes are rotated through an angle of
45 ° , is
a) 2 2 b) 2 2 c) 2 2 d) 2 2
x + 2 y =1 2 x + y =1 x + y =1 x + 3 y =1
446. The equation of the locus of a point equidistant from the points (a 1 , b1 ) and (a 2 , b 2) is
( a 1−a2 ) x + ( b 1−b2 ) y +c=0 , then the value of c is
a) (a2 +b2 +c 2 ) b) a 2−b2−c 2

1 1 1 1 1 1

c) 1 (a2 +b2−a2−b 2) d) None of the above


2 2 1 1
2
447. ABC is a right angled triangle with ∠ B=90 ° , a=6 cm. If the radius of the circumcircle is 5 cm. Then the
area of ∆ ABC is
a) 2 b) 2 c) 2 d) 2
25 c m 30 c m 36 c m 24 c m
448. π
The transformed equation of x 2+ 6 xy+ 8 y 2=10 when the axes are rotated through an angle is
4
a) 2 2 b) 2 2
15 x −14 xy +3 y =20 15 x +14 xy−3 y =20
c) 2 2 d) 2 2
15 x +14 xy+ 3 y =20 15 x −14 xy−3 y =20
449. The angle of elevation of the top of a TV tower from three points A , B and C in a straight line through the

P a g e | 38
foot of the tower are α ,2 α ∧3 α respectively. If AB=a , then height of the tower is

a) a tan ⁡α b) a sin α c) a sin 2α d) a sin 3 α

450. The ratio in which the x -axis divides the line segment joining (3 , 6) and (4 ,−3) is

a) 2 :1 b) 1 :2 c) 3 :4 d) None of these

451. AB is vertical tower. The point A is on the ground and C is the middle point of AB. The part CB subtend
an angle α at a point P on the ground. If AP=n AB, then the correct relation is
a) ( 2 ) b) ( 2 ) c) 2 ( 2 ) d) ( 2 )
n= n +1 tan α n= 2 n −1 tan α n = 2 n + 1 tan α n= 2 n +1 tan α
452. If the points (−2 ,−5),(2 ,−2),(8 , a) are collinear, then the value of a is

a) −5 b) 5 c) 3 d) 1
2 2 2 2
453. If in a ∆ ABC , cos A+2 cos B+cos C=2, then a , b , c are in

a) AP b) GP c) HP d) None of these

454. 31
In a ∆ ABC , a=5 , a=4 and cos ( A+ B)= . In this triangle, c is equal to
32
a) b) 36 c) 6 d) None of these
√6
455. In a triangle r 1 >r 2 >r 3 , then

a) a> b>c b) a< b<c c) a> b and b< c d) a< b and b> c

456. In a ∆ ABC , if a=2 x , b=2 y and ∠ C=120 ° , then the area of the triangle is

a) xy sq unit b) c) 3 xy sq unit d) 2 xy sq unit


xy √ 3 sq unit
457. If the points ( a , b ) ,( a' ,b ' ) and (a−a ' , b−b' ) are collinear, then

a) ' ' b) ' ' c) ' ' d) 2 2


a b =a b ab=a b a a =b b a + b =1
458. If A (−a , 0 ) and B ( a , 0 ) are two fixed points, then the locus of the point at which AB subtends a right
angle, is
a) 2 2 2 b) 2 2 2 c) 2 2 2 d) 2 2 2
x + y =2 a x − y =a x + y + a =0 x + y =a
459. If the coordinates of two vertices of an equilateral triangle ae (2 , 4) and ( 2 , 6 ) , then the coordinates of its
third vertex are
a) ( 3 , 5) b) (2 3 ,5) c) (2+ 3 , 5) d) (2 , 5)
√ √ √
460. If point (x , y ) is equidistant from (a+ b , b−a) and (a−b , a+ b), then

a) ax +by =0 b) ax−by=0 c) bx +ay =0 d) bx−ay=0

461.
If A , B ,C , D are the angles of a quadrilateral, then
∑ tan A is equal to
∑ cot A
a) b) c) d)
∏ tan A ∏ cot A ∑ tan2 A ∑ cot2 A
462. If the angles of a triangle are in the ratio 3:4:5, then the sides are in the ratio

a) b) c) d) 3 :4 :5
2 : √ 6 : √ 3+ 1 √ 2: √6 : √3+1 2 : √ 3: √3+1
463. A line joining A ( 2 , 0 ) and B(3 , 1) is rotated about A in anti-clockwise direction through 15° . Find the

P a g e | 39
equation of the line in the new position. If B goes to C in the new position, then coordinates of C are

( ) ( ) ( )
a) 2+ 1 , √ 3 b) 2− 1 , √ 3 c) 2+ 1 , √ 3 d) None of these
√ 2 √2 √2 √2 √2 2
464. ABC is a triangle with vertices A(−1, 4), B ( 6 ,−2 ) and C (−2 , 4 ) . D , E and F are the points which divide
each AB , BC , and CA respectively in the ratio 3 : 1 internally. Then, the centroid of ∆≝¿ is
a) (3, 6) b) (1, 2) c) (4, 8) d) (−3 , 6)

465. A house subtends a right angle at the window of an opposite house and the angle of elevation of the
window from the bottom of the first house is 60 ° . If the distance between the two house be 6 m, then the
height of the first house is
a) b) c) d) None of these
8 √3 m 6 √3 m 4 √3 m
466. The orthocentre of the triangle whose vertices are
¿is
a) {−a , a ( t +t +t +t +t t ) } b) {−a , a ( t +t +t +t t t ) }
1 2 3 1 2 3 1 2 3 1 2 3

c) {−a , a ( t −t −t −t t t ) } d) {−a , a ( t +t −t −t t t ) }
1 2 3 1 2 3 1 2 3 1 2 3

467. A flag staff is upon the top of a building. If at a distance of 40 m from the base of building the angles of
elevation of the topes of the flag staff and building are 60 ° and 30 ° respectively, then the height of the flag
staff is
a) 46.19 m b) 50 m c) 25 m d) None of these

468. A person observes the angle of elevation of a building as 30 ° . The persons proceeds towards the building
with a speed of 25( √ 3−1) m/h. After two hours, he observes the angle of elevation as 45° .The height of
the building (in metres)is
a) 50( 3−1) b) 50( 3+1) c) 50 d) 100
√ √
469.
In a ∆ ABC , ∑ (b +c) tan
A
2
tan
B−C
2 ( )
is equal to

a) a b) b c) c d) 0

470. In a ∆ ABC , angle A is greater than angle B. If the measures of angles A and B satisfy the equation
3
3 sin x−4 sin x −k=0 , 0<k <1, then the measure of angle C is
a) π b) π c) 2 π d) 5 π
3 2 3 6
471. In a ∆ ABC , a , c , A are given and b 1 , b2 are two values of third side b such that b=2 b1. Then, sin A is
equal to

√ √ √
9 a2−c 2 9 a2−c 2 9 a2 +c 2 d) None of these
a) b) c)
2 2 2
8a 8c 8a
472. The incentre of a triangle with vertices (7, 1), (−1 , 5 ) and (3+2 √ 3 ,3+ 4 √ 3) is

( √3 ) ( 3√3)
a) 3+ 2 , 3+ 4 b) 1+ 2 , 1+ 4 c) (7, 1) d) None of the above
√3 3 √3
473. A B
tan −tan
2 2
In any triangle ABC , is equal to
A B
tan + tan
2 2
a) a−b b) a−b c) a−b d) c
a+b 2 a+b+ c a+b

P a g e | 40
474. If orthocentre and circumcentre of triangle are respectively (1, 1) and (3, 2), then the coordinates of its
centroid are
a) 7 5
(3 , 3) b) 5 7
(3 , 3) c) (7, 5) d) None of these

475. In ∆ ABC , AD is median and ∠ A=60 °, then 4 A D2 is equal to

a) 2 2 b) 2 2 c) 2 2 d) None of these
b + c −bc 2 b +c −2 bc b + c +2 bc
476. The circumcentre of the triangle formed by the lines y=x , y=2 x and y=3 x + 4 is

a) (6, 8) b) (6 ,−8) c) (3, 4) d) (−3 ,−4)

477. A b+ c
If in a ∆ ABC , cot = , then the ∆ ABC is
2 a
a) Isosceles b) Equilateral c) Right angled d) None of these

478. Area of the triangle formed by the lines 3 x 2−4 xy + y 2=0 , 2 x− y=6 is

a) 16 sq units b) 25 sq units c) 36 sq units d) 49 sq units

479. The shadow of a tower is found to be 60 m shorter when the sun’s altitude changes from 30 ° to 60 ° . The
height of the tower from the ground is approximately equal to
a) 62 m b) 301 m c) 101 m d) 52 m

480. π
Let 0< α < be a fixed angle. If P=(cos θ , sin θ) and Q= { cos ( α −θ ) , sin ( α −θ ) }. Then Q is obtained from
2
P by
a) Clockwise rotation around the origin through b) Anti-clockwise rotation around origin through
angle α angle α
Reflection in the line through the origin with slope
c) Reflection in the line through the origin with slope d) α
tan α tan
2
481. The equation of the base of an equilateral triangle is x + y=2 and the vertex is (2 ,−1). The area of triangle
is
a) b) √ 3 c) 1 d) 2
2 √3
6 √3 √3
482. If (0, 1) is the orthocentre and (2, 3) is the centroid of a triangle. Then, its circumcentre is

a) (3, 2) b) (1, 0) c) (4, 3) d) (3, 4)

483. The angle of elevation of the top of the tower observed from each of the three points A , B ,C on the
ground forming a triangle is the same angle α . If R is the circumradius of the ∆ ABC , then the height of the
tower is
a) R sin α b) R cos α c) R cot α d) R tan α

484. The circumcentre of a triangle formed by the lines xy +2 x+ 2 y + 4=0 and x + y +2=0 is

a) (−1 ,−1) b) (0 ,−1) c) (1 , 1) d) (−1 , 0 )

P a g e | 41
485. If the distance of any point P from the points A(a+b , a−b) and B(a−b , a+ b) are equal, then the locus of
P is
a) ax +by =0 b) x− y =0 c) x + y=0 d) bx−ay=0

486. In an equilateral triangle of side 2 √ 3 cm, the circumcentre is

a) 1 cm b) c) 2 cm d)
√ 3 cm 2 √ 3 cm
487. Let ABC be a triangle, two of whose vertices are (15, 0) and (0, 10). If the orthocenter is (6, 9), then the
third vertex is
a) (15, 10) b) (10 ,−15) c) (0, 0) d) None of these

488. An aeroplane flying at a height of 300 metres above the ground passes vertically above another plane at an
instant when the angles of elevation of the two planes from the same point on the ground are 60 ° and 45 °
respectively. The height of the lower plane from the ground (in metres) is
a) b) 100 c) 50 d)
100 √ 3 150 ( √ 3+1 )
√3
489. If the sides of a triangle are in ratio 3:7:8, then R :r is equal to

a) 2:7 b) 7:2 c) 3:7 d) 7:3

490. A
In a ∆ ABC , ( b+c−a ) tan is equal to
2
a) 2 ∆ b) ∆ c) ∆ s d) s R
s s bc a
491. The sum of the radii of inscribed and circumscribed circles for an n sides regular polygon of side a , is

a) a cot π
(n ) b) a cot π
2 (2n) c) a cot π
( 2n ) d) a cot π
4 (2n)
492. If ∆=a 2−( b−c )2, where ∆ is the area of ∆ ABC , then tan A is equal to

a) 15 b) 8 c) 8 d) 1
16 17 15 2
493. The median BEand AD of a triangle with vertices A(0 ,b), B ( 0 ,0 ) , C ¿ , 0 ¿ are perpendicular to each
other, if
a) a= b b) b= a c) ab=1 d)
a=± √2 b
2 2
494. If a , b , c be the sides of a ∆ ABC and if roots of the equation a ( b−c ) x 2+b ( c−a ) x +c ( a−b )=0 are equal,

then sin ( ) ( ) ( )
2 A
2
, sin2
B
2
, sin2
C
2
are in

a) AP b) GP c) HP d) AGP

495. 1+cos ( A−B ) cos C


In ∆ ABC , is equal to
1+cos ( A−C ) cos B
2 2 2 2
a) a−b b) a+b c) a −b d) a +b
a−c a+ c 2
a −c
2
a +c
2 2

496. In a ∆ ABC , a=13 cm, b=12 cm and c=5 cm. The distance of A from BC is

a) 144 b) 65 c) 60 d) 25
13 12 13 13
497. If in a ∆ ABC , r 1< r 2 <r 3, then

P a g e | 42
a) a< b<c b) a> b>c c) b< a<c d) a< c< b

498. A ladder rests against a wall making an angle α with the horizontal. The foot of the ladder is pulled away
from the wall through a distance x , so that it slides a distance y down the wall making an angle β with the
horizontal. The correct relation is
a) x= y tan α + β
(2) b) y=x tan α + β
(2) c) x= y tan ( α + β ) d) y=x tan ( α + β )

499. Let A ( h , k ) , B (1 ,1) and C (2 ,1) be the vertices of a right angled triangle with AC as its hypotenuse. If the
area of the triangle is 1, then the set of values which ' k ' can take is given by
a) {1, 3} b) {0, 2} c) {−1 , 3 } d) {−3 ,−2}

500. x y
A variable line + =1 is such thata+ b=4 . The locus of the mid point of the portion of the line
a b
intercepted between the axes is
a) x + y=4 b) x + y=8 c) x + y=1 d) x + y=2

501.
If in a ∆ ABC , tan ( A2 ) , tan ( B2 ) , tan ( C2 ) are in HP, then the sides a , b , c are in
a) AP b) GP c) HP d) None of these

502. The shadow of tower standing on a level ground is x metres long when the sun’s altitude is 30 ° , while it is

y metres long when the altitude is 60 ° . If the height of the tower is 45. √ m, then x− y is
3
2
a) 45 m b) c) 45 m d) 45. √ 3 m
45 √ 3 m
√3 2
503. All points lying inside the triangle formed by the points (1, 3), (5, 0) and (−1 , 2) satisfy

a) 3 x+ 2 y ≥ 0 b) 2 x+ y−13<0 c) 2 x−3 y−12 ≤ 0 d) All of these

504. If the area of the triangle with vertices (x , 0),(1 , 1) and (0 , 2) is 4 sq unit, then the value of x is

a) −2 b) −4 c) −6 d) 8

505. If the centroid of the triangle formed by the points (0, 0), (cos θ ,sin θ) and (sin θ ,−cos θ) lies on the line
y=2 x , then θ is equal to
a) −1 b) −1 c) −1 d) −1
tan 2 tan 3 tan (−3) tan (−2)
506. In order to remove first degree terms from the equation 2 x 2+7 y 2 +8 x−14 y+ 4=0 , the origin is shifted
at the point
a) (−2 , 1) b) (1 , 2) c) (2 , 1) d) (1 ,−2)

507. If t 1+ t 2 +t 3=−t 1 t 2 t 3, then orthocentre of the triangle formed by the points


[ a t 1 t 2 , a ( t1 +t 2 ) ] , [ a t2 t3 , a ( t 2+ t3 ) ] and [a t3 t 1 , a(t3 +t 1)], lies on
a) (a ,0) b) (−a ,0) c) (0 , a) d) (0 ,−a)

508. If A(−5 ,0) and B(3 , 0) are two vertices of a triangle ABC . Its area is 20 sq cm. The vertex C lies on the
line x− y =2. The coordinates of C are
a) (−7 ,−5 )∨(3 ,5) b) (−3 ,−5 )∨(−5 ,7) c) ( 7 , 5 )∨(3 , 5) d) (−3 ,−5 )∨(7 ,5)

509. The locus of a point P which moves such that 2 PA=3 PB , where coordinates of points A and B are (0,0)
and (4,−3), is
a) 2 2 b) 2 2
5 x −5 y −72 x +54 y+225=0 5 x +5 y −72 x +54 y +225=0

P a g e | 43
c) 2 2 d) 2 2
5 x +5 y +72 x−54 y +225=0 5 x +5 y −72 x−54 y−225=0
510. The coordinates of the centroid of a triangle having its circumcentre and orthocentre at (7 /2 , 5/2) and
(2 , 1) respectively, are
a) (3 , 2) b) (13 /6 ,3 /2) c) (5/2 , 3/2) d) (3 /2 , 5/2)

511. 1 1
The base angle of triangle are 22 ° and112 ° . If b is the base and h is the height of the triangle, then
2 2
a) b=2 h b) b=3 h c) d)
b=( 1+ √ 3 ) h b=( 2+ √ 3 ) h
512. π
Let ABC be a triangle such that ∠ ACB= and let a , b and c denote the lengths of the sides opposite to
6
A , B and C respectively. The value (s) of x for which a=x 2+ x+1 , b=x2 −1 and c=2 x +1 is (are)
a) −(2+ 3) b) c) d)
√ 1+ √ 3 2+ √ 3 4 √3
513. If the distance between the points P(a cos 48 ° , 0) and Q(0 , a cos 12° ) is d , then d 2−a2=¿
2 2 2
a) a ( √5−1 ) b) a ( √5+1 ) c) a ( √ 5−1 ) d) a ( √5+1 )
4 4 8 8
514. In a ∆ ABC , a( b cos C−c cos B) is equal to

a) 2 b) 2 2 c) 0 d) None of these
a b −c
515. If the points ( a 1 , b 1) ,( a2 , b2) and ( a 3 ,b 3 ) are collinear, then lines a i x +bi y +1=0 for i=1 , 2 ,3 are

a) Concurrent b) Identical c) Parallel d) None of these

516. In ∆ ABC , ( b+c ) cos A+ ( c+ a ) cos B+ ( a+ b ) cos C is equal to

a) 0 b) 1 c) a+ b+c d) 2(a+b+ c)

517. π
If in a ∆ ABC a=5 , b=4 , A= + B , then C
2
a) is tan−1 1
( 9) b) is tan −1 9
( 40 ) c) Cannot be evaluated d) is 2 tan−1 1
(9)
518. If p1 , p2 , p 3 are altitudes of a ∆ ABC drawn from the vertices A , B ,C and ∆ the area of the triangle, then
−2 −2 −2
p1 + p2 + p3 is equal to
a) a+b+ c
2
b) a +b + c
2 2 2
c) a +b + c
2 2
d) None of these
∆ 4∆
2

2

P a g e | 44
17.CO-ORDINATE GEOMETRY

: ANSWER KEY :

1) a 2) a 3) d 4) b 177) d 178) a 179) c 180) c


5) a 6) b 7) d 8) a 181) b 182) a 183) d 184) d
9) d 10) b 11) b 12) a 185) b 186) a 187) a 188) d
13) c 14) c 15) b 16) d 189) a 190) a 191) b 192) a
17) c 18) d 19) c 20) b 193) d 194) d 195) b 196) d
21) c 22) a 23) c 24) a 197) c 198) b 199) c 200) a
25) c 26) c 27) b 28) b 201) b 202) a 203) a 204) c
29) a 30) a 31) c 32) b 205) a 206) b 207) a 208) a
33) b 34) a 35) b 36) a 209) c 210) d 211) c 212) c
37) b 38) d 39) d 40) a 213) b 214) b 215) c 216) d
41) a 42) a 43) c 44) c 217) a 218) d 219) b 220) d
45) c 46) a 47) d 48) a 221) a 222) b 223) b 224) c
49) d 50) a 51) d 52) b 225) c 226) c 227) a 228) d
53) a 54) d 55) b 56) a 229) a 230) a 231) a 232) c
57) c 58) d 59) a 60) b 233) c 234) d 235) d 236) a
61) b 62) d 63) a 64) c 237) a 238) d 239) a 240) d
65) c 66) a 67) b 68) d 241) c 242) c 243) a 244) c
69) c 70) d 71) c 72) c 245) a 246) a 247) d 248) d
73) a 74) a 75) d 76) b 249) d 250) d 251) d 252) b
77) c 78) b 79) d 80) a 253) c 254) c 255) c 256) d
81) b 82) d 83) a 84) c 257) a 258) a 259) c 260) c
85) a 86) d 87) c 88) c 261) a 262) c 263) d 264) b
89) c 90) c 91) a 92) d 265) b 266) a 267) c 268) b
93) c 94) c 95) a 96) a 269) d 270) c 271) a 272) d
97) b 98) c 99) c 100) c 273) a 274) d 275) d 276) b
101) a 102) a 103) a 104) b 277) a 278) b 279) b 280) c
105) c 106) b 107) c 108) d 281) c 282) b 283) c 284) c
109) b 110) a 111) c 112) b 285) a 286) d 287) b 288) a
113) a 114) b 115) d 116) b 289) a 290) d 291) b 292) d
117) a 118) a 119) b 120) b 293) a 294) a 295) d 296) c
121) a 122) c 123) d 124) b 297) c 298) a 299) a 300) b
125) c 126) b 127) d 128) d 301) d 302) a 303) b 304) b
129) c 130) c 131) c 132) b 305) a 306) c 307) d 308) d
133) b 134) d 135) c 136) c 309) c 310) a 311) d 312) c
137) b 138) c 139) d 140) c 313) b 314) a 315) d 316) c
141) c 142) b 143) d 144) d 317) a 318) b 319) a 320) d
145) c 146) d 147) b 148) c 321) b 322) c 323) c 324) b
149) b 150) a 151) c 152) a 325) b 326) b 327) d 328) a
153) d 154) b 155) a 156) b 329) d 330) c 331) d 332) a
157) c 158) a 159) c 160) d 333) d 334) d 335) a 336) a
161) d 162) c 163) d 164) d 337) a 338) b 339) b 340) c
165) c 166) b 167) a 168) a 341) c 342) b 343) a 344) d
169) b 170) c 171) c 172) a 345) c 346) d 347) d 348) a
173) c 174) c 175) a 176) a 349) d 350) a 351) c 352) a

P a g e | 45
353) b 354) b 355) b 356) c 437) b 438) b 439) c 440) a
357) c 358) c 359) d 360) d 441) d 442) b 443) a 444) b
361) b 362) b 363) b 364) a 445) b 446) c 447) d 448) c
365) b 366) a 367) c 368) b 449) c 450) a 451) d 452) b
369) d 370) a 371) b 372) a 453) a 454) d 455) a 456) b
373) d 374) b 375) b 376) b 457) a 458) d 459) c 460) d
377) a 378) c 379) a 380) d 461) a 462) a 463) a 464) b
381) d 382) a 383) d 384) a 465) a 466) b 467) a 468) c
385) a 386) b 387) d 388) b 469) d 470) c 471) b 472) a
389) c 390) b 391) b 392) b 473) b 474) a 475) d 476) b
393) b 394) b 395) b 396) d 477) c 478) c 479) d 480) d
397) b 398) a 399) b 400) b 481) b 482) d 483) d 484) a
401) b 402) b 403) c 404) c 485) b 486) c 487) c 488) a
405) b 406) b 407) b 408) b 489) b 490) a 491) b 492) c
409) c 410) c 411) a 412) c 493) d 494) c 495) d 496) c
413) c 414) d 415) d 416) a 497) a 498) a 499) c 500) d
417) d 418) a 419) a 420) c 501) a 502) a 503) d 504) c
421) b 422) c 423) b 424) a 505) c 506) a 507) b 508) d
425) c 426) b 427) a 428) c 509) b 510) a 511) a 512) b
429) b 430) c 431) c 432) d 513) d 514) b 515) a 516) c
433) c 434) b 435) b 436) d 517) b 518) b

P a g e | 46
17.CO-ORDINATE GEOMETRY

: HINTS AND SOLUTIONS :

1 (a)
2 2 2
a
a ( cos B−cos C ) + b ( cos C−cos A ) + c (cos A−cos B)
2
22 2
a
( cos α +cos β ) , (sin α + sin β )
2 2 2 [
2 ]
¿ a 2 ( 1−sin2 B−1+ sin2 C ) +b 2 ( 1−sin2 C−1+sin 2 A )
5 (a)
2 2 2
+c (1−sin A−1+sin B) Area of
¿ a 2 ( sin 2 C−sin2 B ) +b2 ( sin2 A−sin2 C ) +c 2 ( sin2 B−sin2 A∆) PBC
¿ k a ( c −b ) +k b ( a −c ) +k c (b −c )
2 2 2 2 2 2 2 2 2 2 2 2
∆ ABC
=
[ {−3 (−2− y ) + 4 ( y−5 )+ x (5+ 2)}
{6 ( 5+2 )−3 (−2−3 ) +4 (3−5)} ]
|7 x+ 749y−14|=|x + y7−2|
¿0
¿
2 (a)
Let sin A=3 k , sin B=4 k ,sin C=5 k 6 (b)
sin A sin B sinC Let PQ and RS be the poles of height 20 m and 80
∵ = = = p [say]
a b c m subtending angles α and β at R and P
3k 4 k 5k respectively. Let h be the height of the point T , the
⇒ = = =p
a b c intersection of QR and PS
⇒ a=3
k
p
, b=4 ()
k
p
, c=5
k
p () ()
⇒ a=3 l , b=4 l , c=5l let l=
[ k
p ]
2 2 2
b + c −a
∴ cos A=
2 bc
16+25−9 32 4 Then, PR=hcot α +h cot β
¿ = = ¿ 20 cot α =80 cot β
2× 4 × 5 40 5
2 2 2 ⇒ cot α=4 cot β
c + a −b
∴ cos B= cot α
2 ac ⇒ =4
cot β
25+9−16 18 3
¿ = = Again, h cot α +h cot β=20 cot α
2 ×3 ×5 30 5
4 3 ⇒ ( h−20 ) cot α =−h cot β
Now, cos A : cos B= : =4 :3 cot α h
5 5 ⇒ = =4
cot β 20−h
4 (b) ⇒ h=80−4 h
Slope of perpendicular to the line joining the ⇒ h=16 m
points
(a cos α , a sin α ) and 8 (a)
−cos α−cos β Since, α , β , γ are the roots of the equation
( a cos β , a sin β )= 3 2
x −3 p x +3 qx−1=0
sin α −sin β
α +β ∴ α + β + γ=3 p
¿ tan αβ + βγ +γα =3 q
2
Hence, equation of perpendicular is and αβγ=1
Let G(x , y) be the centroid of the given triangle
y=tan ( α +2 β ) x …(i)
∴ x=
α + β +γ
=p
Now, on solving the equation of line with Eq. (i), 3
we get

P a g e | 47
1 1 1 points, then
+ +
and α β γ OP × OQ cos ∠ POQ=x 1 x 2 + y 1 y 2
y=
3 ∴ OP ×OQ × sin ∠ POQ
βγ + γα+ αβ ¿ √ O P2 ×O Q2−O P2 × OQ2 ×cos 2 ∠ POQ
¿ =q
3 αβγ
Hence, coordinates of the centroid of triangle are
¿ √ ( x + y )( x + y )−( x x + y y )
2
1
2
1
2
2
2
2 1 2 1 2
2

¿ √ ( x y −x y ) =¿ x y −x y ∨¿
2
( p , q) 1 2 2 1 1 2 2 1

9 (d) 14 (c)
Let O(0 , 0) be the orthocenter, A(h , k) be the ( 3 )2 + ( 5 )2 −( 4 )2 3
third vertex and B(−2 , 3) and C (5 ,−1) the cos B= =
2 ×3 ×5 5


other two vertices. Then, the slope of the line 9 4
k ⇒ sin B= 1− =
through A and O is , while the line through B 25 5
h ∴ sin 2 B=2sin B cos B
(−1−3) −4 4 3 24
and C has the slope = . By the ¿2× × =
(5+ 2) 7 5 5 25
property of the orthocenter, these two lines must
be perpendicular, so we have 16 (d)
Given that, ∠ A=45 ° ,∠ B=75°
( kh )(−47 )=−1 ⇒ kh = 74 …(i) ∠ c=180 °−45 °−75 °=60 °
5−2+h −1+3+k ∴ a+c √ 2=k (sin A + √ 2sin C )
Also, + =7
3 3 ¿ k (sin 45 ° + √ 2sin 60 ° )

( √12 + √2 √23 )=k ( 1+√ √23 )


⇒ h+ k=16 …(ii)
Which is not satisfied by the points given in the ¿k …(i)
options (a), (b) or (c) b
And k =
10 (b) sin B
= √
Let (h , k ) be the point b 2 2b
¿
According to question, sin 75° √ 3+1
On putting the value of k in Eq. (i), we get
4 √ ( h−h ) + k=h 2+ k 2
2

a+ c √ 2=2 b
⇒ 4|k|=h + k
2 2

Locus of the point is 18 (d)


4 | y|=x + y ⇒ x + y −4| y|=0
2 2 2 2
From figure ABCD is s square

12 (a)
Given points are P ( 4 ,−2 ) , A (2 ,−4) and B(7 , 1)
Suppose P divides AB in the ratio λ :1. Then,
7 λ+2 2
=4 ⇒ λ=
λ+1 3
Thus, P divides AB internally in the ratio 2 :3
Whose diagonals AC and BD are of length 2 unit
The coordinates of the point dividing AB
1
externally in the ratio 2 :3 are Hence, required area¿ AC × BD
2
( 2× 7−3× 2 2×1−3 ×−4
2−3
,
2−3 )
=(−8 ,−14) 1
¿ ×2 ×2=2 sq units
2
Hence, the harmonic conjugate of R with respect
to A and B is (−8 ,−14) 19 (c)
In ∆ APD ,
13 (c) a
tan 45 °= ⇒ AP=a
If O is the origin and P ( x 1 , y 1 ) ,Q (x2 , y 2) are two AP

P a g e | 48
C 22 (a)
D
Let (x , y ) be the coordinates of vertex C and
E
b (x 1 , y 1 ) be the coordinates of centroid of the
a
45o 45o
triangle.
A P B
x +2−2 y−3+1
∴ x1= and y 1=
and in ∆ BPC , 3 3
b x y−2
tan 45 °= ⇒ x 1= and y 1=
PB 3 3
⇒ PB=b Since, the centriod lies on the line 2 x+3 y =1
∴ DE=a+ b∧CE=b−a ∴ 2 x 1 +3 y 1=1
In ∆ DEC , 2 x 3 ( y −2 )
2 2 2 ⇒ + =1
D C = D E +E C 3 3
¿ ( a+ b2 ) + ( b−a2 ) ⇒ 2 x +3 y=9
2
¿ 2(a + b )
2 This equation represents the locus of the vertex C

20 (b) 23 (c)
If the axes are rotated through 30 ° , we have Let the height of the tower be h
P
x=X cos 30° −Y sin30 °= √
3 X−4
2
X +√ 3 Y h
¿ , y= X sin 30 ° +Y cos 30 °=
2
Substituting these values in 30o 60o
B 20 m A O
x + 2 √3 xy− y =2 a , we get
2 2 2

( √ 3 X−Y ) + 2 √ 3 ( √ 3 X−Y ) ( X + √3 Y ) −( X + √ 3 Y ) =8 a2In ∆ PAO , tan 60° = h


2 2

2 2 2 OA
⇒ X −Y =a
h
⇒ OA =h cot 60 °= …(i)
21 (c) √3
Since, F , E and D are the mid points of the sides h
In ∆ PBO , tan 30° =
AB , AC and BC of triangle ABC respectively, OB
then the vertices of triangle are h
⇒ OB=
A ( 0 , 0 ) , B ( 0 , 2 ) ,C (2 , 0) 1
√3
⇒ AB+ AO=√ 3 h
h
⇒ 20+ =√ 3 h[using Eq.(i)]
√3
20
⇒ h=
1
√3−
Now, AB=c= √ 0 2+22 =2 √3
⇒ h= √
BC=a= √ 22 +22=2 √ 2 20 3
2
And CA =b=√ 22 +0 2=2
⇒ h=10 √ 3 m
∴ x coordinates of incentre
a x 1+b x 2 +c x3 24 (a)
¿


a+b+ c ( s−b ) ( s−c )
We have, sin A =
2 √ 2 ( 0 ) +2 ( 0 ) +2(2) 2 bc
¿
2 √ 2+2+ 2 2 A
⇒ bc sin = ( s−b ) (s−c )
2 2
¿ =2−√ 2
2+ √2

P a g e | 49
2 A CD
On comparing with x sin =( s−b ) (s−c) and in ∆ ADC , tan α =
2 AC
We get, x=bc ⇒ CD=b cot β tan α
D
25 (c)
2 2 2
4a a cos 2 α
∴ p 21+ p 22= 2 2
+ 2 2
sec α + cose c α cos α + sin α M N

( )
2 2 2
2 4 cos α sin α cos 2 α
¿a 2 2
+
cos α +sin α 1 b

¿ a 2 ( sin 2 2 α + cos2 2 α ) =a2 A C


2 2
and p1 p 2=a sin 2 α cos 2α =
4 2 2
( 14 ) a sin 4 α
4 2
31 (c)
2 2 b 2 2
p1 p2 2 ( p1 + p2 )
( )
2
Since, R= = =
∴ + = 2 2 2 sin B 2 sin 30 ° 1
p2 p1 p1 p2 Area of circumcircle¿ π R 2
4 2 2
¿ π × ( 2 ) =4 π sq unit
¿ 2 =4 cose c 4 α
sin 4 α
33 (b)
26 (c) 2 2 2
b2−c2 4 R (sin B−sin C )
Let A ( 2 ,1 ) , B(−2 , 4) =
2 aR 4 R2 sin A
∴ AB=5
sin(B+C)sin(B−C )
Hence, the locus is the line segment AB ¿
sin A
27 (b) ¿ sin(B−C)
1 1 1 1
2
+ 2+ 2 + 2 34 (a)
r r1 r2 r 3
Given curve is
2 2 2 2
s + ( s−a ) + ( s−b ) + ( s−c ) y=1−¿ x∨¿
¿
∆2
2 2 2 2
4 s + a + b +c −2 s (a +b+c )
¿
∆2
2 2 2
a +b + c
¿ 2

28 (b)
Let a=4 cm ,b=5 cm and c=6 cm
4 +5+6 15 ∴ Area of ∆ ABC=2 area of ∆ AOB
∴ s= =
2 2 1
Hence, area of triangle ¿ √ s ( s−a )( s−b ) ( s−c ) ¿ 2 × ×1 ×1=1 sq unit
2

¿ (√ 152 )( 152 −4)( 152 −5)( 152 −6) 35 (b)


Given, angles A , B ,C of ∆ ABC are in AP with d
¿
√ 15 7 5 3 15
× × × = √7 c m2
2 2 2 2 4
(common difference)¿ 15 °
∴ B=A +15 ° and C= A+30 °
Also, A+ B+C=180 °
30 (a)
Let CD be the tower
⇒ A+ A +15 °+ A +30 °=180 °
b ⇒ ∠ A=45 °
In ∆ ACM , tan β= ∴ ∠ B=45 ° +15 °=60 °
AC
⇒ AC =b cot β 36 (a)

P a g e | 50
( )( )
r1 r1 41 (a)
Since, 1− 1− =2 3 2
r2 r3 Given that, cot α = and cot β=
5 5
(
∴ 1−
s−b
s−a )(
1−
s−c
s−a
=2 ) In ∆ BCD , tan β=
h
BC
( b−a ) (c−a) D
⇒ =2
( s−a )2
2 2 2 2
⇒ 2 bc−2 ab−2 ac+ 2 a =b + c + a + 2bc −2 ab−2 ac
2 2 2 h
⇒ a =b + c
So, triangle is right angled
A 32 m B C
37 (b)
2h
Let the height of the tower be BC=h ,then length ⇒ BC =h cot β ⇒ BC = …(i)
5
of shadow of tower AB=√ 3 h.
h
BC and in ∆ ACD , tan α =
In ∆ ABC , tan α = 32+ BC
AB
⇒ tan α =
h
√3 h
⇒ h= 32+ (2h 5
5 3 )
[using Eq.(i)]

⇒ 3 h=160+2 h
⇒ tan α =tan 30 °
⇒ h=160 m
⇒ α=30 °
42 (a)
The vertices of quadrilateral ABCD is
A ( 2 ,3 ) , B ( 3 , 4 ) , C ( 4 , 5 )
and D(5 , 6)
∴ AB=√ (3−2 ) + ( 4−3 )
2 2

¿ √ ( 1 ) + ( 1 ) =√ 2
2 2

38 (d) Similarly, BC= √ 2 , CD=√ 2 ,∧DA=3 √ 2


Let the coordinates of P be ( h , k ) . Then, ∴ a=b=c=√ 2∧d=3 √ 2
x=X + h , y=Y +k a+ b+c +d
Substituting these in 2 x 2+ y 2 −4 x−4 y=0 , we and s=
2
get √2+ √2+ √ 2+3 √ 2
2 2 2 2 ¿
2 X +Y +4 ( h−1 ) ×+ 2 ( k−2 ) Y +2 h +k −4 h−4 k=0 2
Comparing this equation with
¿ √ =3 √ 2
6 2
2 2
2 X +Y −8 X−8 Y +18=0 , we get 2
h−1=−2 , ( k −2 )=−4 and ∴Area of quadrilateral
2 2
2 h +k −4 h−4 k=18 ¿ √ ( s−a ) ( s−b )( s−c ) (s−d)
⇒ h=−1 , k=−2 ¿ √ ( 3 √ 2−√ 2 )( 3 √ 2− √ 2 )
40 (a) ( 3 √ 2−√ 2 ) (3 √ 2−√ 2)


¿0
C ( s−a ) (s−b)
We have, tan =
2 s(s−c ) 43 (c)
∵ ( s−a )( s−b ) =s ( s−c ) (given) 1
We have, ∆= bc sin A


C s (s−c) 2
∴ tan = 1 2
2 s (s−c) ⇒ k sin B sinC sin A=∆ …(i)
C π 2
⇒ tan =tan ∴ a2 sin 2 B+b 2 sin 2 A
2 4
2 2
⇒ ∠ C=90 ° ¿ 2( a sin B cos B+b sin A cos A)

P a g e | 51
2 2 2
¿ 2 k (sin A sin B cos B+ sin B sin A cos A) 44 (c)
2
¿ 2 k ( sin A sin B sin C )=4 ∆[¿ Eq .(i)] sin A sin ( A−B )
1. =
sin C sin ( B−C )

⇒ sin (B+ C)sin ( B−C ) =sin( A+ B)sin ( A−B )


2 2 2 2
⇒ sin B−sin C=sin A−sin B
2 2 2 2
⇒ b −c =a −b
2 2 2
⇒ 2 b =a +c
2 2 2
⇒ a ,b , c are in AP

2. r 1 , r 2 , r 3 are in HP

1 1 1
⇒ , , are in AP
r1 r2 r3

2 1 1
⇒ = +
r2 r1 r3

⇒ 2 ( s−b )=s−a+ s−c

⇒ 2 b=( a+ c )

⇒ a ,b , c are in AP

Hence, both of these statements are correct

45 (c)
The largest side of triangle is √ p 2+ q2 + pq
Greatest angle will be opposite to largest side. Let
θ be greatest angle, then
2 2 2 2
p + q − p −q −pq −1
cos θ= =
2 pq 2

⇒ θ=
3

46 (a)
Let area of triangle be ∆ , then according to
question
1 1 1
∆= ax= by = c z
2 2 2
bx cy a z b 2 ∆ c 2 ∆ a 2 ∆
∴ + + =
c a b c a
+
a b
+
b c ( ) ( ) ( )
2 2 2
2 ∆(b + c + a )
¿
abc
2(a +b 2+ c 2) abc
( )
2
abc
¿ ∙ ∵ ∆=
abc 4R 4R

P a g e | 52
2 2
a +b + c
2
49 (d)
¿
2R In ∆ ABC the vertices are A (−3 , 0 ) , B ( 4 ,−1 ) and
C ( 5 , 2)
A (-3, 0)

(4, -1)B L C(5, 2)

∴ BC=√ ( 5−4 ) + ( 2+1 )


2 2

¿ √ 1+9= √ 10
Area of ∆ ABC
1
¿ [ x 1 ( y 2 − y 3 ) + x 2 ( y 3− y 1) + x3 ( y 1− y 2)]
2
1
¿ [ −3 (−1−2 ) + 4 ( 2−0 )+ 5 ( 0+1 ) ]
2
1
¿ [ 9+8+5 ] =11
2
1
As we know that, area of ∆= × BC × AL
2
1
⇒ 11= × √ 10 × AL
2
2× 11 22
⇒ AL= =
√10 √10
50 (a)
As the line divides the ∆ ABC in equal to area.
Mid point of AB(51 ,30) which lies on y=kx

30
∴ 30=51 k ⇒ k =
51

52 (b)
Let (h , k ) be the point

According to question, 4 ( h−h )2+ k 2=h2 + k 2
⇒ 4|k|=h + k
2 2

Locus of the point is 4 | y|=x 2 + y 2


⇒ x + y −4| y|=0
2 2

53 (a)
h
In ∆ CBD , tan 60 °=
x

P a g e | 53
3 1
⇒ 1+sin 2 A= ⇒ sin 2 A=
2 2

⇒ 2 A=30° ⇒ ∠ A=15 °

So, it can form a triangle


⇒ h=x √ 3 …(i)
55 (b)
h
and in ∆ CAD , tan 30 °= h−a
40+ x In ∆ PMC , tan α =
PM
⇒ h √ 3=40+ x
⇒ 3 x=40+ x [from Eq. (i)]
⇒ x=20 m

54 (d)
(a) We know,
tan A+ tan B+ tanC=tan A tan B tan C

Since, tan A+ tan B+ tanC=0

⇒ Let either of tan A , tan B or tanC is zero ie ,


one angle is 0
⇒ PM =( h−a ) cot α …(i)
So, it cannot be a triangle h+a
In ∆ PMC ' , tan β=
sin A sin B sin C PM
(b) = = ⇒ h+ a=PM tan β
2 3 1
⇒ h=( h−a ) cot α tan β−a
⇒ a: b :c=2 :3 :1 ⇒ h ( 1−cot α cot β )=−a(1+cot α tan β)
Let a=2 k , b=3 k , c=k , a+c=b (so triangle not
a ( sin α cos β+cos α sin β )
⇒ h=
possible) sin β cos α−sin α cos β
a sin(α + β )
¿ m
( c ) sin A sin B=
√3 =cos A cos B sin(β−α )
4
56 (a)
Either sin A , sin B are both positive or both Line perpendicular to OA passing through B is
negative but, if both are positive sin A+sin B>0 x=4
but sin A+sin B is negative so both negative but, y
B(4, 6)
if both are negative, then ∠ A and ∠ B are more
than 90 ° , so it cannot be a triangle
O (0, 0)
x
A (8, 0)
(d) ( a+ b )2=c2 +ab
−3
2 2 2 Slope of AB=
⇒ a + b +2 ab=c + ab 2
2 2 2
Line perpendicular to AB through origin is
⇒ a + b −c =−ab 2
y= x
2 2 2 3
a +b −c −1
⇒ = =cos C ∴ The point of intersection of a line x=4 and
2 ab 2

⇒ ∠ C=120 °
2
y= x is 4 ,
3 ( )
8
3

sin A+cos A= √
3 57 (c)
√2 Since, (0, 1), (1, 1) and (1, 0) are mid points of

P a g e | 54
sides AB , BC and CA respectively x
In ∆ BAC , tan θ= …(i)
A (0, 0) y
x+ h
In ∆ DAB , tan 2θ=
c b y

(0, 2)B a C(2, 0)



2 tan θ
=
x +h

2 ( xy ) = x +h [from Eq. (i)]
∴ Coordinates of A , B and C are (0, 0), (0, 2) 1−tan θ
2
y x
2
y
and (2, 0) respectively 1− 2
y
Now, AB=2 , BC =2 √ 2 , CA =2
⇒2 x y 2 −x y 2 + x 3=( y 2−x 2 ) h
∴ x-coordinate of incentre 2 2
x(x + y )
¿
0+0+ 2.2
2+ 2 √2+2 ( a x +b x 2 +c x 3
∵ x= 1
a +b+c ) ⇒ h= 2 2
( y −x )
2 61 (b)
¿ =2−√ 2
2+ √2

58 (d)
2 ac sin ( A−B+C
2 )
¿ 2 ac sin ( )
Let P(x , y) is equidistant from the mid points 180° −2 B
A(a+b , b−a) and (a−b , a+ b) 2
2 2 2
∴ P A 2=P B2 ¿ 2 ac sin (90 °−B)=2 ac cos B=a + c −b
2 2 2 2
⇒ ( a+b−x ) + ( b−a− y ) = ( a−b−x ) + ( a+ b− y ) 62 (d)
⇒ bx −ay=0
( x +1 )2+ y 2+ ( x −2 )2+ y 2=2 [ ( x−1 )2+ y 2 ]
59 (a) On simplification, we get 2 x+3=0
Let the locus of a point in a plane be P(h , k )
63 (a)
y
We know that centroid divides the line segment
joining orthocenter and circumcentre in the ratio
A P (h, k) 2 :1. Since, the coordinates of orthocenter and
circumcentre are (1, 1) and (3, 2) respectively
∴ The coordinates of centroid are
x'
( 2.3+1.1 )=( , )
x 2.2+1.1 7 5
B ,
2+1 2+1 3 3
y'
According to the question, 64 (c)
|PA|+|PB|=1 ⇒|h|+|k|=1 Given equation are
Hence, locus of a point is x cotθ + y cosecθ=2 …(i)
|x|+| y|=1 And x cosecθ + y cotθ=6 …(ii)
On squaring and subtracting Eq. (i) from Eq. (ii),
Which represents the equation of square
we get
x 2 (cose c 2 θ−cot 2 θ)+ y 2 ( cot 2 θ−cose c 2 θ ) =( 6 ) − ( 2 )
2 2
60 (b)
Let BC be the height of tower and CD be height of 2 2
⇒ x − y =32
the flagstaff It represents an equation of hyperbola

65 (c)
Since, cot A+ cotC=2 cot B
cos A cos C 2 cos B
⇒ + =
sin A sin C sin B
2 2 2 2 2 2 2 2 2
b +c −a a + b −c a + c −b
⇒ + =2
2 bc(ka) 2 ab(kc) 2 ac (kb)

P a g e | 55
2 2
⇒ a + c =2 b
2
66 (a)
Hence, a 2 , b2 , c 2 are in AP In ∆ ABC , AC= ( H −h ) cot 15 ° …(i)
and in ∆ ACD , AC= ( H + h ) cot 45 ° …(ii)

From Eqs. (i) and (ii).


( H−h ) cot 15° =( H+ h ) cot 45 °
⇒ H =h ¿ ¿
2500(2+ √ 3+1)
∴ H=
(2+ √ 3−1)
¿ 2500 ¿ ¿
¿ 2500 √ 3 m

67 (b)
Let OP be the clock tower standing at the mid
point O of side BC of ∆ ABC . Let
−1
α =∠ PAO=cot 3.2 and
−1
β=∠ PBO=cose c 2.6
Then, cot α =3.2∧cosec β=2.6
∴ cot β=√ cose c2 β−1= √( 2.6 ) −1=2.4
2

In ∆ PAO and ∆ PBO ,we have


C P
90o -

O
90o
m
100

90o

A B
100 m

AO=h cot α =3.2 h


and BO=h cot B=2.4 h
In ∆ ABO , A B2=O A 2 +O B2
2 2 2
⇒ 100 =( 3.2h ) + ( 2.4 h )
2 2
⇒ 100 =16 h
2
⇒ h =625 ⇒ h=25 m

68 (d)

[ ]
2 2 2 2
3+1−a b + c −a
∵ cos 30 °= ∵ cos A=
2 √3 2bc
√ 3 4−a ⇒ a2=1
⇒ =
2

2 2 √3
⇒ a=1
Here, we see side b is largest, so
∠ B must be greatest

P a g e | 56
b
=
a 15 √ 7 2
∴ By sine rule, x
sin B sin A 4
Inradius, r =
⇒ √3 =1 15
x
sin B sin 30 ° 2
√3 ¿√ x
7
⇒ sin B=
2 2
⇒ ∠ B=120 ° 8x
R
=
√ 7 = 16
69 (c)
r √7 x 7
Let each side of equilateral triangle ¿ a
2
∴ ∆= √ a2 , S=
3 3a
4 2 72 (c)

Now, r = =
∆ √3 a2 ∙ 2 = a In ∆ DAP , tan 60 °=
1
[ ∵ E Q=DP=1]
5 4 3 a 2 √3 AP
3
abc a a
R= = =
4 ∆ √ 3 a √3 2

= √ a2 ∙ = √ a
∆ 3 2 3
r 1=
s−a 4 a 2
a a √3
∴ R : r 1 :r 1= : : a 1
√ 3 2 √3 2 ⇒ AP=
¿ 2 :1:3
√3
EQ
In ∆ EA Q , tan 30 °=
70 (d) AP+ P Q
Given, r 1=2 r 2=3r 3 1
⇒ + P Q= √ 3
∆ 2∆ 3∆ ∆ √3
∴ = = = [say ] 2
s−a s−b s−c k ⇒ P Q= km
Then, s−a=k , s−b=2 k , s−c=3 k √3
⇒ 3 s−( a+b+ c )=6 k ⇒ s=6 k Distance
∴ Speed of plane=
a b c Time
∴ = = =k 2
5 4 3
¿ √
a b c 5 4 3 191 3
∴ + + = + + = =240 √ 3 km/h
b c a 4 3 5 60 10
60 × 60
71 (c)
Let sides of the triangle are 4 x , 5 x , 6 x 74 (a)

| |
4 x+ 5 x +6 x 15 x y1 1
s= = x 1 1
2 2 Area of triangle ¿ x 2 y2 1
2


x3 y3 1
∆=
15 15
2
×
2(x−4 x
15
2 )(
x−5 x
15
2 )(
x−6 x ) = A rational number if vertices have integral


15 7 5 3 coordinates only
¿ x× x× x × x If triangle is equilateral, then area
2 2 2 2
¿ √ [ ( x 1−x 2 ) + ( y 1− y 2 ) ]
15 √ 7 x
2 3 2 2
¿ 4
4
4 x × 5 x ×6 x =irrational quantity
R= So, triangle cannot be equilateral
15 √ 7 x
2
Circumradius,

4 75 (d)
8 b−c k (sin B+ sinC )
¿ x =
√7 a k sin A

P a g e | 57
¿
2sin
B+C
(
2 ) (
cos
B−C
2 ) In ∆ AOC , tan θ1=
AC
=
AO 160
h
… (ii)

A A h
2 sin cos and in ∆ AOB , tan ( θ1+ θ2 )=
2 2 40
tan θ1 + tan θ2
( )
B−C h
cos ⇒ =
b+ c 2 1−tan θ1 tan θ2 40
⇒ =
a A h 3
sin +
2 160 5 h
⇒ = [from Eqs. (i) and (ii)]
h 3 40

Similarly,
b−c
=
sin (B−C
2 ) 1−
160 5
×

a A 5[h+96 ] h
cos ⇒ =
2 800−3 h 40
2
⇒ h −200 h+6400=0
76 (b) ⇒ ( h−160 ) ( h−40 ) =0
−1 3 3 ⇒ h=160∨h=40
Given, θ=tan ⇒ tan θ2= … (i)
5 5 Hence, height of the vertical pole is 40 m

77 (c)
Let A ( x 1 , y 1 ) and B ( x2 , y 2 ) be two points. Let C and D be the points of internal and external division of
AB in the ratio λ :1. Then, the coordinates of C and D are

(λ x 2+ x1 λ y 2 + y 1
λ +1
,
λ+1 )(

λ x 2−x1 λ y 2− y 1
λ−1
,
λ−1 )
respectively

λ λ
∴ AC = AB∧ AD= AB
λ+1 λ−1
1 1 2
Clearly, + = ⇒ AC , AB , AD are in H.P.
AC AD AB

79 (d) points A ( 3 , 0 )
We have, and B (−3 ,0 ) is 4ie , PA−PB=4
A+ B+C=180 °
⇒ √ ( h−3 ) +k − √ ( h+ 3 ) + k =4
2 2 2 2
⇒ 3 B=180 [ ∵ A , B ,C are∈ AP]
P (h, k)
⇒ B=60°
1
⇒ cos B=
2
2 2 2
B (-3, 0) A (3, 0)
A B + B C −A C 1
⇒ = ⇒ √ ( h−3 ) +k 2=4+ √ ( h+ 3 ) + k 2
2 2
2 AB . BC 2
On squaring both sides, we get
⇒ 36+ 49−A C =6 × 7 ⇒ A C =43 ⇒ AC =√ 43
2 2

( h−3 ) +k 2=16 + ( h+3 ) + k 2+ 8 √( h+ 3 ) + k 2


2 2 2

80 (a)
⇒h 2+ 9−6 h+ k 2=16+ h2 +9+6 h+ k 2 +8 √ ( h+3 ) +k 2
2

Let the point be P ( h , k )


⇒−6 h=16+6 h+ 8 √ ( h+ 3 ) + k
2 2
It is given that difference of the distance from

P a g e | 58
⇒−8 √ ( h+3 ) +k =12 h+16 30
2 2
tan60 ° =
Again , squaring both sides, we get x
2
64 ( h+ 3 ) +k = (12 h+16 )
2 2 ⇒ x=10 √ 3 m
2 2 2
⇒ 64 (h + 9+ 6 h+k )=144 h +256+2.16 .12 h
2 2 2
⇒ 64 (h + 9+ 6 h+k )=16 (9 h +16+24 h)
⇒ 4 ( h2 + 9+ 6 h+k 2) =9 h2 +16+ 24 h
⇒ 4 h2 +36+24 h+4 k 2=9 h 2+16 +24 h
2 2
⇒ 5 h −4 k =20
2 2 85 (a)
h k
⇒ − =1 h
4 5 In ∆ ABD , tan α =
2 2 2h
x y
Hence, the locus of points P is − =1
4 5

81 (b)
2 2 2
a +b −c
∵ cos C=
2 ab

( )
2 2 2 2
2 a +b −c
⇒ cos C= 1
2ab ⇒ tan α = ...(i)
4 4 4 2 2 2 2 2
2
[a +b +c + 2a b −2 c (a +b )]
2 h+ p
⇒ cos C= 2 2 In ∆ ABC , tan 2 α =
4a b 2h
1 2 tan α h+ p
⇒ cos C= [ ∵ a +b +c =2 c ( a + b ) given]
2 4 4 4 2 2 2
2 ⇒ =
1−tan α 2h
2

1
⇒ cos C=±
√2
⇒ ∠ C=45 °∨135 ° ⇒
2
1
2() =
h+ p

()
1
2
2h
1−
82 (d) 2
Locus, of P is 4 h+ p
⇒ =
|√ x 2+ y 2−8 y +16−√ x 2 + y 2 +8 y +16|=6 3 2h
On squaring, we get ⇒ 8 h=3 h+3 p
5h
x + y −2= √ x + y + 8 y+ 16 √ x + y −8 y +16
2 2 2 2 2 2
⇒ 5 h=3 p ⇒ p= m
2 2
3
⇒ ( x + y −2 ) =( x + y +16 ) −( 8 y )
2 2 2 2 2

On simplification, we get 86 (d)


2
y x
2 60
− =1 In ∆ ABD , tan β=
9 7 d
D
83 (a)
A C B
Given, sin sin =sin
2 2 2 E C
60

√ √ √
( s−b )( s−c ) ( s−a )( s−b ) ( s−a )( s−c ) d
⇒ = h
bc ab ac
s−b A d B
⇒ =1
b ⇒ d=60 cot β …(i)
⇒ s=2 b DC
In ∆ DEC , tan α =
EC
84 (c) ⇒ DC =d tan α
In ∆ PR Q

P a g e | 59
⇒ 60−h=d tan α ( ∵ BC =EA=h ) √
Q R= (−√ 3+1 ) + ( √ 3+1 )
2 2

⇒ 60−h=60 cot β tan α [from Eq.(i)] ¿ √ 3+1−2 √ 3+3+1+2 √ 3=2 √ 2


⇒ h=60 1−( cos β sin α

sin β cos α ) √
and RP= (−√ 3−1 ) + ( √ 3−1 )
2 2

60 sin( β−α ) ¿ √ 3+1+2 √ 3+ 3+1−2 √ 3


⇒ h= ¿ 2 √2
cos α sin β
60 sin(β−α ) 60 sin (β−α ) ⇒ P Q=Q R=RP
⇒ = (given) ∴ Triangle is an equilateral triangle
x cos α sin β
⇒ x=cos α sin β 91 (a)
87 (c) Let the third vertex be ( a , b )

| |
We know that, in triangle larger side has an larger 0 0 1
1 1
angle opposite to it. Since, angles ∠ A , ∠ B∧∠ C ∴ Area of ∆= a b 1 = ∨[8 a−6 b]∨¿
2 2
are in AP 6 8 1
⇒ 2 B= A+C As (a ,b) are integers, so we take
∵ A +B +C=π (0, 0), (1, 1), (1, 2)
⇒ B=60° At ( 0 , 0 ) , ∆=0, it is not possible
2 2 2
a +c −b At ( 1 ,1 ) ∆=1
∴ cos B=
2 ac At ( 1 , 2 ) , ∆=2
2
1 100+ a −81 Here, we see that minimum area is 1
⇒ cos 60 °= =
2 20 a
2 92 (d)
⇒ a + 19=10 a
2
⇒ a −10 a+19=0
10 ± √100−76
(cot A2 +cot B2 )(a sin B2 +b sin A2 ) 2 2

=5 ± √ 6

( )
∴ a= A B B A
2 cos +sin +cos + sin
88 (c)
¿
2 2
A
sin sin
B
2 2
( a sin B2 + b sin A2 )
2 2

1 1 1 2 2
, ,
Here, A 2 B C are in AP
(
A +B
)( )
2
sin sin sin2 sin
B
a sin2 + b sin2
A
2 2 2 2 2 2
1 1 1 1 ¿
⇒ − = − , A B
2C 2 B 2B 2 A sin sin
sin sin sin sin 2 2
2 2 2 2

( )
ab ac ac bc B A
⇒ − = − sin sin
( s−a ) (s−b) ( s−a ) (s−c ) ( s−a ) (s−c ) ( s−b ) (s−c)
¿ cos ( C
2
a )
2
A
+b
2
B
( )( )
a b ( s−c ) −c (s−b) sin sin
⇒ 2 2
s−a ( s−b )( s−c )

( √ √
)
( s−a ) (s−c) ( s−b ) ( s−c )
¿ ( )(
s−c
c a ( s−b )−b(s−a)
( s−a ) (s−b) ) ¿

s (s−c )
a
ac
+b
bc

√ √
ab ( s−b ) (s−c) ( s−a )( s−c )
1 1 2
⇒ ab+bc=2 ac ⇒ + = bc ac
c a b

√ ( √( ) √( ) )
Hence, a , b , c are in HP s (s−c ) s−a s−b
¿ ab+ ab
90 (c)
ab s−b s−a
Let the vertices of triangle be P ( 1 ,1 ) , Q(−1 ,−1)
and R(− √ 3 , √ 3)
¿ √ s ( s−c)
s−a+ s−b
√(s−a)(s−b) ( )
∴ P Q=√ (1+1 ) + ( 1+1 ) =2 √ 2
2 2

P a g e | 60
¿ √ s ( s−c)
( √(s−a)(s−b)
2 s−a−b
) ∴ r= =

s
2 √ 14
7
=
8
7√
¿c
√ s (s−c)
( s−a)(s−b)
=c cot
C
2
96 (a)

cos B=
2 2 2
3 + 4 −5 9+16−25
= =0
93 (c) 2 ( 3 )(4) 2 ( 3 ) (4 )
Given lines y=mx , y=2 , y=6 ⇒ ∠ B=90 °
B B
y ∴ sin + cos =sin 45 °+ cos 45 °
2 2
y = mx 1 1
y=6 ¿ + = √2
√2 √2
97 (b)
y=2 CD
A In ∆ CAD , tan30 °=
AC
x' x

y'

Coordinates of points A and B are ( m2 , 2) ,( m6 ,6)


respectively

√(
1 h
∴ AB=
2 6 2

m m )
+ ( 2−6 )2 <5 [given] ⇒ =
√ 3 120+ x
⇒ √ 3 h=120+ x …(i)
( )
2
2 6 2
⇒ − + ( 4 ) <25 CD
m m and in∆ CBD , tan60 ° =
BC
⇒ ( − ) <9 ⇒−3< − <3
2
2 6 2 6 h
m m m m ⇒ √ 3= ⇒ h=√ 3 x …(ii)
x
4 4 From Eqs. (i) and (ii), we get, x=60 m
⇒− >m>
3 3 On putting x=60 in Eq.(i), we get
4 4 h=60 √ 3 m
∴ m∈ ¿−∞ ,− [ ∪ ] , ∞ ¿
3 3
98 (c)
94 (c) 1
By sine rule Area of triangle¿ [ x (1−2 ) +1 (2−0 ) +0 (0−1)]
2
sin A sin B sin C 1
= = (say ) ¿ [ −x+ 2+ 0 ] =4 [given]
a b c 2
∴ (b−c )sin A+ ( c−a ) sin B+ ( a−b ) sin C ⇒ 2−x=8 ⇒ x=−6
¿(b−c)ak + ( c−a ) bk + ( a−b ) kc
¿ k [ab−ac+bc −ab+ ac−bc ] 102 (a)
¿0 The sum of the distance of a point P from two
perpendicular lines in aplane is 1, then the locus
95 (a) of P is a rhombus
Given that, a=3 , b=5 ,c=6
a+ b+c 103 (a)
Now, s= =7 150
2 In ∆ DCE , tan 30 °=
CD
∴ ∆=√ s ( s−a ) ( s−b ) ( s−c )
⇒ CD= √ 3 ×150
¿ √ 7 ( 7−3 )( 7−5 ) ( 7−6 )
¿ √ 7 ∙ 4 ∙ 2∙ 1=2 √ 14

P a g e | 61
−1+7 2+ x
∴ = ⇒ x =4
2 2
−6+2 −5+ y
and = ⇒ y=1
2 2
∴ Fourth vertex of D is (4 , 1)

107 (c)
Now, In ∆ DCF ,
We have, cos A cos B+ sin A sin B sin C=1
DF 200 4 ⇒ 2 cos A cos B+2 sin A sin B sinC=2
tanθ= = =
CD √ 3 .150 3 √ 3
⇒ 2 cos A cos B+2 sin A sin B sinC
2 2 2 2
104 (b) ¿ cos A+ sin A+ cos B+ sin B
2 2

(
2 a sin2
C
2
+c sin2
A
2 ) ⇒ ( cos A−cos B ) + ( sin A−sin B ) + 2sin A sin B ( 1−sin C )
⇒ cos A−cos B=0 , sin A−sin B=0
and 1−sinC=0
( ( s−aab) (s−b) + c ( s−bbc) (s−c) )
¿2 a ⇒ A=B∧C=90 °
⇒ a=b∧C=90 °
¿ 2( (s−a+s−c) )
( s−b)
b 108 (d)
2 We have, A+ B+C=180 °
¿ (s−b)b
b ⇒ A=180 °−( B+C )
¿ 2 ( s−b )=a−b+ c ⇒ tan A=tan ( 180° −B−C )
⇒ tan 90 °=−tan ( B+C )
105 (c)
−tan B+ tan C
Let H be the orthocenter of ∆ OAB ⇒ ∞=
1−tan B tan C
⇒ 1−tan B tan C=0
⇒ tan B tanC=1

109 (b)
Since, the given points lies on a line, then

| |
1 1 1
∴ ( slope of OP ) . ( slope of BA )=−1 −5 5 1 =0
⇒ ( )( )
y−0 4−0
.
3−0 3−4
=−1 13 λ 1
⇒ 1 ( 5− λ )−1 (−5−13 ) +1 (−5 λ−65 )=0
4 ⇒−6 λ=42 ⇒ λ=−7
⇒− y=−1
3
3 110 (a)
⇒ y=
4 The vertices of triangle are (0, 0), (3, 0) and (0, 4).
It is a right angled triangle, therefore
∴ Required orthocentre¿ ( 3 , y )= 3 ,( 34 ) circumcentre is ( 32 , 2)
106 (b)
Let the fourth vertex be D(x , y) 111 (c)
D C BC=5 , BA=10
(x, y) (7, 2)
(-1, -7)
A

A B 10 D
(-1, -6) (2, -5)
We know that two diagonals of a parallelogram B
5
C
(1, 4)
(5, 1)
are bisect each other
Let D divides AC in the ratio 2:1

P a g e | 62
( )
1 1 A B
∴ Coordinate of D is ,
3 3
The bisector is the line joining B and D is

150 m
C D
y−1 1
= ∨x−7 y +2=0
x−5 7 h

E F
112 (b)
AE 150
h In ∆ AEF , tan ϕ = =
In ∆ ECD , tan 3 α = EF EF
CD
5 150
=
2 EF
⇒ EF=60 m
and in ∆ ACD
AC
tanθ=
CD
⇒CD=h cot3 α …(i) 4 150−h
h ⇒ = [ ∵ CD=EF ]
In ∆ EBD , tan 2 α = 3 60
BD ⇒ 80=150−h
⇒ BD=h cot 2 α …(ii) ⇒ h=70 m
h ∴ AC =80 m∧CD=60 m
In ∆ EAD , tan α =
AD
⇒ AD=√ A C2 +C D 2
⇒ AD=h cot α …(iii )
¿ √ 6400+3600
From Eqs. (ii) and (iii),
¿ √ 10000=100 m
AD−BD=h cot α −h cot 2 α
AB=h ¿ 115 (d)
From Eqs. (i) and (ii), CP
In ∆ APC , sin ( ∠ PAC )=
BD−CD=h cot 2 α −h cot 3 α AC
⇒ BC =h ¿
From Eqs. (iv) and (v),
AB
=h ¿ ¿
BC
cos α cos 2 α

sin α sin 2 α
¿
cos 2 α cos 3 α
− r α
sin 2 α sin 3 α ⇒ AC = =r cosec
sin(2 α−α ) α 2 …(i)
sin
2
sin α sin2 α
¿ BC
sin (3 α −2 α ) Again, in ∆ ABC , sin β=
AC
sin 2 α sin 3 α
⇒ BC =AC sin β
sin 3 α 2
¿ =3−4 sin α
sin α
¿ 3−2¿
⇒ H =r cosec
α
2 ()
sin β [from Eq.(i)]

¿ 1+2 cos 2 α 116 (b)


2 2 2
114 (b) b +c −a
Since, cos A=
2 bc
⇒b −2 bc cos A + ( c 2−a2 )=0
2

It is given that b 1 and b 2 are the roots of this


equation

P a g e | 63
2 2 3
Therefore, b 1+b 2=2 c cos A∧b1 b 2=c −a 120 k ∙ 2 16
¿ 3
=
⇒ 3 b 1=2 c cos A∧2 b1=c −a
2 2 2
k ∙7 ∙ 5 ∙3 7
[ ∵ b 2=2 b1 ]
120 (b)

( 23c cos A ) =c −a
2
⇒2
2 2 Given equation of lines are
x=0 , y=0 and 3 x+ 4 y =12
⇒ 8 c2 ( 1−sin 2 A ) =9 c 2−9 a 2 Incentre is on the line y=x (Angled bisector of


2 2 OA and OB )
9 a −c y
⇒ sin A= 2
8c (0, 3)B
3x
117 (a) +
4y
2 =
∵ ( √ a+ √ b+ √c ) ( √ a+ √ b−√ c )=( √ a+ √b ) −c 12
O (4, 0)A
x
¿ a+ b−c+2 √ ab> 0
∴ √ a+ √ b> √ c Angle bisector of y=0 and 3 x+ 4 y =12 is
± 5 y=3 x+ 4 y −12
118 (a) ⇒ 3 x +9 y=12
In ∆ ABC , ∠ A=30 ° , BC =10 cm and 3 x− y=12
O is the centre of circle Here, 3 x+ 9 y=12 internal bisector
∴ ∠ BOC=60° So, intersection point of y=x and 3 x+ 9 y=12 is
and OB and OC are the radius (1 , 1)
∴ ∠ OBC=∠ OCB=60 ° ∴ The required point of the incentre of triangle is
⇒ ∆ OBC is an equilateral triangle
(1 , 1)
∴ radius of circle is
OB=OC =BC =10 cm 121 (a)
2
Now, area of the circumcircle is π r BP− AP=± 6∨BP= AP ±6
⇒ √ x + ( y+ 4 ) =√ x + ( y −4 ) ± 6
2 2 2 2 2
¿ π (10 ) =100 π sq cm
A Squaring and simplification, we get
4 y−9=± 3 √ x 2 + ( y−4 )
2

30 o Again squaring, we get


O 2 2
9 x −7 y +63=0
60o
122 (c)
B C
10 cm Let OP be the tower whose height is h metres
OP
In ∆ OAP , tan α =
119 (b) OA
abc ∆ ⇒ OA =h cot α …(i)
We have, R= and r = OP
4∆ s In ∆ OBP , tan β=
R abc s OB
∴ = ∙ P
r 4∆ ∆
abc h
¿
4 ( s−a ) ( s−b ) (s−c) B
O
Since, a :b : c=4 :5 :6
a b c
⇒ = = =k (say ) d
4 5 6
R ( 4 k )( 5 k ) ( 6 k ) A
=
Thus, r
4(15 k
2
−4 k )(
15 k
2
−5 k )( 15 k
2
−6 k ) ⇒ OB=h cot β ...(ii)
Now, in ∆ OAB , A B2=O A2 +O B 2

P a g e | 64
| |
2 2 2 2
⇒ d =h (cot α + cot β) [from Eq. (i) and (ii)] 0 0 1
d 5
⇒ h= 1 5 1
√cot α +cot 2 β
2 ∴ Area of ∆= 2
2
5
5 1
123 (d) 3
Since, the coordinates of P are (1, 0)
Let any point Q on y 2=8 x is (2 t 2 , 4 t )
Again, let mid point of P Q is (h , k ), so
¿
1
2[(
1
25 25
2

3 )]
1 25 25
2
2 t +1 2
¿ × = sq units
h= ⇒ 2 h=2 t +1 …(i) 2 6 12
2
4 t +0 k 127 (d)
and k = ⇒ t= ...(ii)
2 2 Given that,
on putting the value of t from Eq. (ii) in Eq. (i), we 4 5
tanθ= ∧tan ϕ= ...(i)
get 3 2
2
2k 2 E
2 h= + 1⇒ 4 h=k +2
4
x
Hence, locus of ( h , k ) is y 2−4 x +2=0 h
D C
124 (b) d 150 m
Let P(t ,t ) divides AB in the ratio k :1, then
3 k +k 5 k +2
=t and =t A d B
k +1 k +1
150
k P (t, t) 1 In ∆ ABE , tan ϕ=
A B d
(k, 2) (3, 5) ⇒ d=150 cot ϕ
3 k +k 5 k +2 2
⇒ = ¿ 150 × =60 m…(ii)
k +1 k+1 5
⇒ 4 k −5 k=2 h
In ∆ DCE , tan θ=
⇒ k=−2 d
4 h
⇒ = [from Eq.(i)]
125 (c) 3 d
Since, a , b and c , the sides of a triangle are in AP 4
⇒ h= ( 60 ) [from Eq.(ii)]
∴ 2b=a +c …(i) 3
2 2 2
b +c −a ⇒ h=80 m
We know that, cos A=
2 bc Now, in ∆ DCE , D E2=D C 2+ C E2
2 2 2
b +c −(2 b−c) 2 2 2
⇒ x =60 + 80 =10000
⇒ cos A= [from Eq. (i)]
2 bc ⇒ x=100 m
2 2 2 2
b +c −4 b −c + 4 bc
⇒ cos A= 128 (d)
2 bc
Given, A−B=60 °
4 c−3 b
⇒ cos A= C
2c

126 (b) a 2a
The intersection points of given lines are

( 0 , 0) , ( 52 ,5) ,( 53 , 5) A
By sine rule,
B

2a a
=
sin A sin B
⇒ sin A−2 sin B=0

P a g e | 65
C
⇒ sin ( 60 ° + B )−2 sin B=0
15o
⇒ √ cos B+ sin B−2 sin B=0
3 1
60 m
2 2
√ 3 cos B− 3 sin B=0 15o
⇒ A d B
2 2
60
1
( √3
⇒ √ 3 cos B− sin B =0
2 2 ) In ∆ ABC , tan 15 °=
d

⇒ √ 3 [ cos ( 60 °+ B ) ] =0
⇒ 60 ° + B=90°
⇒ d=60 cot 15 °=60
( √ 3−1)
√ 3+1 m

⇒ B=30 ° 133 (b)


⇒ A=90 ° Given that, cos 2 A+ cos2 C=sin2 B
Hence, it is right angled triangle Obviously it is not an equilateral triangle because
A=B=C=60 ° does not satisfy the given
129 (c)
condition. But B=90 ° , then sin2 B=1 and

| |
3q 0 1
0 3 p 1 =0
1 1 1
cos 2 A+ cos2 C=cos2 A+cos 2 ( π2 − A )
2 2
⇒ 3 q (3 p−1 )+ 1 ( 0−3 p )=0 ¿ cos A+ sin A=1
⇒ 9 pq=3 p+ 3 q Hence, this satisfies the condition, so it is a right
1 1 angled triangle but not necessary isosceles
⇒ + =3 triangle
p q

130 (c) 134 (d)


15+36+ 39 Given, a :b : c=1: √ 3 :2
Here, s= =45 Here, c 2=a2+ b2
2


( s−a ) ( s−b ) ∴ Triangle is right angled at C
C
∴ sin = ∴ ∠ C=90 °
2 ab
a 1


C ( 45−15 ) (45−36) and =
⇒ sin = b √3
2 15 ×36 1

√ 30 × 9 1 ⇒ tan A=
¿ = √3
15 ×36 √ 2 ⇒ ∠ A=30 °∧∠ B=60 ° [as ∠ A+∠ B=90 °]
131 (c)
∴ Ratio of angles ¿ ∠ A :∠ B:∠ C=90°
c ¿ 30 ° :60 ° :90 °=1 :2 :3
Since, =2 R ⇒ c=2 R [ ∵ C=90 ° ] …(i)
sin C 135 (c)
C r 1
And tan = Given, tanθ=
2 s−c 9
π r 3h
⇒ tan = In ∆ ABC , tan ϕ= =h ...(i)
4 s−c 3
∴ r=s−c D
⇒ a+b−c=2 r …(ii)
From Eqs. (i) and (ii), we get h
2 ( r + R )=a+b C

132 (b) 3h
Let BC be the light house
A 3 km B

4h
In Δ ABD , tan ( θ+ϕ )=
3

P a g e | 66
tan θ+ tan ϕ 4 h We have,
⇒ =
1−tan ϕ tan θ 3 ∆= Areaof ∆ ABC
1 1 1
+h ⇒ ∆= × AB ×= ×3 × 4=6 sq .units
9 4h 2 2
⇒ =
h 3 1
1− s=Semi− perimeter= ( 3+ 4+5 ) =6 units
9 2
1+9 h 4 h ∆
⇒ = ∴ r=¿−radius= =1
9−h 3 s
2
⇒ 3+ 27 h=36 h−4 h Hence, the coordinates of the incentre are (1 , 1)
2
⇒ 4 h −9 h+3=0
⇒ h= √
9 ± 81−48 9 ± √33
=
2×4 8

136 (c)
Let a=60 °−d , B=60 ° ,C=60 °+d

∴ =
b
c
sin B
sin C
=

3
2


sin 60°
sin(60 ° +d )
=
3
2 √ 140 (c)
Given, a 4 +b 4 +c 4 =2 c2 (a2 +b2 ) …(i)

√3
2 sin(60 °+d )
=

1
3
2 √ ∵ cos C=
2
a +b −c
2 ab
2 2

⇒ sin ( 60 °+d ) =
[ ]
4 4 4 2 2 2 2 2
a +b +c + 2 a b −2 c (a +b )
√2 ⇒ cos C= 2

⇒ 60 ° +d =45 ° ⇒ d=−15 ° 4 a2 b2
So, ∠ A=75°

137 (b)
¿
[ 2 c 2 ( a 2+ b2 ) +2 a2 b2−2 c 2 (a 2+ b2 )
4a b
2 2 ]
87 [from Eq. (i)]
In ∆ ABC , tan 30 °= 1
x ⇒ cos C=
2
2
⇒ x=87 × √ 3
1
⇒ cos C=±
√2
⇒ ∠ C=45 °∨135 °

141 (c)
Let AB= AC and ∠ A=120°
Diatance
∴ Speed = 2 1
Time ∴ Area of triangle¿ a sin 120 °
2
87 × √ 3 ×60 9 √ 3
⇒ Time= = min
5.8 ×1000 10

138 (c)
It is given that the centroid of the triangle formed
by the points ( a , b ) ,(b , c) and (c , a) is at the
origin Where, a= AD+ BD
¿ √ 3 tan 30 °+ √ 3 cot 15 °
∴ ( a+ b+c a+b+ c
, )
=(0 , 0)
3 3
3
⇒ a+b+ c=0 ⇒ a + b +c =3 abc
3 3
¿ 1+ √ 3 (
1+tan 45° tan30 °
tan 45 °−tan 30°
=1+ √ 3 ) ( √3+ 1
√3−1 )
∴ a=4+ 2 √ 3
139 (d)

P a g e | 67
( )
142 (b)
⇒ Area of triangle¿ ( 4 +2 √ 3 ) √ =12+7 √ 3l
1 2 3
2 2 1
Area= ×base ×altitude
2
1
¿ × ( 2 x cos θ ) ×(x sin θ)
2
1 2
¿ x sin 2 θ
2
(Since, maximum value of sin 2 θ is 1)
1
∴ Maximum area= x 2
2

143 (d)
A C 1
Here, tan tan =
2 2 3



( s−b )( s−c ) ( s−b )( s−a ) 1

s−b 1
s ( s−a )
.
s ( s−c )
=
3
⇒ = ⇒ 2 s=3 b
s 3
⇒ 2 b=a+ c
⇒ a ,b , c are∈ AP

144 (d)
Let the vertices of a triangle are P ( 2 ,1 ) , Q ( 5 , 2 )
and R ( 3 , 4 ) and A ( x , y ) be the circumcentre of
∆ PQ R
∴ A P 2=A Q2
2 2 2 2
⇒ ( 2−x ) + ( 1− y ) =( 5−x ) + ( 2− y )
2 2 2 2
⇒ 4+ x −4 x +1+ y −2 y=25+ x −10 x +4 + y −4 y
⇒ 6 x +2 y=24
⇒ 3 x + y=12 …(i)
and A P2= A R2
2 2 2 2
⇒ ( 2−x ) + ( 1− y ) =( 3−x ) + ( 4− y )
2 2 2 2
⇒ 4+ x −4 x +1+ y −2 y=9+ x −6 x +16+ y −8 y
⇒ 2 x +6 y=20
⇒ x +3 y=10 …(ii)
On solving Eqs. (i) and (ii), we get
13 9
x= and y=
4 4

∴ Circumcentre is ( 134 , 94 )
145 (c)

P a g e | 68
( a+ b+c ) ( b+ c−a )=kbc 148 (c)
⇒ 2 s ( 2 s−2 a )=kbc a cot A+ b cot B+ c cot C
s ( s−a) k a b c
⇒ = ¿ cos A+ cos B+ cos C
bc 4 sin A sin B sin C
¿ 2 R ( cos A +cos B+cos C )
⇒ cos 2 ( )A
2
=
k
4
( )r
¿ 2 R 1+ =2 (r + R)
R
∵ 0<cos 2 ( ) A
2
<1
150 (a)
k Given pair of lines are rotated about the origin by
∴ 0< <1
4 π /6 in the anti-clockwise sense.
⇒ 0< k < 4 π √3 x − y π
' '
∴ x=x ' cos −¿ y ' sin =
¿
146 (d) 6 2 6

‖ ‖
π x +√ 3 y
' '
α β 1 ' π '
1 1 and y=x sin + y cos =
∵ Area of ∆ PBC= −3 5 1 = ∨7 α +7 β−14∨¿ 6 6 2
2 2 on putting the values of x and y in given pair of
4 −2 1

‖ ‖
lines, we get
6 −3 1
( ) ( )( ) ( )
1 2 2

√3 √ √3 x ' − y ' x ' + √ 3 y ' + √3 x ' + √3 y '


' '
Also, Area of ∆ ABC= −3 5 1 3x −y
2 −4
4 −2 1 2 2 2 2
1 7 ⇒ √ 3 ( 3 x '2 + y ' 2−2 √ 3 x ' y' ) −4 ( √ 3 x ' 2 3 x ' y ' −x ' y ' −√ 3 y ' 2) +
¿ |42−21−14|=
2 2 ⇒ 3 √ 3 x + √ 3 y −6 x y −4 √ 3 x −8 x y + 4 √ 3 y + √ 3 x
'2 '2 ' ' '2 ' ' '2

Area of ∆ PBC 7 ⇒ 8 √ 3 y −8 x y =0
= ∨α + β−2∨ ¿ ¿
'2 ' '

Areaof ∆ ABC 2 7 ⇒ √ 3 y −x y =0
'2 ' '

2
∴ Required equation is √ 3 y 2−xy=0
¿∨α + β−2∨¿
151 (c)
147 (b)
Using sine rule,
Let DP is clock tower standing at the middle
sin A sin B
point D of BC =
a b
Let ∠ PAD=α =cot−1 3.2 ⇒ cot α =3.2
2
C P 3 sin B
⇒ =
2 3
h
D ⇒ sin B=1
90o 90o ⇒ B=90 °
90o
152 (a)
A 100 m B
since, b , c and a are in AP
and ∠ PBD=β=cose c−1 2.6 a b c
By sine rule, = =
⇒ cosec β=2.6 sin A sin B sin C
∴ cot β=√(cose c 2 β −1) ⇒ a=
b
=
c
[ ∵ ∠ A=90 ° ]
¿ √ 5.76=2.4 sin B sin C
In ∆ PAD∧PBD , b c
⇒ sin B= , sin C=
AD=h cot α =3.2 h a a
and BD=h cot β=2.4 h 153 (d)
In ∆ ABD , A B2= A D2 + B D 2 2 2 2
2 a + 4 b +c =4 ab+ 2 ac
⇒ 1002 =[ ( 3.2 )2+ ( 2.4 )2 ] h2=16 h2 2 2 2 2
⇒ a + ( 2b ) −4 ab+ a +c −2 ac=0
100 2
⇒ ( a−2 b ) + ( a−c ) =0
2
⇒ h= ⇒ h=25 m
4

P a g e | 69
⇒ a=2 c=c 40 ×2 √ 2
¿ =40 ( √ 6−√ 2 ) ft
cos B=
2 2
a + c −b
2
√ 3+1
2 ac A
Tower
()
2
2 2 c c2 75o
c +c − 2
2c −
2 4 30o C
¿ = t
0f
o
2×c×c 2c
2 75
o 8
15
7 B
⇒ cos B=
8
157 (c)
154 (b) Let the points be B(x 1 , y 1 ) and D( x 2 , y2 ) and
Given, M divides AB in the ratio b : a¿ externally¿ coordinates of mid point of BD are
ba cos β−ba cos α
∴ x=
b−a
ab sin β−ab sin α
( x +2 x , y +2 y )
1 2 1 2

and y=
b−a
x cos β−cos α
⇒ =
y sin β−sin α

x
⇒ =
2sin ( ) ( )
α+β
2
sin
α −β
2
And coordinates of mid point of AC are (0, 1)
y
2cos ( ) ( )
α+ β
2
sin
β−α
2 We know that mid point of both the diagonals lie
on the same point E .
⇒ x cos ( ) + y sin (
2 )
α+β α+ β
=0 x1 + x2 y 1+ y 2
2 ∴ =0 and =1
2 2
155 (a) ⇒ x 1 + x 2=0 …(i)
64 and y 1 + y 2=2 (ii)
In ∆ DAB , tan θ=
d also, slope of BD × slope of AC=−1
( y 1− y 2) (3+1)
× =−1
(x 1−x 2 ) (0−0)
⇒ y 1− y 2=0 …(iii)
On solving Eqs. (ii) and (iii), we get
y 1=1 , y 2=1
Now, slope of AB× slope of BC=−1
( y 1 +1) ( y 1−3)
⇒ d=64 cot θ … (i) ⇒ × =−1
( x 1−0) (x 1−0)
(100−64) 2
In ∆ CDE , tan ( 90 °−θ )= ⇒ ( y1 +1 ) ( y 1−3 )=−x 1
d
2
⇒ d=36 tan θ …(ii) ⇒ 2 (−2 )=− x1 [ ∵ y 1=1]
On multiplying Eqs. (i) and (ii), we get ⇒ x 1=± 2
2
d =36 × 64 ⇒ d=48 ∴ The required points are (2, 1) and (−2 , 1 )
156 (b) 158 (a)
Let BC be the declivity and BA be the tower The diagonals meet at the mid point of AC , ie at
∴ In ∆ ABC , on applying sine rule (3, 2) which lies on y=2 x +c
BC AB C(5, 1)
= D
sin 75° sin 30 °
80 sin 30 °
⇒ AB= A
B
sin 75 ° (1, 3)

P a g e | 70
√( )
∴ c=−4 5 2 5
Let B=(α , 2 α −4 ) ¿ −2. .1
2 2
∵ AB ⊥ BC

2α −7
α −1 ( )( )
2 α −5
α −5
=−1
¿
√√
5 5
2 2
−2=
√5
2

∴ α 2 −6 α + 8=0 161 (d)


⇒ α=2 , 4 Area of ∆ ABC=a2
The other two vertices are (2, 0) and (4, 4) C

159 (c) 3a

Given, r 3−r=r 1 +r 2 O A B
x a-x
⇒ 4 R sin
C
2
A B B
(
cos cos −sin sin
2 2 2
A
2 ) x=a

¿ 4 R cos
C
2 [A B A
sin cos +cos sin
2 2 2
B
2 ] 1
⇒ ( a−x ) 3 a=a
2
2

⇒ sin
C
2 [ ( )]
cos
A +B
2
C
=cos sin
2
A +B
2 [ ( )] 2
⇒ a−x= a
3

[ ( )] [ ( )]
C π C C π C a
⇒sin cos − =cos sin − ⇒ x=
2 2 2 2 2 2 3
Hence, one of the line on which third vertex lies is

[ A B π C
∵ A+ B+C=π ⇒ + = −
2 2 2 2 ] x=
a
3
2C 2C C
⇒ sin =cos ⇒ tan =1
2 2 2 162 (c)
π Draw BE perpendicular to CA produced, then
⇒ ∠ C=
2 a
BD=DC= and EA=AC =b
π 2
We know, A+ B+C=π ⇒ A+ B=
2 In ∆ AEB ,
b
160 (d) cos (π− A)=
c
Given a=3 , b=4 , c=5 −b
2 2 2 ⇒ cos A=
⇒ c =a +b c
B 2 2 2
b +c −a −b
⇒ =
2 bc c
2 2 2
5 ⇒ a =3 b +c
3 2 2 2
c + a −b
∴ cos B=
2 ac
2 2 2 2 2 2
C 4 A c +3 b + c −b b +c
¿ =
Therefore, it is a right angled triangle at C 2 ca ca
5 1
∴ R= c=
2 2
1
× 3× 4
∆ 2
and r = = =1
s 12
2 164 (d)
∴ Distance between incentre and circumcentre Let the sides of ∆ ABC be a=n , b=n+1 , c=n+2 ,
¿ √ R2−2 Rr where n is a natural number. Then, C is the

P a g e | 71
greatest and A is the least angle 2
k + 4−2 k−4=0 ⇒ k =0 , 2
As given C=2 A We have already take a point (0 ,−2) , so we take
∴ sin C=sin 2 A=2 sin A cos A only k =2
2 2 2
b +c −a
∴ kc=2 ka 167 (a)
2 bc
2 2 2 2 Let X =x−h , Y = y −k
⇒ b c =a( b + c −a )
⇒ 0=7−h , 0=−4−k
On substituting the values of a , b , c , we get
⇒ h=7 , k =−4
( n+1 )( n+2 )2 =n[ ( n+1 )2 + ( n+2 )2 −n2 ]
Hence, X =x−7 and Y = y+ 4 , then the point (4,
¿ n ( n 2+6 n+5 ) 5) shifted to (−3 , 9)
¿ n ( n+1 ) ( n+5 )
2 168 (a)
⇒ ( n+ 1 ) [ ( n+2 ) −n(n+5)]=0
Since, n ≠ 1
Thus, ( n+2 )2 =n ( n+5 )
( a+ b+c ) tan ( A
2
B
) ( A
+ tan =2 s tan + s tan
2 2
B
2 )
2
⇒ n + 4 n+ 4=n +5 n
⇒ n=4
2
¿2 ( s−a

+
s−b )

Hence, the sides of the triangle are 4, 5 and 6 2 s−(a+ b)


¿2∆
( s−a ) (s−b)
165 (c)

3.
b2 −c 2
=
2
2 R (sin B−sin C)
2 2 ¿2∆
( c
( s−a ) ( s−b ) )
a sin(B−C) 2 R sin A sin(B−C) C
¿ 2 c cot
2
2 R sin( B+C )sin (B−C)
¿ =2 R
sin(B+C)sin(B−C) 169 (b)
4. 2 2 1 8
a
∵ + + =
a sin ( B−C ) +b sin ( C− A )+ c sin ( A−B )=0 1 ! 9 ! 3 ! 7 ! 5 ! 5! (2 b)!
a
1 1 1 1 1 8
¿ 2 R[sin A sin ( B−C ) +sin B sin ( C−A ) +sin C sin ( A−B)]
⇒ + + + + =
1! 9! 3 ! 7 ! 5 ! 5 ! 3! 7 ! 9 ! 1 ! (2 b)!

( )
a
¿ 2 R[sin ( B+C ) sin ( B−C )+ sin (C + A ) sin ( C− A ) +sin ( A+ B1) sin (10
A−B! ) ]10 ! 10 ! 10 ! 10 ! 8
⇒ + + + + =
10 ! 1 ! 9 ! 3 ! 7 ! 5 ! 5 ! 7 ! 3 ! 9 ! 1! ( 2b)!
2 2 2 2 2 2
¿ 2 R[sin B−sin C +sin C−sin A +sin A−sin B] 1 10 10 10 10 10 8
a
⇒ ( C 1+ C 3+ C5 + C7 + C9 )=
10 ! (2 b)!
¿ 2 R ( 0 )=¿ 0 9 a 3a
2 8 2
⇒ = =
Hence, both of statements are correct 10 ! (2 b)! (2 b)!
⇒ a=3 , b=5
166 (b)
Also, 2 b=a+c ⇒ 10=3+c ⇒ c=7
Let the general equation of the circle be
2 2 ∴ a=3 ,b=5 , c=7
x + y + 2 g x +2 fy+ c=0
tan A +tan B
The equation of circle passing through (0, 0), (2, ∵ ≥ √ tan A tan B …(i)
2
0) and (0 ,−2) 2 2 2
a +b −c
c=0 …(i) Also, cos C=
2 ab
4 + 4 g +c=0 …(ii)
9+25−49 −1
and4−4 f +c=0 …(iii) ¿ =
30 2
On solving Eqs. (i), (ii) and (iii), we get
⇒ ∠ C=120 °∧ A , B<60 °
c=0 , g=−1 , f =1
tan A+ tan B+ tanC=tan A tan B tan C
∴The equation of circle becomes
2 2 ⇒ tan A + tan B− √ 3=−√ 3 tan A tan B
x + y −2 x+2 y=0
∴ tan A+ tan B=√ 3(1−tan A tan B) …(ii)
Since, it is passes through ( k ,−2 ) , we get

P a g e | 72
Also, tan A+ tan B>0 172 (a)
⇒ √ 3(1−tan A tan B)>0 ( a+ c )2−b2=3 ac ⇒ a2+ c 2−b2 =ac
⇒ tan A tan B<1 …(iii) 2 2 2
a + c −b 1 π
But cos B= = ⇒ ∠ B= =60 °
From Eq. (i) and (ii), 2 ac 2 3
√3 (1−tan A tan B) ≥ (tan A tan B)
2
√ 173 (c)
Let tan A tan B=λ Given, a 2 , b2 , c 2 are in AP
2 2 2 2
∴ √3 (1−λ) ≥ 2 √ λ ⇒ sin B−sin A=sin C−sin B
2 ⇒ sin ( B+ A ) sin ( B− A )=sin ( C+ B ) sin ( C−B )
⇒ 3 λ −10 λ+3 ≥ 0
⇒ (3 λ – 1)(λ – 3)≥ 0 ⇒ sin C ( sin B cos A−cos B sin A )
∵ λ−3<0 [from Eq. (iii)] ¿ sin A(sinC cos B−cos C sin B)
∴ 3 λ−1 ≤ 0 On dividing by sin A sin Bsin C , we get
1 2 cot B=cot A+ cotC
⇒λ≤ ⇒ cot A , cot B , cot C are in AP
3
1
⇒ tan A tan B≤ 174 (c)
3
ab
Given, sin A sin B= 2
170 (c) c

In ∆ BCD , tan60 °=
H1
d
⇒ c2=
ab
=
a
sin A sin B sin A ( )( sinb B )
( )
D 2
2 c
⇒c =
sin C

E H1
∵ ( sina A = sinb B = sincC )
2
⇒ sin C=1
H2
30o 60o ⇒ c=90 °
A d B d C Hence, ∆ ABC is a right angled triangle
⇒ H 1=d tan 60 ° 175 (a)
H2 Let a , b , c be the sides of triangle, then
and in ∆ ABE , tan 30 °=
d 6
a+ b+c= (sin A+ sin B+sin C )
⇒ H 2=d tan 30° 3
H tan 60 ° √3 = 3 ⇒ a+b+ c=2(sin A+sin B+sin C)
∴ 1= =
H 2 tan 30 ° 1 / √ 3 1 a
⇒ =sin A
2
171 (c) But a=1
2 2 2
a +b −c 1 π
We have, cos C= ∴ sin A= ⇒ ∠ A=
2 ab 2 6
2 2 2
a +b −c
⇒ cos 60 °= 176 (a)
2 ab
2 2 2 The centroid of
⇒ a + b −c =ab
2 2
⇒ b + bc+ a +ac=ab+ac +bc +c
2

On dividing by ( a+ c ) (b+ c) and add 2 on both


∆ ABC= ( 2+8+5
3
,
3+10+5
3 )=(5 ,6)
sides, we get 177 (d)
b a Since, the tower OP makes equal angle at the
1+ +1+ =3
a+c b +c vertices of the triangle, therefore foot of the tower
1 1 3 is the circumcentre
⇒ + =
a+ c b+c a+b+ c

P a g e | 73
P 1
A ∴ Area of triangle= ab sin C
2
R 1
¿ ×1 ×2 ×sin 60 °
O 2

C ¿
√3 sq unit
B D 2
OP
In ∆ OAP , tan α = 180 (c)
OA
The given points are collinear
⇒ OP=OA tan α

| |
⇒ OP=R tan α t 1 2 a t 1+ a t 31 1
If t 2 2 a t 2+ a t 2
3
1 =0
178 (a) 3
t 3 2 a t 3+ a t 3 1
In ∆ ABE ,

| |
sin β=
BE t 1 2t 1 +t 31 1
b ⇒ a t 2 2t 2 +t 32 1 =0
⇒ BE=h1=b sin β
t 3 2t 3 +t 33 1
Using sine rule in ∆ AED ,
Applying R2 → R2 −R 1 , R3 → R 3−R1, we get
sin(α −β ) sin (γ −α )

| |
=
ED b t1 2t 1+ t 31 1
b sin ( α−β ) t 2−t 1 2 ( t 2−t 1 ) + ( t −t )
3 3
0 =0
⇒ ED= 2 1
sin(γ−α) t 3−t 1 2 ( t 3 −t 1 ) + ( t −t
3 3
) 0
3 1
D

| |
t1 2 t 1+ t 31 1
⇒ ( t 2−t 1 ) ( t 3 −t 1 ) 1 2+t + t +t 1 t 2 0 =0
2 2
-

2 1
h2 2 2
1 2+t + t +t 3 t 1 0
3 1
b
-
E
F ⇒ ( t 2−t 1 ) ( t 3 −t 1 ) ( t 3−t 2) ( t 3 +t 2+t 1 )=0
h1
A C ⇒ t 1+ t 2 +t 3=0
B
Now, in ∆ FED , [ ∵ t 1 ≠ t 2 ≠ t3 ]
h2
sin γ = 181 (b)
ED Let P is a point on the perpendicular bisector of
b sin ( α −β ) sin γ AB, its equation is
⇒ h 2=
sin (γ −α ) 1
∴Total height,CD ( y−1 )= ( x−4 ) ⇒ x−3 y−1=0
3
b sin ( α−β ) sin γ So, general point is P ( 3 h+1 , h )
¿ h1 +h 2=b sin β+

[ ]
sin ( γ −α ) 3 h+1 h 1
1
b [ sin β sin ( γ−α ) +sin ( α −β ) sin γ ] Area¿ 3 4 1 =±10
¿ 2
sin(γ −α ) 5 −2 1
¿b¿¿ ⇒ h=2 , 0
¿ Or position of points is (7, 2) and (1, 0)
¿b¿¿
¿ 182 (a)
b sin α sin ( γ−β ) π
¿ Given, ∠ A= , then a 2=b2 +c 2=42 +32 =25
sin ( γ −α ) 2
a
179 (c) and =2 R ⇒ a=2 R and a=5
sin A
Given, a=1 , b=2 , ∠ C=60 °

also, r = =
bc
s a+b+ c
∵ ∆=
bc
2 ( )
P a g e | 74
R a (a+b +c) 5 ×12 5 183 (d)
∴ = = =
r 2 bc 2× 4 × 3 2 Since A , B ,C are in AP
⇒ 2 B= A+C ⇒ ∠ B=60 °
a c
∴ ( 2 sin C cos C ) + (2sin A cos A )
2 a
¿ 2 k (a cos C+ c cos A)
¿ 2 k (b)
¿ 2 sin B [using, b=a cos C +c cos A ]
¿√3

184 (d)
Given equation is
2
x −5 x+ 6=0
⇒ ( x−3 )( x−2 )=0
⇒ x=3 , 2
These are the sides of a triangle
π
Let a=3 , b=2, ∠ C=
3

() [ ]
2 2 2 2 2 2
π 3 +2 −c a + b −c
⇒ cos = ∵ cos C=
3 2.3.2 2 ab
2
1 13−c 2
⇒ = ⇒ c =7
2 12
⇒ c=√ 7 [sides cannot be negative]
∴ Perimeter of a triangle ¿ a+ b+c
¿ 3+2+ √ 7=5+ √ 7

185 (b)
Given, ∠ A=60 ° , a=5 , b=4
2 2 2
b + c −a
∴ cos A=
2 bc
2
1 16+ c −25
⇒ cos 60 °= =
2 8c
2
⇒ 4 c=c −9
2
⇒ c −4 c−9=0

186 (a)
Let PQ be the height h of the tower and A , B are
the points of observations
Q

B2
B
/6
A B1 P

π π
We have, ∠ QAP= ,∠ BAP= ,
4 6
AB=10 m , BQ=10 m

P a g e | 75
π 1
∴ ∠ QAB=
12
=∠ AQB ¿
2
[ 0 ( 0 ) +12 ( 2 ) +12 ( 7 ) +6 ( 5 ) +0 ( 0 ) −{0+ 0+2 ( 6 ) +7 ( 0 ) +5(
π 5π 1
⇒ ∠ ABQ=π− = ¿ [(24+84 +30−12)]
6 6 2
On applying cosine rule in ∆ ABQ , we get ¿ 63 sq unit
2 2 2 5π
A Q = A B +B Q −2 AB∙ BQ cos 189 (a)
6
Let the height of the flag be h
¿ 100+100+200 ∙ √
3
2
¿ 100(2+ √ 2)
⇒ AQ=10 √ 2+ √ 3
π 10 √ 2+ √ 3
In ∆ APQ , AP= AQ cos =
4 √2

¿ 5 √ 4+ 2 √ 3=5 ( √ 3+1 )
2

b
⇒ AP=5 ( 1+ √ 3 ) m In ∆ AR Q , tan α = … .(i)
a
h+ b
187 (a) and in ∆ PR Q , tan 2 α = …(ii)
a
Let the points be A=( a cos θ , a sin θ) and
2 tan α h+b
B=(a cos ϕ , a sin ϕ ) ⇒ =
1−tan α
2
a
∴ AB=√ ( a cos θ−a cos ϕ ) + ( a sin θ−a sin ϕ )
2 2
b

¿ √ a2 cos 2 θ+ a2 cos 2 ϕ−2 a2 cos θ cos ϕ+ a2 sin 2 θ a h+ b
2 2 2 ⇒ = [¿ Eq . ( i ) ]
+ a sin ϕ−2a sin θ sin ϕ b
2
2
1− 2
¿ √ 2 a2−2 a2 (cos θ cos ϕ +sin θ sin ϕ ) a
2 2
¿ √ 2 a( √ 1−cos (θ−ϕ )) 2 ab h+b b (a +b )
⇒ 2 2= ⇒ h= 2 2
( )
θ−ϕ a −b a (a −b )
⇒ 2 a=√ 2 a √2 sin
2
190 (a)
⇒ sin
θ−ϕ
2
=1( ) Let BC be the height of kite
C
θ−ϕ π
⇒ =nπ ±
2 2 0
m
⇒ θ−ϕ=2 nπ ± π 12 h
⇒ θ=2 nπ ± π −ϕ 60o
A
Where n ∈ Z B
h
188 (d) In ∆ ABC , sin 60 °=
120
Area of pentagon
⇒√ =
3 h
¿
1
[ x1 y2 + x2 y3 + x3 y4 + x4 y5
2 + x 5 y 1−( y 1 x 2+ y 2 x 3+ y 3 x 4 ¿ + y 4 x 5 + y 5 x1 ) ] 2 120
⇒ h=60 √ 3 m
The height of the kite is 60 √ 3 m

191 (b)
a cos A+ b cos B+c cosC
a+b+ c
( 2 R sin A ) cos A+ ( 2 R sin B ) cos B+2 R sin C (cos C )
¿
2 R sin A +2 R sin B+2 R sinC

P a g e | 76
(∵ R= 2 sina A = 2 sinb B = 2sinc C = abc
4∆ )
¿ R¿¿
1 (sin 2 A+ sin 2 B+sin 2 C)
¿ .
2 (sin A+ sin B+sin C )
4 sin A sin B sin C
¿
2[4 cos ( A /2)cos(B/2)cos(C /2)]

( )
∵ sin 2 α +sin 2 β+ sin 2 γ =4 sin α sin β sin γ
α β γ
¿ sin α + sin β +sin γ=4 cos cos cos
2 2 2

¿
[
4 2sin
A
2
A B B C
cos ×2 sin cos × 2sin ×cos
2 2 2 2
C
2 ]
[
2× 4 cos
A
2
B
cos cos
2
C
2 ]
A B C
¿ 4 sin
sin sin
2 2 2
r
[ A B
¿ ∵ r=4 R sin sin sin
R 2 2
C
2 ]
192 (a) Thus, Q divides AB externally in the ratio 2 :3
Let ∠ APC=α . Then, and hence its coordinates are
tan α =
AC AC
= =
AB
=
1
AP n AB 2 n AB 2 n ( 2× 6−3×
2−3
1 2×16−3 ×1
,
2−3 )≡(−9 ,−29)
194 (d)
Let the angles of a triangle are 3 x , 5 x and 10 x
∴ 3 x +5 x +10 x=180° ⇒ x=10 °
∴ Smallest angle of a triangle ¿ 30 °
And the greatest angle¿ 100 °
Required ratio¿ sin 30 ° : sin100 °
1
¿ :cos10 °=1 :2 cos10 °
In ∆ APB , we have 2
AB AB 1
tan(α + β)= = = 195 (b)
AP n AB n

| |
Now, β=α + β−α 1 a b
tan ( α + β )−tan α Given, 1 c a =0
⇒ tan β= 1 b c
1+ tan(α+ β )tan α
1/n−1 /2 n n ⇒ c2−ab−a ( c−a )+ b ( b−c )=0
⇒ tan β= = 2 2 2
1+1 /n ∙ 1/2 n 2 n2 +1 ⇒ a + b +c −ab−bc−ca=0
193 (d) 1
⇒ [ 2 a +2 b +2 c −2 ab−2 bc−2 ca ] =0
2 2 2

Suppose P(3 , 7) divides the segment joining 2


A(1 , 1) and B(6 , 16) in the ratio λ :1. Then, 1
⇒ ¿
6 λ+1 3∧16 λ +1 2
= =7 ⇒ a=b=c
λ+1 λ+1
2 ⇒ ∠ A=60 ° ,∠ B=60° , ∠ C=6 0 °
⇒ λ=
3 ∴ sin2 A +sin2 B+sin 2 C
2 2 2
⇒ P divides AB internally in the ratio 2 :3 ¿ sin 60 °+ sin 60° +sin 60 °

P a g e | 77
(2) (2) (2)
2 2
√ 3 + √3 + √3 = 9
2
3 ×6−1 ×1 17
¿ ∴ x1= =
4 4 4
−2 ×3+ 4 −2 −1
and y 1= = =
197 (c) 4 4 2
Let ∠ OAB=θ 5
y
similarly, x 2=0 , y 2=
2
−5
B and x 3= , y 3=4
4
(1, 4)
let ( x , y ) be the coordinates of centroid of ∆≝¿

O A
x

Then, OA + AB=1+ 4 cot θ+ 4+ tan θ


∴ x=
1 17
3 4 ( +0− =1)
5
4

y=(
2 2 )3
−1 5 1
¿ 5+ 4 cot θ+ tan θ≥ 5+ 4=9 and + + 4 =2
(using AM ≥ GM )
∴ Coordinates of centroid are (1, 2)
199 (c)
202 (a)
Given ∠ A=20 °
1 1 3
∴ ∠ B=∠ C=80 ° We have, = =
a+c b+c a+b +c
Then, b=c
a+b+2 c 3
a b c ⇒ =
∴ = = ( a+ c )(b+c ) a+b+ c
sin 20 ° sin 80° sin 80 °
2 2 2
a b ⇒ a + ab+ ac+ ab+b +bc +2 ca+ 2bc +2 c
⇒ = 2
sin 20 ° cos 10° ¿ 3(ab+ac +bc +c )
⇒ a=2 b sin 10 ° …(i) 2 2 2
⇒ a + b −c =ab
∴ a3 +b 3=8 b3 sin3 10 °+ b3 2 2 2
a +b −c 1
⇒ = =cos C
¿ b3 {2 ( 4 sin3 10 ° ) +1} 2 ab 2
3
¿ b {2 ( 3 sin 10 °−sin 30 ° ) +1} ⇒ ∠ C=60 °
3
¿ b {6 sin10 ° } 203 (a)
2
¿ 3 b {2 b sin 10 ° } Let CD be the tower of height H metre
2
¿ 3 b a [from Eq. (i)] A
60o
2
¿ 3 a c ( ∵ b=c)
C
h
200 (a)
H
a ( cos2 B+ cos2 C ) + cos A (c cos C +b cos B) 30o 60o D
B d
¿ cos B ( a cos B+b cos A ) +cos C ( a cos C+ c cos A )
¿ ( cos B ) c + ( cos C ) b H
From ∆ BCD , =tan 30 °
¿a d
and from ∆ ABD ,
201 (b) h
=tan 60 °
Let ( x 1 , y 1 ) ,( x 2 , y 2) and (x 3 , y 3 ) are the d
coordinates of the points D , E and F H /d tan30 ° h
∴ = ⇒ H=
h /d tan60 ° 3

204 (c)
Using sine rule in ∆ ADC ,
sin( y + z ) sin C
=
DC AD

P a g e | 78
sin x sin B Hence, locus of (h , k ) is
In ∆ ABD , =
BD AD ( x−x 1 ) ( x−x 2 ) + ( y− y1 ) ( y − y 2 )=0
sin z sinC Which is a circle having AB as diameter
In ∆ AEC , =
EC AE
sin( x + y ) sin B 208 (a)
In ∆ ABE , =
BE AE C
2 2 A 3b
Given, a cos +c cos =
2 2 c

⇒a [s( s−c )
ab
+c ] [
s( s−a) 3 b
bc
=
2 ]
s ( s−c +s−a) 3 b
⇒ =
b 2
2
sin ( x + y ) sin ( y+ z ) BE DC AD AE ⇒ 2 s ( 2 s−c−a )=3 b
∴ = × × ×
sin x sin z AE AD BD EC ⇒ ( a+b+ c ) b=3 b2
2 BD × 2 EC ⇒ 2 b=a+ c
¿ =4
BD × EC
209 (c)
205 (a)
sin A sin( A−B)
∵ =
sinC sin (B−C) R 2
sin A sin A cos B−cos A sin B r
⇒ = n n
sin C sin B cos C−cos B sin C
a a cos B−b cos A a π
⇒ = ∵ =sin
c b cos C−c cos B 2R n
⇒ ab cos C+ bc cos A=2 ac cos B a π r π
and =tan ∴ =cos
2 2 2
a 2+b −c b +c −a
2 2 2 2 2
c + a −b
2 2r n R n
⇒ + =2 r 1
2 2 2 For n=3 gives =
2 2 2 R 2
⇒ a + c =2 b
r 1
For n=4 gives =
2 2 2
⇒ a ,b , c are in AP
R √2
206 (b) r √3
For n=6 gives =
Let point ( x 1 , y 1 ) be on the line 3 x+ 4 y =5 R 2
∴ 3 x 1 +4 y 1=5 …(I) 210 (d)
2 2 2 2
Also, ( x 1−1 ) + ( y 1−2 ) =( x 1−3 ) + ( y 1−4 ) Given, the vertex angles are A , B ,C are side BC
2 2 2 2
⇒ x 1 + y 1−2 x 1−4 y1 +5=x 1+ y1 −6 x 1−8 y 1+ 25 1
∴ Area= a ( k sinC ) sin B
⇒ 4 x 1+ 4 y 1=20 …(ii) 2
2
On solving Eqs. (i) and (ii), we get 1 a k sin C sin B
¿ .
x 1=15 , y 1=−10 2 a
2
1 a k sin C sin B
¿ .
207 (a) 2 k sin A
Let A ( x 1 , y 1 ) , B (x 2 , y 2) be two fixed points and 1 2 sin B sin C
¿ a .
let P(h , k ) be a variable point such that 2 sin A
π
∠ APB= 211 (c)
2
Let OP be the flag staff of height h standing at the
Then, slope of AP× slope of BP=−1
centre O of the field
k − y1 k − y1
⇒ . =−1
h−x 1 h−x 2
⇒ ( h−x 1 ) ( h−x 2 ) + ( k− y 1 ) ( k − y 2 )=0

P a g e | 79
P =5 unit
45o 45o
C A
D F h
o
90o 15 E
90o O O
A B
B C
In ∆ OEP ,OE=h cot 15° =h(2+ √ 3) D
and in ∆ OFP ,OF =h cot 45 °=h In ∆ BOD ,
∴ EF=h √1+ ( 2+ √3 )
2
BD
cos 30 °=
¿ 2 h √ 2+ √ 3 OB
Since, AC=1200 √3
⇒ BD= ×5=
5 √3
unit
⇒ 2 EF=4 h √ (2+ √ 3) 2 2
300 ∴ BC =2 BD =5 √3
⇒h= √ 3 ( 5 √ 3 )2= 75 √3 sq
√2+ √3 Hence, area of ∆ ABC=
4 4
¿ 300 √ 2−√ 3 m
units
214 (b)
218 (d)
A+ B+C=π
Let A ( 1 , √ 3 ) , B ( 0 , 0 ) ,C (2 , 0) be the given points
2 B= A+C
∴ BC=√ ( 2−0 ) + ( 0−0 ) =2
2 2
⇒ 3 B=π
⇒ B=
π √
CA = (2−1 )2+ ( 0− √ 3 ) =2
2

3 and AB=√ 1+3=2


a+c sin A+ sin C ∴ Triangle is equilateral
∴ =
b sin B We know that in equilateral triangle incentre is
A +C A−C the same as Centroid of the triangle
2sin cos
2 2
¿
sin
π
3
∴ Incentre is ( 1+0+2
3
,√
3+0+0
3 )=( 1 , )
1
√3
π A−C 219 (b)
2sin cos
3 2 We know that area of the triangle with polar
¿
π coordinates
sin
3 1
¿ ∨∑ r 1 r 2 sin(θ1−θ2 )∨¿
A−C 2
¿ 2 cos
2

215 (c)
1 π
( )
¿ ∨2.1sin 0− +2.3 sin −
2 3
π 2π
3 3 (
+ 3.1sin

3 )
−0 ∨¿ ( )
The altitudes from the vertices A , B and C are
2∆ 2∆
, and
2∆
respectively
¿
1
2|−2. −
2 2 2 4 |
√3 6 √ 3 + 3 √ 3 = 5 √3 sq units

a b c
220 (d)
Also, these are in HP
For finding the distance AP, first we find out the
1 1 1
⇒ , , are in HP perpendicular bisector of ABand AC
a b c
A (1, 3)
⇒ a ,b , c are in AP
⇒ sin A , sin B ,sin C are in AP

216 (d)
Given, the equation of circle is B (-3, 5) P C (5, -1)
2 2
x + y −4 x−6 y−12=0 Perpendicular bisector of A ( 1 ,3 ) and B(−3 , 5) is
Radius of this circle¿ √ 4 +9+12

P a g e | 80
2 x ( x 1−x 2 ) +2 y ( y 1− y 2 )=( x21 + y 21 ) −( x 22 + y 22 )
⇒ 2 x ( 1+3 )+ 2 y ( 3−5 )=( 1−9 )−( 9−25 )
⇒ 2 x− y +6 …(i)
Similarly, perpendicular bisector of A(1 , 3) and
C (−3 , 5) is
2 x ( 1−5 )+ 2 y ( 3+1 )=(1+9)−(25+1) 7 7
⇒ h= =
(1− √13 )
⇒−8 x +8 y=−16 cot 45 °−cot 60 °
⇒ x− y−2=0 …(ii)
Point of intersection of Eqs. (i) and (ii) is
¿7
√3 (¿ √3+ 1) m¿
P= (−8 ,−10 ) 2
Then, the distance between P and A is
225 (c)
PA=√ ( 1+8 ) + ( 3+10 )
2 2
Here, coordinates are A ( 2 , 4 ) , B(−2 ,−4) and
¿ √ 81+169
C (2 ,−4)
¿ √ 250= √25 ×10=5 √10
y
A(2, 4)
222 (b)
Let sides are a=13 , b=12 , c=5
x' x
Now, a 2=b2 +c 2
2 2 2
⇒ ( 13 ) =( 12 ) +5
B(-2, -4) C(2, -4)
⇒ 169=169
y'
∴ ∠ A=90°
Since, R=
a
=
13
=
13 √
Now, | AB|= (−2−2 )2+ (−4−4 )2
2 sin A 2sin 90° 2 ¿ √ 16+64=√ 80=4 √ 5
223 (b) 226 (c)
Since, the vertices of the triangle are Let the vertex A ( x , y ) is equidistant from B and
( a cos t , a sin t ) ,(b sin t ,−cos t) and (1, 0) C
Let the coordinate of centroid be ∴ ( x−1 )2+ ( y−3 )2= ( x +2 )2 + ( y−7 )2
a cos t +b sin t+ 1 ⇒ 6 x−8 y+ 43=0
x=
3 Only the option (c) satisfy it
⇒ 3 x−1=a cos t +b sin t …(i)
a sin t−b cos t+0 227 (a)
and y=
3 Let the ratio be k :1
3 y=a sint−b cos t …(ii) −7 k +3 1 1
∴ = ⇒k=
On squaring and adding Eqs. (i) and (ii), we get k +1 2 3
( 3 x−1 )2 + ( 3 y )2=a 2 ( cos 2 t +sin2 t ) + b2 (si n2 t +cos 2 t) Hence, ratio is 1 :3 internally
2 2 2 2
⇒ ( 3 x−1 ) + ( 3 y ) =a + b 228 (d)
We know that orthocenter of the right angled
224 (c)
triangle ABC , right angled at A is A
In ∆ ABC , BC =h cot 60 °
Here, triangle is right angled at O(0 , 0)
and in ∆ ABD , BD=h cot 45°
Since, BD−BC=DC
∴ Orthocentre =(0, 0)
⇒ h cot 45 °−h cot 60 °=7 229 (a)
Here, x 2− y 2 +2 y=1
2 2
⇒ x =( y −1 )
⇒ x= y−1 and x=− y +1

P a g e | 81
sin A sin B sin C 1
We have, = = =
a b c 2R
a
⇒ sin A=
2R
a b c
∴ 2 R 2 sin A sin B sin C=2 R2 ∙ ∙
2 R 2 R 2R
abc
¿ =∆
4R
Which could be graphically as shown in figure
233 (c)
Which gives angle bisector as y=1 and x=0
1
∴ Required area¿ ×2 ×2=2 sq units
2

230 (a)
12
In ∆ ABE , tan 60 °= ⇒ AB=4 √ 3 m
AB In ∆ BAD ,
AD
cos ( 90°−B )=
c
⇒ AD=c sin B
Similarly, BE=a sin C and CF=b sin A
Since, AD , BE ,CF are in HP
∴ c sin B , a sin C , b sin A are in HP
and in ∆ ACE 1 1 1
12 ⇒ , , are in AP
tan 45 °= ⇒ AC=12 m sin C sin B sin A sinC sin B sin A
AC ⇒ sin A , sin B ,sin C are in AP
In
∆ ABC , BC =√ A C2− A B2= √ 144−48=4 √ 6 m 234 (d)
∴ Area of rectangular field= AB × BC Let P be the middle point of the line segment
joining A(3 ,−1) and B(1 ,1) is (2, 0)
¿ 4 √ 3 × 4 √ 6=48 √ 2 sq m Q

231 (a)
In ∆ ABD ,
2 2 2
2 +5 −B D A B
cos 60 °= (3, -1)
P
(1, 1)
2 ( 2 ) (5)
2 Let P be shifted to Q by 2 unit and y -coordinate
⇒ B D =19
of Q is greater than that of P
Now, in ∆ BCD
1−(−1)
Now, slope of AB= =1
1−3
then slope of PQ=−1
The coordinate of Q becomes
( 2 ±2 cos θ ,0 ± 2 sinθ ), where tanθ=1
2
C D +9−19 ie ,(2 ± √ 2 , ± √ 2)
cos 120 °=
(2 )( 3 ) (CD ) As, y -coordinates of Q is greater than that of P
2
⇒ C D +3 CD−10=0 ∴ We take Q=(2+ √ 2 , √2)
⇒ CD=−5 ,2
235 (d)
⇒CD=2 ( ∵ CD ≠−5)
1
×6 ×|α|=15
Area of triangle ¿
232 (c) 2
⇒|α|=5 ⇒ α =± 5

P a g e | 82
and β can take any real value 236 (a)
Let points are A ( 2 ,3 ) , B ( 3 , 4 ) , C ( 4 , 5 ) , D(5 ,6)
∴ AB=a= √ ( 3−2 ) + ( 4−3 ) =√ 2
2 2

Similarly, BC=b= √ 2 , CD=c=√ 2


And DA=d=3 √ 2
a+ b+c +d
Now, s=
2
¿
√2+ √2+ √ 2+3 √ 2 =3 √2
2
∴ Area= √( s−a ) ( s−b ) ( s−c ) (s−d )
¿ √ ( 3 √ 2−√ 2 )( 3 √ 2− √ 2 ) ( 3 √ 2−√ 2 ) ( 3 √ 2−3 √ 2 )
¿0

237 (a)
√ ( 2+5 ) +( 3−2 ) =√ ( x −1 ) + (2−3 )
2 2 2 2

2
⇒ 49+1=( x−1 ) +1
⇒ x−1=±7 ⇒ x=−6 , 8

238 (d)
In an equilateral triangle length of median

CD=
√3 a
2
C

a
O a

A a B
D

Also, centroid of triangle lies on the centre of


circle

∴ OC= × √ a=
2 3 a
3 2 √3
2
πa
∴ Area of ¿˚ π ( OC ) =
2
sq units
3

239 (a)
CB
In ∆ ABC , tan 30 °=
CA
1 CB
⇒ =
√3 6
6 √3
⇒ CB= =2 √ 3
3

P a g e | 83
y ∆ ∆ ∆ ∆ ∆ 2
4
rr 1 r 2 r 3= . . . = 2 =∆
s ( s−a ) ( s−b ) (s−c ) ∆
A (6, 0)
2. Now, r 1 r 2 +r 2 r 3+ r 3 r 1

∆∆ ∆∆ ∆∆
o

¿ + +
( s−a )( s−b ) ( s−b ) (s−c) (s−c ) ( s−a )

[ ]
B ( s−c )+ ( s−a )+(s−b)
x' x 2
C ¿∆
( s−a )( s−b ) (s−c)
y' 2
∆ [3 s−(a+ b+c )] 2
1 ¿ =s
∴ Area of ∆ ACB= × CA ×CB ∆ 2 /s
2
1 244 (c)
¿ ×6 × 2 √3=6 √3 sq units
2 Let angles of a triangle are x ,2 x abd 7 x
respectively
240 (d)
∴ x+2 x +7 x=180 ° ⇒ x =18°
cos A cos B cos C
We have, = = Hence, the angles are 18 ° ,36 ° , 126 °
a b c
Greatest side ∝ sin18 °
cos A cos B cos C
⇒ = = sin 126°
k sin A k sin B k sin C ∴ Required ratio ¿
sin 18°
⇒ cot A=cot B=cot C
sin(90 ° +36 °)
⇒ A=B=C=60 ° ¿
sin 18 °
∆ ABC is an equilateral triangle
cos 36 ° √ 5+1
∴ ∆= √ a2= √ ( 2 ) [ ∵ a=2( given)]
3 3 2 ¿ =
sin 18 ° √ 5−1
4 4
¿√3 246 (a)

| |
241 (c) x1 y1 1
1
b +c −a
2 2 2 Area of triangle ¿ r x 1 r y 1 1 =0
We have, cos A= 2 2 2
2 bc r x 1 r y1 1
or c −2 bc cos A +b −a2 =0
2 2
∴ Points are collinear
∴ c 1+ c2 =2b cos A=2× 2× √ =2 √3
3
247 (d)
2
2 2 Given, new coordinates¿(4 ,−3) and θ=135°
and c 1 c 2=b −a =4−5=−1
∴ 4=x cos 135 °+ y sin 135 °
∴|c 1−c 2|= √( c1 +c 2 ) −4 c 1 c 2
2
−x y
⇒ 4= + …(i)
¿ √ 12+ 4=√ 16=4 √2 √2
and −3=−x sin 135 °+ y cos 135 °
242 (c) −x y
C 2C
⇒−3= − …(ii)
( a−b )2 cos 2 2
+ ( a+ b ) sin √2 √2
2 2
On adding Eqs. (i) and (ii), we get
C 2C
¿ ( a + b −2 ab ) cos + ( a +b +2 ab ) sin
2 2 2 2 2
−2 x −1
2 2 1= ⇒ x=
√2 √2
C
2(
¿ a 2+ b2 +2 ab sin 2 −cos 2
C
2 ) On subtracting Eqs. (i) and (ii), we get
2y 7
2 2 7= ⇒ y=
¿ a + b −2 ab cos C √2 √2
¿ a 2+ b2−( a2+ b2−c2 ) =c 2

243 (a)
Thus ( x , y ) =
−1 7
¸
(
√ 2 √2 )
1. 248 (d)

P a g e | 84
ac ( b )
¿ =4 R
We know, if coordinate axes are rotated, then ∆
p= ( x cos θ− y sin θ , x sin θ+ y cos θ )
It is rotated at an angle 135 ° ie , θ=135 ° and the 254 (c)
new point be
Areaof ∆ CAB=area of ∆ CAD+ areaof ∆CDB
1 1 C 1 C
p= [ 4 cos ( 90° + 45 ° ) ] + 3 sin(90 °+45 ° ), 4 sin(90°+ 45 °)−3
⇒ bacos(90 ° + 45b °)¿
sin C= ∙ CD sin + a ∙CD sin
2 2 2 2 2
¿ [−4 sin 45 ° +3 cos 45 ° , 4 cos 45 ° +3 sin 45 ° ]
2ab C

[( ) ](
⇒ CD + cos
¿ 4.
−1
√2
1
+3. , 4. +3.
1
√ 2 √2 √ 2 √2 √ 2
1
=
−1 7
, ) a+b 2

255 (c)
249 (d) Let h be the height of a tower.
Let the height of the tower be h meters Since, ∠ AOB=60°
D Sun
∴ ∆ OAB is an equilateral triangle.
∴ OA=OB= AB=a
C
h h
O 90o
30 o 45o a a
60o 30o
A 60 m B C
B a A
h
In ∆ BCD , tan 45 °= h a
BC In ∆ OAC , tan 30 °= ⇒ h=
⇒ BC =h ...(i) a √3
h
In ∆ ACD , tan30 °= 256 (d)
AC
∆=¿
h
⇒ AC = Clearly, ∆ ≠ 0 for any value of α , β , θ, Hence,
tan 30°
points are non-collinear
⇒ AB+ BC= √ 3 h
⇒ 60+ h= √ 3 h[from Eq.(i)] 257 (a)
60 Given, 8 R2=a2 +b2 +c 2
⇒ h=
√3−1 2 2
¿ 4 R (sin A +sin B+ sin C )
2 2

60 ( √3+ 1 ) 2 2
⇒ sin A+sin B+sin C=2
2
⇒h=
2 ⇒ ( cos2 A−sin 2 C ) +cos 2 B=0
⇒ h=30( √ 3+1)m 2
⇒cos ( A−C ) cos ( A+C ) +cos B=0
251 (d) ⇒ 2 cos B cos A cos C=0
2 B ⇒ cos A=0∨cos B=0∨cos C=0
Given, r 1 +r 3=k cos π π π
2 ⇒ A= ∨B= ∨C=
A C 2 B
2 2 2
ie , s tan + s tan =k cos
2 2 2
258 (a)
⇒s
[√ s (s−a)
+
s(s−c )√
( s−b ) (s−c) ( s−a ) (s−b)
=k
s (s−b)
ac ] ∵ AB+ √ ( 4 +1 ) + ( 0+1 ) = √26
BC= √ ( 3+1 ) + ( 5+1 ) =√ 52
2
2

2
2

⇒ k=
ac
s−b [√ s( s−a)
+

( s−b ) (s−c) ( s−a ) (s−b)
s(s−c ) ] CA=√ ( 4−3 ) + ( 0+5 ) =√ 26
2

So, in isosceles triangle side AB=CA


2

×√
[ √ ( s−c ) + √( s−a )
]
2 2
¿
ac s−b Also, ( √ 26 ) + ( √26 ) =52
s−b √s √( s−a ) √ ( s−c ) So, triangle is right angled and also is isosceles
ac (2 s−a−c) triangle
¿
√ s (s−a)(s−b)( s−c) 259 (c)

P a g e | 85
By Standard results, ⇒ ( a−b ) ( b2 +b−a2 +a )=0
A B C ⇒ ( a−b )( a+b )( b+1−a )=0
3. r =4 R sin sin sin
2 2 2
⇒ Either a−b=0∨( a+b ) =0∨ ( b+1−a )=0
A ⇒ a=b+ 1
4. r 1=s tan
2
263 (d)
∆ Let A A1 , B B1 and C C 1 be the towers and O be
5. r 3=
s−c the circumcentre of ∆ ABC
∠ A1 OA=θ A ,∠ B1 OB=θB , ∠ C 1 OC =θC
260 (c)
Now, OA= A A 1 cot θ A
On solving given equation of lines, we get the
OB=B B 1 cot θ B
points A ( 4 , 0 ) ,(0 , 3)B(0 , 3) and O(0 , 0)
and OC =C C1 cot θC

∴ Area of ∆ OAB
1 Since,O is the circumcentre of a triangle
¿ ×OA ×OB
2 ∴ OA=OB=OC
1 ⇒ A A1 cot θ A =B B1 cot θB =C C 1 cot θ c
¿ × 4 ×3
2 tan θ A tan θB tanθ C
¿ 6 sq units ⇒ = =
A A1 B B1 C C1
261 (a) Any other relationship between tanθ A , tan θ B
Let (x , y ) denotes the coordinates in A , B and C and tanθ C cannot be establish
plane
264 (b)
( x−1 )2+ y 2 1
Then, 2
= Let a=6 , b=5 , c=√ 13
( x+ 1 ) + y 2 9 2 2
6 +5 −13 4
2 2 2
⇒ 9 x +9 y −18 x +9=x + y +2 x+1
2 ∴ cos C= =
2× 6 ×5 5
2 2


⇒ 8 x +8 y −20 x +8=0 16 3
2 2 5 Now, sin C= 1− =
⇒ x + y − x +1=0 25 5
2 1
∴ Area of ∆ ABC= ab sin C
5
∴ A , B , C lie on a circle with C , 0
4 ( ) 1 3
2
¿ ×6 × 5× =9 sq unit
2 5
262 (c)
Since, points ( a , b ) ,(b , a) and (a 2 ,−b2 ) are 265 (b)
collinear Since, y 1 , y 2 , y 3and x 1 , x 2 , x 3 are in AP

| |
a b 1 ∴ y 2− y 1= y 3− y 2 and x 2−x 1=x 3−x 2
∴ b a 1 =0 y 3− y 2 y 2− y 1
2
a −b 1
2 So, =
x 3−x 2 x 2−x1
Applying, R2 → R2 −R 1 and R3 → R 3−R1, we get So, points are collinear

| |
a b 1
266 (a)
b−a a−b 0 =0
2 2 Let ∠ RPQ=θ∧∠ RQP=ϕ
a −a −b −b 0

P a g e | 86
y
R (h, k) Since, Q is the image of P
y
Q
y=x
x O
x'
Q M P
(-a, 0) (a, 0)
y' P'
∴ θ−ϕ=2 α
Let RM ⊥ PQ, so that RM =k , O
x
MP=a−h∧MQ=a+h P (4, -1)
RM k
Then, tanθ= = ∴ P Q=2 P P' '
MP a−h
¿ 2∨4−(−1)∨ ¿ =5 √ 2 ¿
RM k
and tan ϕ= =
MQ a+h
√ 1 + 12
2

Again now, 2 α =θ−ϕ 269 (d)

| || |
∴ tan 2 α =tan ( θ−ϕ ) a b +c 1 a a+ b+c 1
tan θ−tan ϕ 1 1
¿ Area ¿ b c+ a 1 = b a+ b+c 1
1+ tan θ tan ϕ 2 2
c a+ b 1 c a+ b+c 1

| |
k ( a+h )−k ( a−h )
¿ 2 2 2 ( a+b+ c ) a 1 1
a −h + k ¿ b 1 1 =0
2 2 2 2
⇒ a −h + k =2 h k cot 2 α c 1 1
Hence, the locus is x 2− y 2 +2 xy cot 2 α −a 2=0
270 (c)
267 (c) We have,
2 2 2 2
Since, the centroid G divide the line AD in the x + 4 xy + y =a X +b Y
ratio 2:1 2
⇒ ( X cos θ+Y sin θ ) +14 ( X cos θ+ Y sin θ )( X sin θ−Y cos θ
⇒ a=1+ 4 sin θ cos θ , b=1−4 sin θ cos θ and
2 2
sin θ−cos θ=0
⇒ a=1+ 4 sin θ cos θ , b=1−4 sin θ cos θ and
π
θ=
4
8 4 ⇒ a=3 , b=−1
∴ AG= ∧DG=
3 3
π AG 271 (a)
In ∆ ABG , tan = Given, distance r =√ 2 and θ=45 °
3 BG
π 1
⇒ BG= AG cot ∴ x=4 + √ 2cos 45 °=4+ √ 2 . =5
3 √2
8 1 8 1
⇒ BG= × = and y=3+ √ 2 sin 45° =3+ √ 2. =4
3 √ 3 3 √3 √2
1
Areaof ∆ ADB= × AD × BG 272 (d)
2
Let the height of the cliff be BD=50 m and height
1 8 16
¿ ×4× = of the tower be AE=h metre.
2 3 √3 3 √ 3 In ∆ DEC ,
Since, median divides a triangle into two triangle
50−h
of equal area. Therefore, tan30 °=
x
Area of ∆ ABC=2× area of ∆ ADB
50−h
16 32 ⇒ x= =√ 3 (50−h )
¿2× = sq units 1 …(i)
3 √3 3 √ 3
√3
268 (b)

P a g e | 87
D 276 (b)
Line perpendicular to OA passing through B is
50 - h x=3
E
x C
50 m
h

A x B

and in ∆ BAD ,
50
tan 45 °= ⇒ x=50 m 3 √ 3−0
x Slope of AB+ =−√ 3
3−6
From Eq. (i),
Line perpendicular to AB through origin is
50=√3 (50−h)
1
50( √ 3−1) y= x
⇒ h= m √3
√3 ∴ The point of intersection of a line x = 3 and

(
⇒ h=50 1− √ m
3
3
) y=
1
√3
x is (3 , √ 3) , which is the required

orthocenter
273 (a)
Let the vertex of triangle be A(2,−1) and 277 (a)
equation of BC is We know that, in any ∆ ABC
x +2 y=1 A B C
cos A+ cos B+cos C=1+ 4 sin sin sin
2 2 2
A B C
and r =4 R sin sin sin
2 2 2
r
∴ cos A+cos B+ cos C=1+
R

Since, the triangle is equilateral triangle 278 (b)


∴ ∠ ABC =60 ° Given, sin P , sinQ , sin R are in AP

| |√
2 (1 )+2 (−1 )−1 1 ⇒ a ,b , c are in AP
and AD= = sin P sin Q sin R
√ 1+ 2 2
5 ∵
a
=
b
=
c
= λ(say )
AD P
In ∆ ABD , =sin 60 °
AB
1 c b
p1

⇒ AB=
√5 = 2
Q a R
√3 √15
2 Let p1 , p2 , p 3 be altitudes from P , Q, R
2 ∴ p 1=c sinQ=λbc ,
⇒ AB=BC =AC =
√15 p2=a sin R=λac ,
p3=b sin P=λab
275 (d)
Since, a , b , c are in AP,

| |
2 2 1
1 1 1 1
Area of ∆= 5 5 1 ∴ , , are in HP
2 a b c
6 7 1
abc abc abc
1 ⇒ , , are in HP
¿ [2 ( 5−7 )−2 ( 5−6 )+ 1(35−30)] a b c
2 ⇒ bc , ac , ab are in HP
1 3
¿ (−4 +2+5 )= sq units ⇒ λbc , λac , λab are in HP
2 2

P a g e | 88
⇒ p1 , p 2 , p3 are in HP 280 (c)
BD AD
In ∆ ABD , =
sin θ sin B
sin θ
⇒ BD= AD= …(i)
sin B

CD AD
In ∆ ACD , =
sin( A−θ) sin C
sin ( A−θ)
⇒ CD= AD
sin C
∴ BD=CD
sin θ sin( A−θ)
⇒ AD =AD
sin B sin C
sin C c
⇒ sin (A−θ)= sin θ= sin θ
sin B b

282 (b)
Since, circle is inscribed in the quadrilateral
ABCD, then
a+ c=b+ d …(i)
And quadrilateral is cyclic, then
A+C=π
or C=π− A …(ii)
now, in ∆ ABD
2 2 2
a +d − ( BD )
cos A=
2 ad
2 2 2
⇒ B D =a + d −2 ad cos A …(iii)
And in ∆ BCD ,
2 2 2
b + c − ( BD )
cos C=
2 bc
2 2 2
⇒ ( BD ) =b + c −2bc cos C
2 2
¿ b + c −2bc cos (π − A) [from Eq.(ii)]
2 2 2
⇒ ( BD ) =b + c +2 bc cos A …(iv)
From Eqs. (iii) and (iv), we get
2 2 2 2
a +d −b −c
cos A= …(v)
2(bc +ad )
Now, from Eq. (i),
( a−d )2=( b−c )2
⇒ a2+ d 2−b 2−c 2=2 ( ad−bc ) …(vi)
∴ From Eqs. (v) and (vi),
cos A= (
ad−bc
ad+ bc )
283 (c)

P a g e | 89
We have, a+ b+c= λ=2 s 288 (a)
λ sin A sin B sin C
∴ s= We have, = =
2 a b c
C B s( s−c ) s (s−b) b+ c sin B+sin C
b cos 2 +c cos 2 =b +c ∴ =
2 2 ab ac a sin A
s
¿ { s−c +s−b }
a
¿
sin (B+ C
2 ) (
cos
B−C
2
=
cos) B−C
2 ( )
s λ A A A
¿ ∙ a=s= sin cos sin
a 2 2 2 2
[ A +B +C=π ⇒ A=π−(B+C)]
284 (c)
1 c 2+ a2−b 2 289 (a)
cos B= =
√2 2 ac Let h be the mid point of BC . Since, ∠ TBH =90 °
⇒ √ 2 ac=c +a −b , then T H 2=B T 2+ B H 2=52 +52 =50
2 2 2

⇒2 a 2 c 2=c 4 + a4 + b4 +2 c 2 a2−2 ( c 2 +a 2) b 2 Also since,


2 2 2
⇒ c 4+ a4 + b4 =2 ( c 2+ a2 ) b2 ∠ THG=90 ° ,T G =T H +G H =50+25=75
Let θ be the required angle of elevation of G at T
285 (a) GH
Then, sin θ=
Let the point of the line divides the line in the TG
ratio m :1 5 1
¿ =
∴ Coordinates of point are ( 5m+m+11 , 7m+1
m−1
) 5 √3 √ 3
⇒ θ=sin (1 / √ 3)
−1

Which lies on y + x=4


7 m−1+5 m+1
⇒ =4
m+1
12 m
⇒ =4 ⇒ 12m=4 m+4
m+1
1
⇒ 8 m=4 ⇒ m=
2 290 (d)
∴ Required ratio is 1 :2 1 1 1
Given, ∆= a p1= b p2 = c p3
2 2 2
286 (d) 2∆ 2∆ 2∆
Since, line perpendicular to OA passing through B ∴ p 1 = , p2 = , p3 =
a b c
is x=4
( )
3 3 2 2 2
8∆ 8 abc a b c
−3 ∴ p 1 p2 p 3= = =
Slope of AB= abc abc 4 R 8R
2
2
Line perpendicular to AB through origin is, 291 (b)
2 BC
y= x In ∆ BCD , cot 60 °=
3 300
D
o
30

300 m

60o
30o
∴ The point of intersection of lines x=4 A B C
2
3
8
3 ( )
And y= x is 4 , . Which is the required
⇒ BC =300×
1
√3
…(i)
orthocenter

P a g e | 90
AC
In ∆ ACD , cot 30 °=
300 Triangle is right angled triangle. In right angled
⇒ AC=300 √3 …(ii) triangle mid point of hypotenuse is circumcentre
∴ Distance between two boats ¿ AB So, coordinates of the circumcentre are (5, 2)
300 y
¿ AC−BC=300 √3− [using Eqs.(i)and (ii)] (8, 6)
√3
x' x
(3−1) 600 × √ 3 O
¿ 300 = =346.4 m
√3 3 (2, -2) (8, -2)
y'
292 (d)
296 (c)
∵ 32+ 4 2=52
sin ( y + z) sin C
⇒ Given triangle is a right angled triangle whose From ∆ ABC , =
DC AD
length of hypotenuse is 5 unit
sin x sin B
5 From ∆ ABD , =
∴ R= =2.5 BD AD
2
sin z sinC
From ∆ AEC , =
293 (a) EC AE

( )( )
r1 r1 sin( x + y ) sin B
Given, 1− 1− =2 From ∆ ABE , =
r2 r3 BE AE

∴ 1− ( s−b
s−a
1−
s−c
s−a)(
=2 )
( b−a ) (c−a)
⇒ =2
( s−a )2
2 2 2 BE sin B DC sin C
⇒ a =b + c ×
sin ( x + y ) sin( y + z ) AE AD
Hence, triangle is a right angled triangle ∴ =
sin x sin z BD sin B EC sin C
×
294 (a) AD AE
From given, we get BE DC AD AE
¿ × × ×
( ) ( )
A +B A+ B AE AD BD EC
a tan A−tan =b tan −tan B
2 2 2 BD × 2 EC
¿ =4
BD × EC
sin ( A− ) sin ( −B )
A +B A+ B
2 2 297 (c)
⇒ a∙ =b ∙
A+ B A+ B h
cos A ∙ cos cos cos B In ∆ ABC , tan 60 °=
2 2 BC


a sin
A−B
2 ( =
b sin )
A−B
2 ( ) ⇒ BC =h cot 60 °
In ∆ ABD , tan 30° =
h
cos A cos B BD
⇒ sin
A−B
2 ( )
{ a cos B−b cos A }=0
15o
A

A−B 60o
⇒ Either sin =0 or a cos B=b cos A h
2
A−B
=0 ⇒ A=B 30o 60o
When, sin
2 D d C B
or when a cos B=b cos A
⇒ BD=h cot 30 °
⇒ sin A cos B−cos A sin B=0
⇒ BC +CD=h cot30 °
⇒ sin ( A−B )=0 ⇒ CD=h cot30 ° −BC
⇒ A=B ⇒ d=h cot 30 °−h cot 60 ° [from Eq. (i)]
295 (d)
P a g e | 91
∴ Speed of car =distance ¿ D ¿ C ¿ Let h metres be the height of tree CD and x
time taken meters be the width of river
d h cot 30 °−h cot 60°
¿ =
3 3
distance
∴ Time taken¿ C ¿ B=
speed
h cot 60 °
¿
h(cot 30 °−cot 60 °)
h
1 In ∆ BCD , tan60 ° = ⇒ h=√ 3 x … ..(i)
×3 x
¿ √ 3
h
( )
1 and in ∆ ACD , tan30 °=
√ 3− x +20
√3 1 h
3 ⇒ =
¿ =1.5 min √3 x +20
2 ⇒ 3 x=x +20[¿ Eq . ( i ) ]
298 (a) x=10 m
Let h be the height of the tower, then
302 (a)
h=AQ tan α=BQ tan β=CQ tan γ We have,
E 2
2 s=a+b+ c , A =s ( s−a ) ( s−b ) (s−c)
∵ AM ≥GM
h s−a+s−b+ s−c 3
⇒ ≥ √( s−a ) ( s−b ) (s−c)
3
2 1 /3
A
B C Q 3 s−2 s ( A )
⇒ ≥ 1/ 3
⇒ BC =BQ−CQ =h(cot β−cot γ ), 3 s
CA=h(cot α−cot γ ) s3 A 2 s2
⇒ ≥ ⇒ A≤
and AB=h(cot α −cot β) 27 s 3 √3
Now, BC cot α −CA cot β+ AB cot γ
303 (b)
¿ h [cot α ( cot β−cot γ )−cot β ( cot α −cot γ ) +cot γ (cot α−cot β)] 5
¿0 In ∆ DBC , tan α = ...(i)
x
30
299 (a) and in ∆ DAC , tan2 α =
x
On solving the given equations of sides, we get the
coordinates of the vertices of the triangle,
D x C
A ( 2 ,−1 ) , B (6 ,−1) and C (2 ,1) 5m
B
30 m

y
25 m
C (2, 1) A
M E
x
O 2 tan α 30
⇒ =
1−tan α x
2
y = -1 A (2, -1) B (6, -1)
x + 2y = 4 5
()
x=2

2
x 30
⇒ =
The circumcentre of ∆ ABC is the mid point of 25 x
1− 2
BC x
ie , M ≡ ( 82 , 02 )=(4 , 0) ⇒ 2
10 x
x −25 x
=
30

2 2
⇒ 10 x =30 x −750
300 (b)

P a g e | 92
2
⇒ 20 x =750
2 75
⇒x =
2

⇒ x=5
√ 3
2
m

and in ∆ ACD ,
305 (a)
Here, ratio of angles are 4:1:1
CD h 1 h
tan 30 °= = ⇒ =
⇒ 4 x+ x+ x =180° AC x+ 40 √ 3 40+ x
⇒=30 ° 40+ x=h √ 3
∴ ∠ A=120 ° ,∠ B=∠ C=30 ° ⇒ 40+ x=3 x [using Eq.(i)]
⇒ x=20
On putting this value is Eq.(i), we get h=20 √ 3 m

308 (d)
Let H (3 , α ) is the orthocenter
Thus, the ratio of longest side to the perimeter y
a C (3, 4)
¿
a+b+ c
Let b=c=x H

∴ a2=b2 +c 2−2 bc cos A x


(0, 0) A B
2 2 2 2 (4, 0)
⇒ a =2 x −2 x cos A=2 x (1−cos A )
2 2 2 A
∴ Slope of BH × Slope of AC =−1
⇒ a =4 x sin 4
2 ⇒−α . =−1
A 3
⇒ a=2 x sin 3
2 ⇒ α=
⇒ a=2 x sin 60 °= √ 3 x 4
Thus, required ratio is
( 34 )
Hence, orthocenter of a triangle is 3 ,
= √ = √
a 3x 3
a+b+ c x + x + √ 3 x 2+ √ 3
309 (c)
306 (c) Let the axes be rotated through an angle θ . Then,
Let the third angle is θ 2h 4 √3 2π π
tan2 θ= = =− √3 ⇒ 2 θ= ⇒ θ=
π −1 a−b 5−9 3 3
In a triangle, + tan 2+ θ=π
4
310 (a)
π −1
⇒ θ=π− −tan 2 Since, AC=2 BC
4
∴ Coordinates of C are
⇒ tan θ=tan π−
[ ( π4 + tan 2)]
−1

A
(-3, 4) 2
C (x, y)
1
B
(2, 1)
¿−tan ( π4 +tan −1
2)

−1+2
⇒ tan θ= =3
( 4−3 ,
2+ 4
2+ 1 2+ 1 ) ie , ( , 2 )
1
3
1−2
−1 311 (d)
⇒ θ=tan 3
Let P be the image of the origin about the line
307 (d) x + y=1. Since, OA=OB, therfore Q is mid point
CD h of AB
In ∆ BCD , tan60 °= = ⇒ h= √ 3 x …(i)
BC x

P a g e | 93
y h
⇒ 60=h √ 3+ [from Eq. (i)]
√3
P
B(0, 1) ⇒ h=15 √ 3 m

Q 315 (d)
x+y=1 DE
x In ∆ DEC , tan θ=
O (1, 0)A CE
h−a
⇒ tan θ=
∴ Coordinates of Q are , ( )
1 1
2 2
⇒ CE=
CE
h−a
Let the coordinates of P are (x 1 , y 1 ) tan θ
Also, Q is the mid point of OP D
0+ x 1 1 0+ y 1 1
∴ = and =
2 2 2 2
⇒ x 1=1 , y 1=1
E C
∴ The cordiantes of P are (1, 1) h 2

312 (c) a
We have, cosec A( sin B cos C +cos B sin C) 2
A
( )
sin B sin C B
¿ cos C+ cos B
sin A sin A In ∆ ABC ,
a a
¿ ( cos C + cos B )=1
b c tan2 θ= = ( ∵ CE= AB)
a a AB CE
2 tan θ a
( ∵ a=b cos C +c cos B) ⇒ =
1−tan θ
2
h−a [from Eq.(i)]
313 (b) tan θ
Let coordinates of fourth vertex are ( x , y ) 2 tan θ
⇒ a tan θ=(h−a)× 2
x−5 7+3 1−tan θ
∴ =
2 2 ⇒ a tan θ ( 1−tan2 θ )−2 ( h−a ) tan θ=0
⇒ x=15 ⇒ tan θ ( a−a tan 2 θ−2 h+ 2 a )=0
y−4 10+5 ∵ tan θ ≠ 0
and =
2 2 −2 h+3 a
⇒ y =19 ∴ tan 2 θ=
a
∴ Coordinates of fourth vertex are (15, 19)
314 (a)
⇒ θ=tan−1

−2 h+3 a
a
Let AB and DE be two towers of equal height ‘h ’
¿ tan−1
√ 3−
2h
a

316 (c)
Given vertices are A ( 2 a , 4 a ) , B ( 2 a , 6 a ) and
C ( 2 a+ √ 3 a ,5 a ) , a> 0

h h

Now, AB= ( 2 a+2 a )2 + ( 4 a−6 a )2=2a
∴ ∈∆ ABC , tan 60 °= ⇒ x= …(i)
x √3 √
BC= ( √ 3 a ) + a2=2 a
2

Again, in ∆ CDE , tan 30 °=


h and CA = √( √3 a ) + (−a ) =2a
2 2

60−x ∴ AB=BC =CA


⇒ 60−x=h √ 3 Hence, triangle is an equilateral triangle, therefore
it is an acute angled triangle

P a g e | 94
317 (a) On putting x=17 in Eq. (i), we get
Let D and E are the mid points of AB and AC . So, y=17
coordinates of B and C are (−3 , 3 ) and (5, 3) Hence, required point is (17, 17)
respectively
321 (b)
1
Since, tanθ=
2
h
In 3 h ...(i)
∆ ABC , tan α = =
AB 120
Centroid of triangle ¿ ( 1−3+5
3
,
3 )
1+ 3+3 D

( ) 7 2h/3
¿ 1, C
3
h/3
318 (b)
R is mid point of PT A 40 f t B
P(a, x) T(b, y) 3h
Q R S In ∆ ADB , tan β= ...(ii)
120
Point ( 5 a+38 b , 5 x+83 y ) divides PT in ratio 3 : 5 ∴ tan θ=tan( β−α)
tan β−tan α
and that is mid point of QR ⇒ tan θ=
1−tan β tan α
319 (a) 3h h

⇒ =
1 120 120
2
1+
3h
2 ( 1
∵ tanθ= , given
2 )
14400
2h
1 120
⇒ =
2 14400+3 h2
On solving the given equations of lines, we get the 14400
coordinates of the vertices of a ∆ OP Q which are 1 240 h
⇒ =
O ( 0 , 0 ) , P(1, 0) andQ(0 , 1). Since, the triangle is 2 14400+ 3 h2
right angled at O(0 , 0), therefore O(0 , 0) is its 2
⇒ 14400+3 h =480 h
orthocenter 2
⇒ 4800+h −160 h=0
320 (d) ⇒ ( h−40 )( h−120 ) =0
Let the circumcentre of triangle be P(x , y) and Since, the height of the pole is more than 100 m
let the vertices of a ∆ ABC be A ( 0 , 30 ) , B ( 4 , 0 ) ∴ h=120 ft
and C (30 , 0) 322 (c)
2 2 2
∴ P A =P B =P C 3
2 2 2 2 Slope of line OP= , let new position is Q(x , y)
⇒ ( x−0 ) + ( y−30 ) = ( x −4 ) + ( y−0 ) 4
2
¿ ( x−30 ) + ( y −0 )
2 y
slope of OQ= also x 2+ y 2=OQ 2=25=(O P 2)
From Ist and IInd terms, a

| |
2 2
x + y −60 y +900=x + y −8 x+16
2 2 y 3

⇒ 8 x−60 y + 884=0 …(i) x 4 4 y−3 x
tan 45 °= ⇒ ± 1=
From IInd and IIIrd terms, 3y 4 x+ 3 y
1+
2 2 2 2 4x
x −8 x +16+ y =x −60 x +900+ y
⇒ 52 x=884 ⇒ x=17 Straight lines and pair of straight lines
4 x+3 y =4 y −3 x

P a g e | 95
or −4 x−3 y=4 y−3 x H−h
In ∆ ECD , tan ( α −β )=
1 d
x= y …(i)
7 H−h
⇒ tan(α−β )= [from Eq.(i)]
−x=7 y …(ii) H cot α
1 ⇒ H [ 1−cot α tan ( α −β ) ] =h
Correct relation is x= y as new point must lies
7 hcot (α−β )
in Ist quadrant ⇒H=
cot ( α− β ) −cot α
2 2
x + 49 x =25
−1 7 326 (b)
⇒ x= , y=
√2 √2 Let the equation of line be
x y
323 (c)
+ =1
a b
BC
In ∆ ABC , tan 30 °=
AC Since, it passes through ( 15 , 15 )
1 1
∴ + =1
5a 5b
⇒ a+b=5 ab …(i)
Since, the point P(x , y) divides AB joining
A(a , 0) and B(0 , b) internally in ratio 3:1
a 3b 4y
∴ x= , y= ⇒ a=4 x and b=
4 4 3
1 h−150
⇒ = On putting the value of a and b in Eq. (i), we get
√3 60
⇒ h=(150+20 √3) m 4 x+
4y
3
=5( 4 x)
4y
3( )
324 (b) ⇒ 3 x + y=20 xy
a b c
Since, = = =k (say) 327 (d)
sin A sin B sin C
( 3−10+a ) ( 9−20+a )> 0
∴ ( a+b+ c )( a+b−c )=3 ab
2 2 2 or ( a−7 ) ( a−11) >0
⇒ a + b +2 ab−c =3 ab
2 2 2 ∴ a ∈ (−∞ , 7 ) ∪(11 , ∞)
a +b −c 1 π
⇒ = ⇒ cos C=cos
2 ab 2 3 328 (a)
π y=| x−2|⇒ y=x−2 and y=2−x
⇒ ∠ C=
3

325 (b)
H
In ∆ ABD , tan α =
d
D
1
Area of shaded region ¿ 2 ABC =2. .1 .1=1
H -h
2

E - 329 (d)
CH
d A B
cot cot −1
2 2 A B
h =1−tan tan
a B 2 2
cot cot
2 2
A d B
⇒ d=H cot α …(i) ¿ 1−
√ s (s−a) √
( s−b ) (s−c) ( s−a ) (s−c)
s(s−b)

P a g e | 96
s−c c C
¿ 1− =
s s
2c
¿
a+b+ c
D 90o - E

100 m
330 (c)
d
We have, AG=1
1 1 64 m
∴ GD= AG=
2 2
Hence, the coordinates of D are (1 , 1/2) A d B
Clearly, AG is parallel to y -axis and the triangle ⇒ d=64 cot θ ...(i)
ABC is equilateral. Therefore, BC is parallel to x - (100−64)
1 In ∆ CDE , tan ( 90 °−θ )=
axis at a distance of unit from it d
2 ⇒ d=36 tan θ ...(ii)
Y
From Eqs. (i) and (ii), we get
2
A(1, 2) d =36 × 64 ⇒ d=48 m

G(1, 1) 334 (d)


B 1 C We know that area of circle is π r 2
X' D 1, X
2
O (1, 0) If radius, r =a, then A=π a2
And the area of the segment of angle 2 π =π a2
Y'
2

Let a be the length of each side of ∆ ABC . Then, ∴ Area of 1 angle¿ π a



√3 √3 a= 3 ⇒ a=√ 3 2
AD= a ⇒
2 2 2 ∴ Area of 2 α angle ¿ 2 απ a =α a2

Since D is the mid-point of BC and BC= √ 3

∴ BD=CD = √
3 336 (a)
2 Let a=5 k , b=6 k and c=5 k
5 k +6 k +5 k
Hence, the coordinates of B and C are 1− ( √3 , 1
2 2 ) s=
2
=8 k

and 1+( √3 , 1
2 2 ) respectively
∴ r=
s

331 (d)
¿

8 k ( 8 k−5 k )( 8 k −6 k )( 8 k −5 k )
8k


2 2
c sin 2 B+b sin 2 C 8 k .3 k .2 k .3 k 3 k
¿ =
¿ c 2 ( 2sin B cos B ) +b2 ( 2 sinC cos C ) 8k 2
2r 2 ×6
¿ 2 c2( 2∆
ac ) (
cos B +2 b2
2∆
ab
cos C ) ⇒ k= =
3 3
=4

¿ 4 ∆( )
c cos B+b cos C 338 (b)
a Let the height of the vertical tower PQ=h , C is
¿ 4 ∆ ( )=4 ∆
a the middle point of line segment AB. Since, PQ is
a perpendicular to the plane QAB ,
∴ ∠ PQA =∠ PQC=∠ PQB=90 ° , we get
332 (a)
64
In ∆ DAB , tan θ=
d

P a g e | 97
G
b1
F
b2
E

PQ
=tan θ ⇒ QA=h cot θ
QA
Similarly, QB=h cot θ and QC =h cot ϕ
Since, QA=QB, the Δ QAB is an isosceles triangle A B C D
a2 a1
Here, QCA is a right angled triangle in which
Since, a 1 a 2=b1 b2
∠ QCA=90 °
a1 b2
∴ OC 2+ A C2=Q A2 ⇒ =
2 2 2 2 2 b1 a2
⇒ h cot ϕ +a =h cot θ

or h =
2 a
2
⇒ tan ( α +2 β )= tan 1β +γ
2
2
cot θ−cot ϕ
2

a
2
(2)
⇒ tan ( ) tan (
2 )
⇒h = α+ β β+ γ
( cose c2 θ−1 )−(cose c2 ϕ−1) =1
2
2
a α + β β +γ π
¿ 2 2 ∴ + =
cose c θ−cose c ϕ 2 2 2
2
a ⇒ α + β +γ =π−β < π
¿
1 1
2
− 2 341 (c)
sin θ sin ϕ
2 2 2 2 2 2 We know that, 2 s=a+b+ c
2 a sin θ sin ϕ a sin θ sin ϕ
⇒h = 2 = ( a+b+ c ) ( b+ c−a ) (c+ a−b)(a+b−c )
sin ϕ−sin θ sin(ϕ−θ)sin (ϕ−θ)
2

Hence, the required height h of the peak 4 b2 c 2
a sinθ sin ϕ 2 s ( 2 s−2 a )( 2 s−2 b ) (2 s−2 c)
¿ ¿ 2 2
√sin (ϕ +θ)sin(ϕ−θ) 4b c
s(s−a) ( s−b )(s−c )
339 (b) ¿4 ×
bc bc
cos A cos B cos C 2 A 2 A
Given, = = ¿ 4 cos ×sin =sin A
2
K sin A K sin B K sin C 2 2
⇒ cot a=cot B=cot C
⇒ A=B=C=60 ° 342 (b)
⇒ ∆ ABC is an equilateral triangle. Let P ( h , k ) be the required point, then
2 PA=3 PB
∴ ∆ √ a2= √ × = √ sq unit
3 3 1 3
2 2
4 4 6 24 ⇒ 4 P A =9 P B
⇒ 4 [ ( h−0 )2 + ( k −0 )2 ]=9[ ( h−4 )4 + ( k +3 )2]
340 (c)
⇒ 4 ( h2 + k 2 )=9 [ h 2+16−8 h+ k 2+ 9+6 k ]
( )
a α +β
∵ 1 =tan …(i) 2 2
b1 2 ⇒ 5 h +5 k −72h+ 54 k +225=0
∴ Required locus of P(h , k ) is
( )
a2 β+ γ
and =tan …(ii) 2 2
b2 2 5 x +5 y −72 x +54 y +225=0

343 (a)

Here, a= ( 16−5 )2+ (12−12 )2=11
b=√ ( 16−0 ) + ( 12−0 ) =20
2 2

P a g e | 98

And c= ( 5−0 )2 + ( 12−0 )2=13 ∴ OP ×OQ × cos ∠ POQ=8+15=23
∴ Incentre 347 (d)
¿ (
11× 0+20 ×5+13 ×16 11× 0+20 ×12+13 ×12
11+20+13
,
11+20+13
=(7sin
, 9)3 B
sin B
)
=3 sin
B−¿ 4 sin B
sin B
3
2
=3−4 sin B ¿
2
344 (d) ¿ 3−4 (1−cos B)
2
tan
A
tan
C 1
= 4 ( a 2+ c 2−b2 )
We have, ¿−1+
2 2 2 4 ( ac )2


s−b 1
s (s−a) √
( s−b ) (s−c) ( s−a ) (s−b) 1
s (s−c)
=
2
¿−1+
2
a +c
2
2 2

( )
⇒ = ⇒ 2 s−2 b−s=0 ( ac )2
s 2 2
⇒ a+c−3 b=0 ( a2 +c 2 ) 2 2 2
¿−1+ 2
[ ∵ 2 b =a + c (given)]
4 ( ac )
345 (c)
( )
2

A
( a2 +c 2 ) −4 a 2 c 2 c 2−a2
2
2 2 B 2C ¿ =
Since, sin , sin , sin be in HP 4 ( ac )2 2 ac
2 2 2
1 1 1
⇒ , , 348 (a)
A B C are in AP
sin2 sin2 sin 2 Since, ∠ QPC=α
2 2 2
Q
1 1 1 1 r
⇒ − = −
C B B A B
sin2 sin2 sin2 sin2
2 2 2 2
l
ab ac C
⇒ − h
( s−a ) (s−b) ( s−a ) (s−c )
ac bc
¿ − P O
( s−a ) (s−c) ( s−b ) (s−c )
α
( )( ) ( )( )
a b ( s−c ) −c (s−b) c a ( s−b )−b(s−a) ∴ ∠ QPB=∠ BPC =
⇒ = 2
s−a ( s−b )( s−c ) s−c ( s−a ) ( s−b) α r
In ∆ PQB ,sin =
⇒|−abc|−acs+ abc=acs−abc−bcs +abc 2 l
⇒ ab−ac=ac−bc ⇒ ab+ bc=2 ac α
⇒ l=r sec …(i)
1 1 2 2
⇒ + =
c a b h
and in ∆ POB ,sin β=
⇒ a ,b , c are in HP l
⇒ h=l sin β
346 (d)
α
It is given that O is the origin and P(2 , 3) and ⇒ h=r cosec sin β [from Eq.(i)]
2
Q(4 , 5) are two points
2 2
O P + OQ −P Q
2 349 (d)
∴ cos ∠ POQ= 2 2 2
2(OP)(OQ ) O P + P Q −O Q
cos OPQ=
1 2 OP . PQ
⇒ OP ×OQ cos ∠ POQ= { O P + OQ −P Q }
2 2 2
2 ( 4 + 2 )+ 2 −(4 2+ 42 )
2 2 2
¿
1 2 √ 4 +2 . √ 2
2 2 2
⇒ OP ×OQ cos ∠ POQ= { 13+ 41−8 }=23
2 24−32
¿
ALITER If O is the origin and ( 1 1 )
P x , y ,Q (x 2 , y 2 ) 42 √5
are two points, then
OP × OQ ×cos ∠ POQ=x 1 x 2 + y 1 y 2
Here, x 1=2 , y 1=3 , x 2=4 and y 2=5

P a g e | 99
y
6 3
P (2, 4) ¿ = >1
Q (4, 4) 4 2
Which is not possible
Hence, no triangle is possible
x' x
O (0, 0)
y' 353 (b)
Let the height of pole AD=BC=h
⇒ ∠ OPQ=cos−1
( −1
√5 ) In ∆ OBC , tan 60 °=
h
x
350 (a) h
⇒ x=
Let ABbe a hill whose height is h metres and CD √3
be a pillar of height h ’ meters. h
In ∆ AOD , tan30 °=
h−h ' 100−x
In ∆ EDB , tan α = …(i) D C
ED
h h
and in ∆ ACB , tan β= = …(ii)
AC ED
h h

30o 60o
A O x B
100 m
1
⇒ h=( 100− x )
√3
∴ ¿ Eqs . ( i )∧( ii ),
tan α h−h '
=
(
¿ 100−
h 1
√3 √ 3 )
tan β h ⇒ 3 h=100 √ 3−h[from Eq.(i) ]
tan α ' 100 √ 3
⇒h. =h−h' ⇒ h =h ¿ ¿ ⇒ h=
tan β 4
⇒ h=25 √ 3 m
351 (c)
We have, 354 (b)
Areaof ∆ ABC=3 Area of ∆ GAB
Now, The vertices of a triangle are ( 0 , 2 ) , ( 12 , 1) ,(1 , 1)
| | | |
1 4 1
1 0 2 1
Areaof ∆ GAB= × Absolute value of 4 −3 1
2 1 1
−9 7 1 ∴ Area of triangle ¿ 1 1
2 2
1 61 1 1 1
⇒ Area of ∆ GAB= |−10−52+1|= sq. units
2 2
Hence, Area of ∆ ABC=
183
2
sq .units ¿
1
2 [ ( ) ( )]
1 1
−2 −1 +1 −1
2 2
1
352 (a) ¿ sq unit
4
Using sine rule,
a b 356 (c)
=
sin A sin B Since, triangle is isosceles, hence centriod is the
desired point
⇒ √ =
2 2 6
sin 45 ° sin B
6 1
⇒ sin B= ×
2 √ 2 √2

P a g e | 100
B
30o

A 30o C
h
150 m h

∴ Coordinates of R 3 ,( 43 ) O
A
60 D

⇒ h=(150+20 √ 3)m
357 (c)
Let AQ (¿ PQ ) be the broken part of the tree OP . 360 (d)
It is given that OA=10 m and ∠ OAQ=45 ° The sum of the distance
In ∆ OAQ , we have a b a b
+ −1 + −1
OQ a a b a
tan 45 °= ⇒ OQ=10° ¿ +

√( ) ( ) √( ) ( )
OA 1 1
2 2
1
2
1
2
+ +
a b b a

¿ ( ba + ba ) 1

¿ √ a +b2 2
√ 1 1
+
a2 b2

361 (b)
2 2
Also, A Q =O A +O Q 2 1 1 3
Given, + =
⇒ AQ=√ 100+100=10 √ 2 b+c c+ a a+b+ c
∴ OP=OQ+ PQ=OQ + AQ=10+10 √ 2=10 ( √ 2+ 1 ) mts⇒ 1+ b a
+1+ =3
358 (c)
a+ c b+c
⇒ b ( b+c ) +a ( a+ c )=( a+ c ) (b+ c)
Let the new coordinates be P ( x ' , y ' ) after shifting
2 2 2
⇒ a + b −c =ab
origin to P ( x ' , y ' ) ie , x=x ' + h and y= y ' + k 2 2 2
a +b −c ab
2 2
∴ ( x +h ) + ( y + k ) −4 ( x + h ) +6 ( y +k )−7=0
' ' ' ' ∴ cos C= = ⇒ ∠ C=60 °
2 ab 2 ab
' 2 ' 2
⇒ ( x ) + ( y ) +2 ( h−2 ) x +2 ( k + 3 ) y + ( h + k −4 h+6 k −7 ) =0
' ' 2 2

362 (b)
According to the question,
sin A sin B sin C
h−2=0 and k + 3=0 Given, = =
1/ 4 1 /4 1/3
⇒ ( h , k )=(2,−3)
sin A sin B sinC
∴ = =
359 (d) 3 3 4
BC Here, a=3 k , b=3 k and c=4 k , where k is a
In ∆ ABC , tan 30 °= proportionality constant
AC
2 2 2
1 h−150 9 k +9 k −16 k 1
⇒ = ∴ cos C= =
√3 60 2 ×3 k × 3 k 9
60
⇒ h−150= 363 (b)
√3 ( b+ c )( bc ) cos A+ ( a+c ) ( ac ) cos B+ ( a+b )( ab ) cos C

( ) ( )
2 2 2 2 2 2
b + c −a a +c −b
¿ ( b+ c )( bc ) + ( a+ c )( ac ) × + ( a+b )
2 bc 2 ca
1
¿ {( b +c ) ( b +c −a )+ ( a+c ) × ( a +c −b ) +(a+ b)(a +b
2 2 2 2 2 2 2 2
2

P a g e | 101
1 364 (a)
¿ {2 a3 +2 b3 +2 c 3 }=a3 +b3 + c3
2 h
In ∆ DCB , tan θ= …(i)
x
Sun
A

6
B

E 2 3 D x C

h+ 6
and in ∆ ECA , tan θ=
2 √ 3+ x
h h+6
⇒ = [from Eq.(i)]
x 2 √ 3+ x
⇒ 2 √ 3 h+h x=h x +6 x
⇒ 2 √ 3 h=6 x
6x
⇒ h=
2√ 3
From Eq. (i), we get
6x
tanθ= =√ 3
2 √3 x
⇒ θ=60 °

365 (b)
Since, angles A , B ,C are in AP
∴ 2 B= A +C
∵ A +B +C=180°
⇒ B=60°
2 2 2
a + c −b
Now, cos B=
2 ac
2 2 2
1 a +c −b
⇒ =
2 2 ac
2 2 2
⇒ a + c −b =ac
2 2 2
⇒ b =a + c −ac

366 (a)
Let a=7 cm , b=4 √ 3 cm∧c= √ 13 cm
Here, we see that the smallest side is c
Therefore, the smallest angle will be C
2 2
( 7 )2 + ( 4 √3 ) −( √ 13 ) √ 3
∴ cos C= =
2 ×7 × 4 √ 3 2
π
⇒ ∠ C=
6

367 (c)
On solving the given straight lines, we get vertices
of ∆ ABC
Which are A ( 2 ,−2 ) , B ( 1 , 1 ) ,C (−2 , 2)

P a g e | 102
∴ AB=√ (1−2 ) + ( 1+ 2 ) =√ 10 D
2 2

BC= √ (−2−1 ) + ( 2−1 ) =√ 10


2 2

30o

And CA= (2+ 2 )2+ (−2−2 )2=√ 32 h L B
Here, AB=Bc a
45o
⇒ Triangle is isosceles triangle C A
h−a
369 (d) In ∆ BLD , tan 30° =
LB

| |
x1 y1 1 (h−a)
∴ LB= =√ 3 (h−a)
If x 2 y 2 1 =0 , then the points are collinear tan 30 °
x3 y3 1 h
In ∆ ACD , tan 45 °=
LB
370 (a) or h ( √ 3−1 )=√ 3 a
Let ∠ A=45 °∧∠ B=60°
3 a √ 3 ( √ 3+1 ) a
∴ ∠ C=75° ∴ h= √ =
Let smallest and greatest sides are a and c
√ 3−1 2
∴ a :c=sin 45° :sin 75 °
1 √ 3+1
∴ h= ( 3+2√ 3 ) a
¿ : =2 : √ 3+1
√2 2 √2 373 (d)
¿ 2 ( √ 3−1 ) : ( √ 3+1 ) ( √ 3−1) sin 3 B 3 sin B−4 sin B
3
=
¿ ( √ 3−1 ) :1 sin B sin B
2
¿ 3−4 (1−cos B)
371 (b)

( )
2 2 2
1 a +c
In ∆ ADF , tan 60 °=
( )
2 2 2 2
AF a +c −b 2
¿−1+4 =−1+
E D d C 2 ac ( ac )2

=(
2 ac )
2 2
( a +c ) −4 a c c −a
2 2 2 2 2 2
¿ 2
4 ( ac )
1 km

60o
30o 375 (b)
A F B
BD 1
Since, =
1 DC 3
⇒ AF=cot 60 °= ...(i)
√3
1
In ∆ ABC , tan 30 °=
AB
⇒ AB=cot 30 °
⇒ AF+ FB=√3
1 2
⇒ d=√ 3− = [using Eq.(i)]
√3 √ 3 In ∆ ABD , by sine rule
∴ Speed of aeroplane=distance ¿ D ¿ C ¿ π
time taken sin
3 …(ii)
2 AD= . BD
sin∠ BAD
¿
√ 3 ×60 × 60=240 √ 3 km/h In ∆ ADC , by sine rule
10 π
sin
4 …(iii)
372 (a) AD= . DC
sin∠ DAC
Let CD is a tower of height h . AB is building of
From Eqs. (ii) and (iii),
height a

P a g e | 103
π ab sin 2θ
sin ¿
sin∠ BAD 3 BD √ 3 1 1 2
= . = . √2 . =
sin ∠ DAC π DC 2 3 √6 π
sin Since, maximum value of sin 2 θ is 1, when θ=
4 4
ab
376 (b) ∴ ∆max =
2
Since, reflection of the orthocenter of ∆ ABC in
base BC will always lie on the circumcircle of the 380 (d)
triangle ABC , therefore coordinate of a point Let O(x , y) be the circumcentre. Then, G divides
lying on the circumcircle is O ' O in the ratio 2 :1

(1− 1 ×2 4 ,1− 1 ×2 4 ) ie ,(−1 ,−1) and coordinates ∴


2 x+ 0 2∧2 y +1
2+ 1
=
2+1
=3⇒ x=3∧ y=4

of the circumcentre is (2, 0) Hence, the coordinates of O are (3 , 4)


∴ Radius of the circumcentre of ∆ ABC
381 (d)
¿ √ ( 2+1 ) + ( 1 ) =√ 10
2 2
Perpendicular bisector of A(1 , 3) and B(−3 , 5)
377 (a) is
Given, b+ c=2 a , ∠ A=60 ° 2 x− y +6=0 …(i)
Since, ∠ A=60 ° ∠ B+∠ C=120 ° And perpendicular bisector of A(1 , 3) and
Also, b+ c=2 a [given] C (5 ,−1) is
sin B+sin C=2 sin 60° x− y −2=0 …(ii)

[
⇒ ∵
sin A sin B sin C
a
=
b
=
c
=k
] On solving Eqs. (i) and (ii), we get
x=−8 , y=−10
∴ Coordinates of P are (−8 ,−10)
⇒ 2 sin ( ) ( )
B+C
2
cos
B−C 2 √ 3
2
=
2 √
Thus, PA= ( 1+8 )2+ ( 3+10 )2

= √3 [∵ sin 60 °= ]
√3 ¿ √ 81+169=5 √ 10
⇒2 sin 60 ° cos ( )
B−C
2 2
382 (a)
⇒ cos (
2 )
B−C B−C Given sides are 3 x−4 y=0 , 5 x+12 y=0 and
=1 ⇒ =0
2 y−15=0 . The vertices of a triangle are
⇒ ∠ B=∠ C A ( 0 , 0 ) , B ( 20 , 15 ) ,C (−36 , 15 )
Hence, triangle is an equilateral triangle

Now, AB=c= ( 20−0 )2 + ( 15−0 )2
378 (c) ¿ √ 400+225
Let the vertices of the triangle are ¿ 25
A (−2 ,−6 ) , B(−2 , 4 ) and C (1 ,3) BC=a= √ ( 20+36 ) + ( 15−15 )
2 2


Now, AB= (−2+2 ) + (−6−4 ) =10 2 2
¿ 56
BC= √ (−2−1 ) + ( 4−3 ) = √ 10 CA=b=√ (−36−0 ) + ( 15−0 )
2 2 2 2


and CA= (1+2 )2+ ( 3+ 6 )2=√ 90 ¿ √ 1296+225=39
Here, A B =BC 2 +C A2
2 ∴ Incentre
∴ Triangle is right angled triangle at C
∴ ∠ C=90 °
¿ (
a x 1 +b x 2+ c x 3 a y 1+ b y 2+ c y 2
a+ b+c
,
a+b+ c )

( )
So, orthocenter is (1, 3) 56 × 0+39 ×20−36 ×25
,
¿ 56+39+25
379 (a)
56 × 0+39 ×15+ 25× 15

‖ ‖
0 0 1 56+39+25
1
Area of ∆= a cos θ b sin θ 1
2

1
a cos θ −b sin θ 1 ¿ ( −120
120 120 )
,
960

⇒ ∆= ∨[1(−ab sin θ cos θ−ab sin θ cos θ)]∨¿


2
P a g e | 104
¿(−1, 8)

383 (d)
¿
√π ∆ (
1 s−a s−b s−c
+

+
∆ )
The mid point of the line joining the points
(−10 , 8 ) and (−6 ,−12 ) is
¿
√π (
1 3 s−(a+b+ c)
∆ )
1 3 s−2 s 1 s
( −10−6
2
,
2 )
8+12
ie , (−8 , 10 ) . Let (−8 , 10 ) divides
¿
√π
1
.

1
= .
√π ∆
the line joining the points ( 4 , .−2 ) and (−2 , 4 ) in ¿ =
r √π √ A
the ratio m :n
m (−2 ) + n(4 ) 386 (b)
Then, =−8
m+n We have,
⇒−2 m+ 4 n=−8 m−8 n 2 2
x + 4 xy + y =a X +b Y
2 2

m −2 2
⇒ ( X cos θ+Y sin θ ) +14 ( X cos θ+ Y sin θ )( X sin θ−Y cos θ
⇒ 6 m=−12n ⇒ =
n 1 ⇒ a=1+ 4 sin θ cos θ , b=1−4 sin θ cos θ and
∴ Required ratio 2 :1 externally. 2 2
sin θ−cos θ=0
384 (a) ⇒ a=1+ 4 sin θ cos θ , b=1−4 sin θ cos θ and
Given the circle is inscribed in square formed by π
θ=
the lines 4
2 2
x −8 x +12=0 and y −14 y+ 45=0 ⇒ a=3 , b=−1
⇒ x=6 and x=2 , y=5 and y=9
387 (d)
y
Let S(x , y), then
D (2, 9) C (6, 9) y=9 ( x +1 )2+ y 2+ ( x −2 )2+ y 2=2[ ( x−1 )2+ y 2 ]
⇒ 2 x +1+ 4−4 x=−4 x+2
−3
⇒ x=
y=5 2
A (2, 5) B (6, 5)
Hence, it is a straight line parallel to y -axis
x
O
388 (b)
ABCD clearly forms a square Given lines are
∴ Centre of inscribed circle
¿point of intersection of diagonals
¿mid point of AC or BD
¿(2+6 5+ 9
2
,
2 )
⇒ Centre of inscribed circle ¿(4 , 7)

385 (a) 2 2
x −3 y =0 …(i)
Area of a circle ¿ π × ( radius )2 And x=4 …(ii)
∴ A=π r 3 , A 1=π r 21 , A2=π r 22 , A 3=π r 23 Put x=4 from Eq. (ii) in Eq. (i), we get
2
1 1 1 16−3 y =0
∴ + + 16 4
√ A1 √ A2 √ A3 2
⇒ y = ⇒ y=±
1 1 1 3 √3
¿ + + The vertices of the triangle are
r1 √ π r2 √ π r3 √ π

¿
(
1 1 1 1
+ +
√ π r1 r2 r3 ) (
O ( 0 , 0) , P 4 ,
4
√3) (
,Q 4 ,−
4
√3 )

P a g e | 105

⇒ OA =h cot α
( )
OP= 4 2 +
4 2 8
=
√3 √3 In ∆ AOP , tan β=
l
AO


( √43 ) = √83
2
O Q= 4 + 2 T

√(
√3 √3 ) √3
2
4 4 8
P Q= + =
P h
8
⇒ OP=Q P=P Q= l
√3
∴ ∆ OP Q is an equilateral triangle A O

389 (c) l
⇒ tan β= [from Eq.(i)]
Let BC be the incomplete and BD be the h cot α
complete pillar. In ∆ s ABC and ABD , we have ⇒ h=l tan α cot β
BC BD
tan 45 °= ∧tan 60 °= 392 (b)
AB AB
6.
⇒ BC =100 m∧BD =100 √ 3
C B s (s−c) s (s−b)
⇒ BC +CD=100 √ 3 b cos 2 +c cos 2 =b . +c .
2 2 ab ac
⇒ 100+ x =100 √ 3 ⇒ x=100 ( √ 3−1 ) m
D s
¿
a
[ 2 s− ( b+c ) ]=s
xm
Hence, statement I is true
C
A b+c
7. Let cot =
2 2
60o
A
45o cos
2 sin B+sin C
A
⇒ =
100 m B A sin A
sin
2
390 (b)

( ) ( )
The coordinates of the vertices of ∆ OAB are A 2 sin B+C cos B−C
O ( 0 , 0 ) , A (4 ,0) and B(0 , 3) cos
2 2 2
⇒ =
∴ OA=4 , OB=3 and AB=5 A A A
Y sin 2 sin cos
2 2 2

( )
B(0, 3) A B−C
⇒ cos =cos
2 2
3 5
A B−C
X' X ⇒ = ⇒ A+C=B
O(0, 0) 4 A(4, 0) 2 2
π
Y' But A+ B+C=π , therefore B=
2
Hence, the coordinates of the excentre opposite to
But given statement is
the vertex O are
A b+c
( −5 ×0+3 × 4+ 4 × 0 −5 ×0+3 × 0+4 × 3
−5+3+ 4
,
−5+3+ 4
=(6 , 6) ) cot
2
=
2
⇒ ∠ B=90 °

Hence, statement II is not true


391 (b)
h 393 (b)
In ∆ AOT , tan α = ∵ ∠ CDA=β , ∠ DCB=γ ,
OA

P a g e | 106
and ∠ ACB=π −(α + β + γ ) 3 sin B−4 sin3 B 2
⇒ =
C D sin B √ 3−1
2 2( √ 3+1)
⇒ 3−4 sin B=
2
2− √3 2
⇒ =sin B
4
⇒ sin B= √
A a B 3−1
In ∆ ABC , on applying sine rule, we get 2 √2
AB CA ⇒ ∠ B=15 ° , ∠ A=45 °∧∠ C=120 °
=
sin[π −(α + β + γ )] sin γ
396 (d)
a sin γ
⇒ CA= h
sin(α + β+ γ ) In ∆ ABC , tan α =
x+ d
Now, in ∆ CAD , on applying sine rule
⇒ x +d=h cot α
CA CD
=
sin β sin α
CA ∙ sin α
⇒ CD=
sin β
a sin γ sin α
¿
sin β sin(α + β + γ )
and in ∆ ABD ,
394 (b)
h
Let PD be a pole tan β = ⇒ x=h cot β
x
DP
In ∆ DAP , tan 30° = On putting this value in Eq. (i), we get
AD
h cot β +d=h cot α
a
⇒ DP= d
√3 ⇒ h=
cot α −cot β
DP
In ∆ PDB , tan θ=
BD 397 (b)
⇒ tan θ= √ =
a/ 3 1 tan α+ tan β
tan(α + β)=
√2 a √ 6 1−tan α tan β
P y x
+
z z
¿
y x
1− ×
z z
o
D 3 2
0 +
30

3 6 6
o

a a ¿ =1
A C 3 2
1− ×
6 6
a a
B

395 (b)
a 2
Since, = and A=3 B π
b √ 3−1 ⇒ α + β=∠ QPR=
sin A a 4
By sine rule, =
sin B b 398 (a)
sin 3 B 2 Let (x , y ) be the original coordinates of P . Then,
⇒ =
sin B √ 3−1 x=1+cos θ and y=1+ cos ϕ

P a g e | 107
2 θ 2ϕ 399 (b)
⇒ x=2 cos , y =2cos
2 2 Given, r 1=2 r 2=3r 3
∆ 2∆ 3∆
⇒ = =
( s−a ) ( s−b ) ( s−c )
⇒ ( s−b ) =2(s−a) and ( s−c )=3(s−a)


a+ b+c
2
−b=2 (
a+b+ c
2
−a )
and
a+b+ c
2
−c=3
a+ b+c
2 (−a )
⇒ a+c−b=2 (−a+ b+c )
and a+ b−c=3(−a+b+ c)
⇒ 3 a=3 b+ c and 2 a=b+2 c
a 5
⇒ 4 a=5 b ⇒ =
b 4

400 (b)
Since a , b and x are in AP
⇒ 2 b=a+ c
3b
⇒ 3 b=2 s ⇒ s=
2
Now,
A C
sin sin
2
sin
B
2
2
=
√ ac ( s−b )( s−c )( s−b ) (s−a)
( s−a ) (s−c)×bc × ab

3b
−b
s−b 2 1
¿ = =
b b 2

401 (b)
Let AD be the building of height h and BP be the
hill. Then,
h+ x
tan q= …(i)
y
x
and tan p=
y
⇒ y =x cot p…(ii)
P

D p
C

h h
q
A y B
From Eq. (i) and (ii), we get

P a g e | 108
h+ x A 2 2 A
tan q= ¿ cos −sin
x cot p 2 2
⇒ x cot p=( h+ x ) cot q 2A
¿ cos =cos A
h cot q 2
⇒ x=
cot p−cot q
407 (b)
h cot q
⇒ h+ x= +h Let the third vertex of the triangle be (x , y ), then
cot p−cot q
h cot p x+ 4+(−2)
∴Height of hill¿ =2⇒ x =4
cot p−cot q 3
y+ 8+6
and =7 ⇒ y=7
402 (b) 3

() ∴ The coordinates of the third vertex are (4, 7)


2
2 2 3 16
cos A=1−sin A=1− =
5 25
408 (b)
4
⇒ cos A= Let ABCD be a square of each side of length a . It
5
is given that ∠ BPC=60 ° . Let M be the
( 20 ) + ( 21 )2−a2 4
2
midpoint of BC . Then ∠ BPM=∠ CPM =30 °
⇒ =
2.20 .21 5 In ∆ BPM ,we have
2
⇒ a =169 ⇒ a=13 BM
tan∠ BPM =
PM
403 (c)
Given, ∠ C=60 ° , a=2 , b = 4 ⇒ PM =√ 3 BM =
√3 a
2 2 2 2
a +b −c
∴ cos C= In ∆ OPM , we have
2 ab P
2 2 2
⇒ 2 ab cos 60°=a +b −c
2 2 2
⇒ ab=a + b −c 30o
2 2
⇒ 8=4 +16−c ⇒ c =12
⇒ c=√12=2 √ 3 30o

2√
3 D C
We have, a sinC 2 1
sin A= = =
c 2 √3 2
⇒ A=30 ° M

4√
O
3
and b sin C 2
sin B= = =1⇒ B=90 °
c 2√3
A B
405 (b) 2 2 2 3a a
2
2
2
2 2
b +c −a
2 2 2 P M =O M +O P ⇒ = + h ⇒ a =2h
b− 4 4
b−c cos A 2b 409 (c)
= 2 2 2
c−b cos A b +c −a C 2C
c− ( a−b )2 cos 2 2
+ ( a+ b ) sin
2c 2 2

( ) 2C 2C
2 2 2
b + a −c c ¿(a +b −2 ab)cos + ( a + b +2 ab ) sin
2 2 2 2
¿ 2 2 2
= 2 2
c +a −b b

¿
2
b +a −c
2 ab
2
2 ac
∙ 2 2 2=
2
cos C
c +a −b cos B
C
¿ a 2+ b2 +2 ab sin 2 −cos 2
2 ( C
2 )
2 2
¿ a + b −2 ab cos C
406 (b) 2 2 2 2
¿ a + b −(a +b −c )
2

s (s−a) ( s−b ) (s−c )


We have, −
bc bc

P a g e | 109
( )
2 2 2 C
2 a +b −c
¿ c ∵ cos C= 60o
2 ab

410 (c) h

Given, ∆=a 2−( b−c )2 60o


A B
¿ ( a+ b−c )( a−b+ c ) 1
¿ 2 ( s−c ) ∙2 (s−b) 3+1

√ s ( s−a )( s−b ) ( s−c )=4 ( s−b )( s−c ) h


¿
1
1
⇒ =
4
A 1

( s−b ) ( s−c )
s ( s−a )
=tan
A
2 ⇒ h=
√3 (√3−1)
3−1
∴ tan = 3−√ 3
2 4 ¿ m
2
411 (a)
414 (d)
If O ' is the orthocentre of a ∆ ABC , then the 2 2 2
b + c −a
points O ' , A , B ,C are such that each point is the ∵ cos A=
2 bc
orthocentre of the triangle formed by the
remaining three points. So, the coordinates of the
( 8 ) + ( 10 ) −( 6 )2 4
2 2
¿ =
orthocentre of ∆ O ' AB are (0 , 0) 2 ×8 × 10 5
3
⇒ sin A=
412 (c) 5
cot B+cot C−cot A ∴ sin 2 A=2 sin A cos A
cos B cos C 3 4 24
¿ + −cot A ¿2× × =
sin B sin C 5 5 25
sin C cos B+cos C sin B
¿ −cot A 415 (d)
sin B sin C
1+cos ( A+ B ) cos C
sin(B+C) cos A
¿ − 1+ cos ( A−C ) cos B
sin B sin C sin A
2
sin A−sin B sinC cos A 1+cos [ π −( A+ B ) ] cos( A−B)
¿ ( ∵ A + B+C=π ) ¿
sin A sin B sin C 1+ cos [ π−( A+C ) ] cos( A−C)
2
a −bc cos A 1−cos ( A+ B ) cos ( A−B)
¿ ¿
k (abc) 1−cos ( A−C ) cos ( A+C )
2 2 2 2 2 2 2
(b + c −a ) 1−cos A +sin B sin A +sin B
a2−bc ¿ 2 2
= 2 2
2 bc 1−cos A+ sin C sin A+ sin C
¿
(abc)k 1+cos ( A−B ) cos C a 2+ b2
∴ = [by sine rule]
2 a2−(3 a2−a 2) 1+cos ( A−C ) cos B a2 +c 2
¿ ( ∵ b2 +c 2+ ¿ 3 a2 given)
2(abc )k
2 2 416 (a)
(a −a )
¿ =0 500 500
abck ¿ ∆ AEC , tan 60 °= ⇒ d 1= m …(i)
d1 √3
413 (c)
Let h be the height of the tower.

P a g e | 110
and in ∆ BEC , tan 30 °=
500
d2 Similarly,
x 2+ x3 =6 ,
y 2 + y 3=10 } …(ii)

⇒ d2 =500 √ 3 m …(ii)
∴ Required diameter,
500 2000
And
x 1+ x 3=−2,
y 1 + y 3=−4 } …(iii)

AB=d 1 +d 2= +500 √3= m On solving Eqs. (i), (ii) and (iii), we get
√3 √3 x 1=2 , x 2=10 , x 3=−4
417 (d) And y 1=−6 , y 2=8 , y3 =2
Using sine rule in ∆ ACD , Now, the vertex opposite to D is C ie ,(−4 , 2)
A
419 (a)
Let A ( x 1 , y 1 ) , B (x 2 , y 2) and C (x 3 , y 3 ) be the
90 o
90o

c
vertices of the triangle ABC and let P(h , k ) be
A-

any point on the locus,


90o + C 90o + C Then, P A 2 + P B2 + P C2 =c (constant)
B D C 3
⇒ ∑ [ ( h−x i ) + ( k− y i ) ]=c
a 2 2

i=1
sin ( 90 °−C ) sin 90 ° 2b
= ⇒ cos C= 2h 2k
3
b a a ⇒ h + k − ( x 1+ x 2+ x 3 )− ( y 1+ y 2+ y3 ) + ∑ ( x 2i + y 2i )−
2 2

c 3 3 i=1

And in ∆ ABD , So, locus of (h , k ) is


sin( A−90 °) sin( 90° +C ) 2 2 2x 2y
= x + y − ( x 1 + x 2+ x 3 ) − ( y + y + y ) + λ=0
a c 3 3 1 2 3
3
2
Where λ=∑ ( x i + y i ) −c=0 (constant)
2 2
cos A cos C −b
⇒− = ⇒cos A= i=1
a c c Clearly, the locus of a point is a circle with, centre
2 at

( x + x3 + x , y + y3 + y )
2 2 2
b +c −a −b 2 2 2
⇒ = ⇒ c −a =3 b 1 2 3 1 2 3
2 bc c
2
−2 b 2 2 2
∴ cos A cos C= = (c −a ) 420 (c)
ac 3 ac
A +C
Since, B=
418 (a) 2
Let the vertices of triangle of A , B ,C are B
⇒ B=90 °− ⇒ B=60 ° [ ∴ A+ B+C=180 ° ]
( x 1 , y 1 ) , ( x 2 , y 2 ) ,(x 3 , y 3 ) respectively 2
Given mid points of the sides AB , BC and CA of sin 60 ° sin C
∴ = [by sine rule]
∆ ABC are D ( 6 , 1 ) , E(3 ,5) and F (−1 ,−2) √3 √2
respectively 1
⇒ sin C= ⇒ C=45 °
√2
∴ ∠ A=180 °− (60 °+ 45° )=75 °

421 (b)
Since, A ( 0 , 1 ) , B(0 ,−1) and C (x , 0) are the
vertices of an equilateral∆ ABC .
x +x y +y ∴ AB=BC
∴ 1 2 =6 , 1 2 =1 ⇒ √ 0+ 4=√ x 2 +1
2 2

}
2
x + x =12, ⇒ x =3
⇒ 1 2 …(i)
y 1+ y 2=2 ⇒ x=± √ 3

422 (c)
P a g e | 111
Let a=sin α ,b=cos α , c=√ 1+sin α cos α 1 3 2
⇒ tan θ+ =3 tan θ− tan θ
Here, we see that the greatest side is c 2 2
2 2 2 2
a +b −c ⇒ 3 tan θ−4 tan θ+1=0
∴ cos C=
2 ab ⇒ ( 3 tanθ−1 ) ( tan θ−1 )=0
2 2
sin α cos α −1−sin α cos α 1
⇒ cos C= ⇒ tan θ= ,1
2 sin α cos α 3
−sin α cos α ∴ tan θ=1
⇒ cos C=
2 sin α cos α
426 (b)
−1
⇒ cos C= ⇒ ∠ C=120° Since, the sides of a triangle are in AP
2
∴ 2b=a +c and c=7 cm
423 (b) 2
a +b −c
2 2
∵ cos C=
Let the distance of two consecutive stones are 2 ab
x , x +1 a + c +2 ac 2
2
2 2
a+ −c
−1 4
⇒ cos 120° = =
2 (a+ c)
2a
2
2 2 2 2
⇒−2 a ( a+ c )=4 a +a +c +2 ac−4 c
2 2 2
⇒−2 a −2 ac=5 a −3 c +2 ac
2 2
In ∆ BCD , ⇒ 7 a + 4 ac−3 c =0
2
h
tan60 ° = ⇒ x=
h ⇒ 7 a + 28 a−147=0 ( ∵ c=7 )
…(i)
x √3 2
⇒ a + 4 a−21=0
h ⇒ ( a+7 ) ( a−3 )=0
In∆ ABC , tan 30 °=
x+1 ⇒ a=3 and a ≠−7
1 h h ∵ b=5
⇒ = ⇒ + 1=√ 3 h ¿]
√ 3 x +1 √ 3 a+ b+c 3+5+7 15
Now, s= = =
√3
⇒ h= km 2 2 2
2 ∴ Area of triangle¿ √ s ( s−a )( s−b ) ( s−c )
424 (a)

Given, tan ϕ=0.5=


1
¿
√ 2 2(
15 15
−3
15
2 )(
−5
15
2
−7 )( )

2 15 9 5 1 15
10 ¿ ∙ ∙ ∙ = √ 3 c m2
In Δ ABC , tan θ= 2 2 2 2 4
AB
10 427 (a)
⇒ AB=
tan θ b+c a+c a+b
Given, = = =k [say]
D 11 12 13
20 m ⇒ b +c=11k , c +a=12 k , a+b=13 k
⇒ 2 ( a+ b+c )=36 k
C
⇒ a+b+ c=18 k
10 m ⇒ a=7 k , b=6 k , c=5 k
2 2 2
A B b + c −a
∴ cos A=
In Δ ABD , 2 bc
30 36+25−49 1
tan (θ+ ϕ )= ¿ =
AB 2 ×6 ×5 5
tan θ+ tan ϕ 30 tan θ
⇒ = 428 (c)
1−tan θ tan ϕ 10

P a g e | 112
y
B M A 3, 3 3 18+24 +30
Here, s= =36
2
30o Now, ∆=√ 36 ( 36−18 ) (36−24 ) (36−30 )
60o
x ∆=√ 36 ×18 ×12 ×6=216
O (0, 0)
So, radius of the incircle
Angle AOM is 30 ° . Hence, required point of B is
∆ 216
(−3 , 3 √3) r= = =6 cm
s 36
429 (b)
434 (b)
Let AB=BC=2 x ,
We know that the mid point of diagonals lies on
Then BD=DC=x and AD=( √ 5) x line y=2 x +c , here mid point is (3, 2), hence
c=−4

435 (b)
Clearly, ∆ OAB is an isosceles right angled
triangle. So, its orthocentre is at O(0 , 0) and the
circumcentre is the mid-point of AB having
x AD
Applying sine rule, = coordinates (a /2 ,a /2)
sin β sin 45 °

⇒ sin β=
x
√2 = 1
Hence, required distance ¿
√ a2 a2 a
+ =
4 4 √2
( √ 5 ) x √ 10 436 (d)
x 1 sin α √ 2 The lines of a triangle are x= y , x−2 y=3 and
and sin α= = ⇒ =
√ 5 x √ 5 sin β 1 x +2 y=−3. Intersection points at sides are
430 (c)
We know that, in a parallelogram diagonals cut
A (−3 ,−3 ) , B(−1 ,−1) and C 0 ,− ( 3
2 )
each other at middle point ∴ AB=√ 4+ 4=2 √2


9 3 √5
AC= 9+ =
4 2
and BC= 1+ =

1 √5
4 2

a+4 1+5 437 (b)


∴ = ⇒a=2
2 2 Let the vertices of a triangle be A ( 6 , 0 ) , B (0 , 6)
b+6 2+7 and C (6 , 6)
and = ⇒ b=3
2 2 Now, AB=√ 62 +6 2=6 √ 2

431 (c) BC + √ 62 +0=6


Assume that angles are 4 x , x and x And CA =√ 0+62=6
As 4 x+ x + x=180 ° [ ∴ ∠ A +∠ B+ ∠ C=180°] Also, A B2=BC 2 +C A2
⇒ x=30 ° Therefore, ∆ ABC is right angled atC . So, mid
∴ Angles are 120 ° ,30 °∧30 ° point of AB is the circumcentre of ∆ ABC
Ratio of sides ¿ sin A :sin B: sinC ∴ Coordinate of circumcentre are (3, 3)
¿ sin 120 °:sin 30 ° :sin 30 ° Coordinates of centroid are,
¿ √ 3 :1:1
∴ Required ratio ¿ √
3
= √
3
G ( 6+0+
3 3 )
6 0+6+ 6
, , ie ,(4 , 4)

1+ 1+ √ 3 2+ √ 3 ∴ Required distance ¿ √ ( 4−3 )2 + ( 4−3 )2=√ 2


433 (c) 438 (b)
P a g e | 113
( ) ( )
2 2 2 2 2 2
a +b −c a + c −b
3 ¿ ab −ac
Given that, a=5 , b=7 and sin A= 2 ab 2 ac
4 2 2 2 2 2 2
a +b −c a + c −b
sin A sin B ¿ −
As we know, = 2 2
a b 2 2
¿ b −c
3 sin B 21
⇒ = ⇒ sin B=
4×5 7 20 444 (b)
Which is not possible because its value is greater Since, D is the midpoint of BC . So, coordinate of
than one

440 (a)
D are ( x +2 x , y +2 y )
2 3 2 3

Since, coordinates of the centroid are Given, G(7 , 5) is the centroid of ∆ ABC

( )
x 1+ x 2 + x 3 y 1 + y 2 + y 3 2+ x 2+ x3 3+ y 2 + y 3
, , the centroid is always a ∴ 7= and 5=
3 3 3 3
rational point

441 (d)
∵ In a ∆ ABC ,
a b c
= = =2 R
sin A sin B sin {π−( A+ B)}
a b c ⇒ x 2 + x 3=21−2 and y 2 + y 3=15−3
⇒ = = =2 R
sin A sin B sin ( A+ B )
x 2+ x 3 19 y 2+ y 3
8. If we know a , sin A , sin B , then we can ⇒ = and =6
2 2 2
find b , c , A , B and C

9. We can find A , B ,C by using cosine rule


∴ Coordinates of D
19
2
,6 ( )
10. ∵ a , sin B , R are given, then we can find 445 (b)
sin A , b Since, the axes are rotated through an angle 45 ° ,
then we replace (x , y ) by
11. a , sin A , R are given, then we know only
the ratio
b
sin B
( x cos 45 °− y sin 45 ° , x sin 45°+ y cos 45 ° ) ie , ( √x2 − √y2 ,
in the given equation
c 2
3 x +3 y +2 xy=2
2
or ; we cannot determine the values of
sin ( A +B )
( ) ( ) ( )( )
2 2
x y x+ y x− y x + y
b , c , sin B , sin C separately ∴3 − +3 +2 =2
√2 √ 2 √2 √2 √2
∴ ∆ ABC cannot be determined in this case 3 2 3 2 2 2 2 2
⇒ ( x − y −2 xy ) + ( x + y +2 xy ) + ( x − y )=2
2
2 2 2
442 (b) 2 2
⇒ 4 x + 2 y =2
A−B+C π 2 2
⇒ 2 x + y =1
Since, A+C=π −B ⇒ = −B
2 2
∴ 2 ac sin ( A−B+
2
C
)=2 ac cos B 446 (c)
Let P ( x , y ) be any point on the line
2 2
c +a −b
2
Also, ( a 1−a2 ) x + ( b 1−b2 ) y +c=0 …(i)
¿ 2 ca . 2 2 2 2
2 ca Since, ( x−a1 ) + ( y−b 1 ) =( x−a2 ) + ( y−b 2 )
2 2 2
¿ a + c −b 2 2 2 2
⇒ x +a1 −2 a1 x + y +b1 −2b 1 y =x + a2−2 a2 x+ y + b2−2
2 2 2 2

2 2 2 2 2
443 (a) ⇒ 2 ( a 2−a1 ) x +2 ( b2−b 1 ) y=a2 +b 2+ b2−a1−b 1
a ( b cos C−c cos B )

P a g e | 114
2 2 2 2
(a2+ b2−a1−b 1)
⇒ ( a1−a 2 ) x + ( b 1−b2 ) y + =0 …
2
(ii)
Since, Eqs. (i) and (ii) represents the same
equation of line
2 2 2 2
a +b −a −b
∴ c= 2 2 1 1 In ∆ BCE ,using sine rule,
2
BE CE
=
447 (d) sin(180 °−3 α ) sin 2 α
Let D be the centre of circumcircle asin 2 α
⇒ CE = … (i)
∴ BD=5 cm sin 3 α
In ∆ ABC , h
Now, In ∆ DCE ,sin 3 α =
2
A C + A B + BC
2 2 CE
2
⇒ 100 = A B +36 h
⇒ sin 3 α = [¿ Eq .(i)]
2 a sin 2 α /sin3 α
⇒ A B =64 ⇒ AB=8
⇒ h=a sin 2 α
1
∴ Area of ∆ ABC= × AB × BC
2 450 (a)
1 2 We know that the x -axis divides the segment
¿ ×8 × 6=24 c m
2 joining P ( x 1 , y 1 ) and(x 2 , y 2 ) in the ratio − y 1 : y 2 .
A
So, the required ratio is −6 :−3 i.e. 2 :1
5
D
5
451 (d)
5
AB
In ∆ PAB , tan β=
B 6 C AP
B
448 (c)
The given equation is
2 2 C
x + 6 xy+ 8 y =10 …(i)
π
Since, axes are rotated through an angle
4 P A
π π x −y
∴ x=x 1 cos − y 1 sin = 1 1 AC
4 4 √2 In ∆ PAC , tan θ=
AP
π π x1 + y 1 ∴ tan α =tan ( β−θ )
and y=x 1 sin + y 1 cos =
4 4 √2 tan β−tan θ
on putting the value of x and y in Eq. (i) ¿
1+ tan β tan θ

( ) ( )( ) ( )
2
x 1− y 1 x 1+ y 1 x 1 + y 1 x1 + y1 AB AC
+6 +8 =10 −
√2 √2 √2 √2 ¿
AP AP
…(i)
⇒ x 1 + y 1−2 x 1 y 1 +6 x 1−6 y 1 +8 x 1+ 8 y1 +16 x 1 y1 =20 1+ AB ∙ AC
2 2 2 2 2 2

2 2 AP AP
⇒ 15 x 1+ 3 y 1+14 x1 y 1=20
∵ AP=n ( AB ) =n ( 2 AC ) (∵ C is the mid point
∴ Required equation is
2 2
of BA )
15 x +14 x y +3 y =20 From Eq. (i), we get
449 (c)
1 1

In ∆ ABE ,∠ BAE=∠ AEB
n 2n n
tan α = = 2
1
∴ AB=BE 1+ 2 2 n + 1
2n
⇒ n=( 2 n +1 ) tan α
2

P a g e | 115
452 (b)
Let the three points be A (−2 ,−5 ) , B ( 2,−2 ) and We have, a=2 x , b=2 y and ∠ C=120 °
C (8 , a ) 1
Area of triangle, ∆= ab sin C
If three points are collinear, then 2
sope of AB=¿ slope of BC 1
¿ ×2 x × 2 y sin 120 °
−2+5 a+2 2
⇒ =
2+2 8−2 ¿ xy √ 3 sq unit
3 a+2 5
⇒ = ⇒ 9=2 a+ 4 ⇒ a= 457 (a)
4 6 2
x −x y − y
( )
' ' ' '
a−a −a b−b −b
453 (a)
= ∵ 3 2= 3 2
'
a −a
'
b −b x2 −x1 y 2− y 1
From given relation, we can write ' '
a−2 a b−2 b
cos A+ cos C=2(1−cos B) ⇒ '
= '
a −a b −b
A+C A−C 2 B
⇒ 2 cos cos =2 ∙2 sin a b
2 2 2 ⇒ '= '
A−C B a b
⇒ cos =2 sin ' '
⇒ a b =a b
2 2
A +c A−C B B
⇒ 2 sin ∙ cos =4 sin cos 459 (c)
2 2 2 2
If two vertices of an equilateral triangle have the
⇒ sin A+ sinC=2sin B
coordinates (x 1 , y 1 ) and ( x 2 , y 2 ) , then the
⇒ a ,b , c are in AP
coordinates of its third vertex are
454 (d)
31
Given, cos ( A+ B )= ⇒ cos ( π −C )=
32
31
32
( x 1+ x 2 ± √ 3 ( y 1− y 2 ) y 1 + y 2 ± √ 3 ( x 1−x 2)
2
,
2 )
Here, we have
2 2 2
−a + b −c 31 x 1=2 , y 1=4 , x 2=2 and y 2=6
⇒−cos C= =
2 ab 32 Hence, the coordinates of the third vertex are
2 2 2
( 5 ) + ( 4 ) −c −31

2 ×5 × 4
2 −155
=
32 ( 4 ± √23 (−2) , 10± √23 ×0 )=(2 ∓ √ 3 ,5)
⇒ 41−c =
4 461 (a)
2 319 ∵ A +B +C+ D=2 π
⇒c =
4 ⇒ tan ( A + B+C+ D )=0
⇒ c=
√ 319 ∑ tan A−∑ tan A tan B tan C
2 ⇒ =0
1−∑ tan A tan B+tan A tan B tanC tan D
455 (a) ⇒ ∑ tan A−∑ tan A tan B tan C=0
We know, r 1=

,r 2 =

,r 3 =
∆ ⇒ ∑ tan A=tan A tan B tan C tan D ∑ cot A
s−a s−b s−c
∑ tan A = tan A
r >r
Given that, 1 2 3 >r ⇒ ∏
∆ ∆ ∆ ∑ cot A
⇒ > >
s−a s−b s−c 462 (a)
1 1 1 Let the angles of a triangle are 3 θ , 4 θ , 5 θ
⇒ > >
s−a s−b s−c ∴ 3θ +4 θ+ 5θ=180 ° ⇒ θ=15 °
⇒ s−a< s−b< s−c ∴ Angles of a triangle are 45 ° , 60 ° , 75 °
( s−a , s−b , s−c are positive) 1
⇒−a ←b←c Now, sin A=sin 45 °=
√2
⇒ a>b> c
sin B=sin 60 °=
√3
456 (b) 2

P a g e | 116
and sin C=sin 75 °=
√3+ 1
2 √2 Let D be the position of window of second house
∴ a :b :c=sin A :sin B :sin C and BC be the position of the first house.
1 √ 3 √ 3+1 AD
¿ : : In ∆ ADB , tan60 ° =
√2 2 2 √2 AB
¿ 2 : √ 6: √ 3+1 ⇒ AD=6 √ 3
2
and D B 2=( 6 √ 3 ) + ( 6 )2
463 (a) ⇒ DB=12 m
Since, AB= AC= √ 2 12
C In ∆ DCB ,sin 60 °=
h
B (3, 1)
o
15
45o
A
(2, 0)

The slope AB is 1. Hence, AB is inclined at 45 °


with the x -axis and AC is inclined at 60 ° with the
x -axis. Equation of AC is
12
y= √ 3( x−2) h= =8 √ 3 m
√3 /2
The coordinates of C is
(2+ √ 2 cos 60 ° ,0+ √ 2 sin 60 °) 466 (b)
Let the vertices be C , A , B respectively. The
¿ 2+( 1
,
√2 2
3
√) altitude from A is
y−a (t 2 +t 3)
=−t 1
464 (b) x −a t 2 t 3
Let ( x 1 , y 1 ) ,( x 2 , y 2) and (x 3 , y 3 ) are coordinates ⇒ x t 1 + y=a t 1 t 2 t 3 + a(t 2+ t 3 ) …(i)
of the points D , E and F , which divide each The altitude from B is
AB , BC and CA respectively in the ratio 3:1 x t 2 + y=a t 1 t 2 t 3 +a (t 3+t 1 ) …(ii)
(internally) Subtracting Eq. (ii) from Eq. (i), x=−a
3 ×6−1 ×1 17 Hence, y=a(t 1+ t 2 +t 3+ t 1 t 2 t 3 )
∴ x1= =
4 4 ∴ The orthocenter is {−a , a(t 1+t 2 +t 3 +t 1 t 2 t 3 )}
−2 ×3+ 4 ×1 −2 −1
y 1= = =
4 4 2 467 (a)
A(-1, 4) BC
In ∆ ABC , tan 30 °=
40
(x1, y1)D F(x3, y3)
40
⇒ BC = …(i)
(6, -2)B E(x2, y2) C(-2, 4) √3
D
5
Similarly, x 2=0 , y 2=
2
−5
and x 3= , y 3=4 C
4
Let ( x , y ) be the coordinates of centroid of ∆≝¿ 60o
∴ x=
3 4(
1 17 5
+0− =1
4 ) A
30o
40 m B

and y= (
1 −1 5
+ + 4 =2
3 2 2 ) In ∆ DAB , tan 60 °=
BD
40
∴ Coordinates of Centroid are (1, 2) ⇒ BD=40 √ 3
465 (a)
⇒ DC + BC =40 √ 3

P a g e | 117
40 Thus, A+ B=60°
⇒ DC + =40 √ 3[from Eq.(i)]
√3 and A+ B+C=180 ° ⇒ ∠ C=120°

⇒ DC =40 √ 3−
( 1
√3 √3
=
)
80
471 (b)
b +c −a
2 2 2
⇒ DC =46.19 m We have, cos A=
2 bc
468 (c) ⇒ b −2 bc cos A + ( c −a2 )=0
2 2

Let DC be the height of building It is given that b 1 and b 2 are roots of this equation
∴ b1 +b2 =2 c cos A and b 1 b 2=c 2−a2
⇒ 3 b 1=2 c cos A and
2 2 2
2 b1=c −a [ ∵ b2=2 b1 (given)]

( )
2
2c 2 2
⇒2 cos A =c −a
3
∴ AB=Speed ×Time
¿ 25 ( √ 3−1 ) .2 ⇒8 c2 ( 1−sin 2 A ) =9 c 2−9 a 2

( )
2 2
¿ 50( √ 3−1) 29 c −9 a
⇒ sin A= 1− 2
DC 8c
∴ ∈∆ DBC , tan 45 °= ⇒ BC =h


BC 9 a2−c 2
h ⇒ sin A= 2
In ∆ DAC , tan 30 °= 8c
50 ( √ 3−1 ) +h
⇒ 50 ( √ 3−1 )+ h= √3 h 472 (a)
⇒ h=50 m Let the vertices of the ∆ ABC are
A ( 7 , 1 ) , B(−1 , 5) and C (3+2 √ 3 ,3+ 4 √ 3)
469 (d)

Now, AB= ( 7+1 )2 + ( 1−5 )2
A
( b+ c ) tan tan
2 ( B−C
2 ) ¿ √ 64+16=√ 80

A (b−c) A √
BC= (−1−3−2 √ 3 ) + ( 5−3−4 √ 3 )
2 2

¿ ( b+ c ) tan . cot ¿ √ 16+12+16 √ 3+4 +48−16 √ 3


2 (b+c ) 2
¿ b−c ¿ √ 80

and CA= ( 3+2 √ 3−7 ) + ( 3+ 4 √ 3−1 )
2 2

∴ ∑ (b +c) tan ( B−C


2 )
tan
A
2 ¿ √ 16+12−16 √ 3+ 4+ 48+16 √ 3
¿ b−c +c−a+a−b=0 ¿ √ 80
Here, AB=BC=CA=√ 80
470 (c)
∴ Hence, it is an equilateral triangle, so incentre
As, A> B and 3 sin x−4 sin3 x −k=0 , 0<k <1
and centriod coincides
⇒ sin 3 x=k
As A and B satisfy given equation
∴ sin 3 A=k , sin 3 B=k
So, incentre ¿( 7−1+3+2
3
√ 3 , 1+5+ 3+4 √3
3 )
¿(
3 )
⇒ sin 3 A−sin 3 B=k 9+ 2 √ 3 9+ 4 √ 3
,
3
⇒ 2 cos (
3 A+ 3 B
) (
sin
3 A−3 B
=0)
2 2
( √23 √43 )
¿ 3+ ,3+
⇒ cos ( 3 A +32 B )=0∨sin ( 3 A−3
2
B
)=0 473 (b)
3 A +3 B 3 A−3 B
⇒ =90 °∨ =0
2 2
⇒ A+ B=60 °∨ A=B
But given, A> B( ∴ neglectring A=B)

P a g e | 118
√ √
( s−b ) (s−c ) ( s−a ) (s−c) B+ C B−C
A B − 2sin cos
tan −tan 2 2
2 2 s ( s−a ) s (s−b) ¿
= A A

√ √
A B ( s−b ) (s−c ) ( s−a ) (s−c) 2 sin cos
tan + tan + 2 2
2 2 s ( s−a ) s(s−b) A B−C
⇒ cos =cos
( s−b ) √ s(s−c )−( s−a ) √ s(s−c ) 2 2
¿
( s−b ) √ s(s−c )+ ( s−a ) √ s(s−c ) A B−C
∴ =
2 2
¿√
s (s−c ) ( s−b−s +a ) a−b
= π
√ s (s−c )( s−b−s +a ) c ⇒ A+ C=B ⇒ ∠ B= ( ∵ A+ B+C=π )
2
474 (a)
As we know that orthocenter, centroid and 478 (c)
circumcentre are collinear and the centroid Given lines are
2 2
divides the line segment joining orthocenter and 3 x −4 xy + y =0
circumcentre in the ratio 2:1. If the coordinates of ⇒ ( 3 x− y ) ( x− y )=0
orthocentre and circumcentre are (1, 1) and (3, 2) ⇒ 3 x− y =0 , x − y=0 and 2 x− y=6
respectively, the coordinate of centroid is The points of intersection of these lines are

(
2.3+1.1 2.2+1.1
,
7 5
)=( 3 , 3 ) ( 0 , 0 ) ,(−6 ,−18) and (6, 6)

| |
2+1 2+1 0 0 1
1
∴ Area of triangle ¿ −6 −18 1
475 (d) 2
6 6 1
2 2 2
b +c −a 1 1
∵ ∠ A=60 ° , cos 60 °= ¿ (−36+108 )= ( 72 )
2 bc 2 2
2 2 2
⇒ b + c −a =bc ¿ 36 sq units
Now, A B2+ A C2=2( A D 2+ B D2)
479 (d)
()
2
2 2 2 a
⇒ c +b =2 A D +2 h
2 In ∆ ABC , tan 60 °= ⇒ h √ 3 x …(i)
2 2 2 2
x
⇒ 2 b +2 c −a =4 A D
2 2 2 2 2 2
⇒ b + c + bc=4 A D ( ∵ b + c −a =bc )

476 (b)
The intersection of lines are ( 0 , 0 ) ,(−4 ,−8) and
(−2 ,−2)
and in ∆ ABD ,
Let circumcentre is (x 1 , y 1 )
h
∴ x21 + y 21=( x1 + 4 )2 + ( y1 +8 )2 tan30 °= ⇒ x+60=√ 3 h …(ii)
x +60
⇒ 8 x 1 +16 y 1+ 80=0 …(i) From Eqs. (i) and (ii),
2 2
And x + y =( x 1+ 2 ) + ( y 1 +2 )
1 1
2 2
h=√ 3× 30=51.96 m
⇒ 4 x 1+ 4 y 1 +8=0 …(ii) =52m (approx)
On solving Eqs. (i) and (ii), we get 480 (d)
x 1=6 and y 1=−8 OP is inclined at angle θ with x -axis and OQ is
inclined at angle α −θ with x -axis
477 (c) y
A b+c Q
Given, cot =
2 a P
A -2
cos
2 sin B+sin C x
⇒ = O
A sin A The bisector of angle POQ is inclined at angle
sin
2

P a g e | 119
θ+α −θ α 481 (b)
= with x -axis
2 2
The altitude h is ¿ 2−1−2∨
¿ = 1 ¿
√2 √2
A (2, -1)

h
o
60
B
Dx + y = 2 C

In ∆ ABC , BC =2 h cot 60 °=

h
√ 2
3
∴ Area of ∆= BC
2

¿
12 √3
=
2√ 2 3 6 √
483 (d)
Let OP be the tower. Since, the tower make equal
angles at the vertices of the triangle, therefore
foot of tower is at the circumcentre
P
A

O
R R
B C

OP
In ∆ OAP , tan α =
OA
⇒ OP=OA tan α
⇒OP=R tan α ( ∵ OA =R , given)

484 (a)
Given, xy +2 x+ 2 y + 4=0 …(i)
and x + y +2=0 …(ii)
From Eqs. (i) and (ii),
xy=0
⇒ x= y=0
y

(-2, 0) O
x' x

(-1, -1) (0, -2)

y'
∴ vertices of triangle are (−2 , 0 ) , ( 0 , 0 ) ,(0 ,−2)
Since, this triangle is right angled triangle and in a
right angled triangle circumcentre is mid point of

P a g e | 120
hypotenuse. 485 (b)
∴ (−1 ,−1) is the circumcentre Let P be (x , y ) and we have
A=( a+b , a−b ) , B=( a−b , a+b )
Here, PA=PB ⇒ P A2=P B2
⇒ [ x− ( a+b ) ]+ [ y−( a−b ) ]
2 2

2 2
¿ [ x−( a−b ) ] + [ y −( a−b ) ]
2 2
⇒ [ x−( a+b ) ] −[ x−( a−b ) ]
2 2
¿ [ y− ( a+b ) ] −[ y−( a−b ) ]
⇒ [x −( a+b )+ x−(a−b)][x− ( a+b )−x+ a−b]
¿ [ y −( a+b )+ y−(a−b)][ y−( a+ b )− y+ a−b]
⇒ ( 2 x−2 a ) (−2 b ) =( 2 y −2 a ) (−2 b )
⇒ x− y=0

486 (c)
a
Since, R=
2 sin A
2 √3
⇒ R= =2 cm
2 sin 60 °

487 (c)
Let two of vertices of a triangle are A(15 ,0) and
B(0 , 10) and third vertex is C (h , k )
We know that line passing through A and B
should be perpendicular to line through C and
orthocenter O

∴ ( −23 )( 9−k
6−h )
=−1

⇒ 2 k =3 h
Which is satisfied by (0, 0). Hence, coordinates of
third vertex are (0, 0)

488 (a)
Let P and Q be the positions of two planes. It is
given that OP=300 m . From triangle OAQ , we
have OA=OQ .
From ∆ OAP , we have
OP 300 300
tan60 ° = ⇒ √ 3= ⇒ OQ= =100 √ 3 mts
OA OQ √3

489 (b)

P a g e | 121
Let the sides be a=3 x , b=7 x ,c=−8 x . Then, 492 (c)
2 s=a+b+ c ⇒ s=9 x ∆=2 bc−( b 2+ c 2−a2 ) =2 bc ( 1−cos A )
∴ ∆=√ s ( s−a ) ( s−b ) (s−c) 2 A
¿ 2 bc .2 sin …(i)
¿ √ 9 x × 6 x ×2 x × x=6 √ 3 x 2
2

abc ∆ 1 1 A A
Now, R= and r = But ∆= bc sin A= bc .2 sin cos
4∆ s 2 2 2 2
R sabc 9 x × 3 x ×7 x ×8 x 7 A A
∴ = = = ⇒ ∆=bc sin cos …(ii)
r 4 ∆2 4 ×108 × x
2
2 2 2
On dividing Eq. (ii) by Eq. (i), we get
490 (a) A 1
tan =
A A 2 4
( b+ c−a ) tan =( 2 s−2 a ) tan
2 2 A 1
2 tan

√ ( s−b ) ( s−c) 2 2 8
¿ 2(s−a) ∴ tan A= = =
s (s−a) A 1 15
1−tan2 1−
2 16
¿2
√ s ( s−a ) ( s−b )( s−c ) = 2 ∆
s s 493 (d)
The vertices of a ∆ ABC are A ( 0 , b ) , B ( 0 , 0 ) and
491 (b)
Let AB=a C (a , 0)
ON ⊥ AB A (0, b)

and AN =BN
E
O
B(0, 0) D C(a, 0)

( a2 , b2 ) and ( a2 , 0)
n
A a/2 N a/ 2 B Mid point of E and D are

π AN b
In ∆ AON , tan = The slope of median BE ,m 1=
n ON a
π a π −2 b
⇒ ON = AN cot = cot and slope of AD , m 2=
n 2 n a
π AN Since, the medians are perpendicular to each
and sin =
n OA other
a π ∴ m1 m2=−1
⇒ OA = cosec …(ii)
2 n
Now, sum of the radii ¿ ON +OA
b −2 b
⇒ ×
a a ( )
=−1
a π a π
⇒−2 b =−a ⇒ a=± √ 2 b
2 2
¿ cot + cosec
2 n 2 n
[from Eqs. (i) and (ii)] 494 (c)

[ ]
π ∵ a ( b−c )+ b ( c−a ) +c ( a−b )=0
1+cos
a n ∴ x=1 is a root of the equation
¿
2 π 2
a ( b−c ) x +b ( c−a ) x +c ( a−b )=0
sin
n Then, other root =1 (∵ roots are equal)

[ ]
π c (a−b)
2cos 2 ∴ Product of roots ¿ 1=
a 2n a (b−c)
¿
n π π ⇒ ab−ac=ca−bc
2 sin cos
2n 2n 2 ac
⇒ b=
a π a+ c
¿ cot ∴ a , b , c are in HP
2 2n

P a g e | 122
1 1 1 s−a s−b s−c
Then, , , are in AP ⇒ > >
a b c ∆ ∆ ∆
s s s ⇒ ( s−a ) > ( s−b )>(s−c)
⇒ , , are in AP
a b c ⇒−a>−b>−c
s s s ⇒ a<b< c
⇒ −1 , −1 , −1 are in AP
a b c
( s−a) (s−b) (s−c) 498 (a)
⇒ , , are in AP Let l be the length of the ladder
a b c
abc ie , BP=CQ=1
Multiplying each by PA AB
( s−a )( s−b ) (s−c ) In ∆ PAB , cos α = ∧sin α=
bc ca ab PB PB
Then , , ⇒ PA=l cos α ∧ AB=l sin α …(i)
( s−b ) (s−c) (s−c) ( s−a ) ( s−a )( s−b )
AQ
are in AP In ∆ QAC , cos β=
QC
( s−b ) (s−c) (s−c ) ( s−a ) ( s−a ) ( s−b )
⇒ , , are in ⇒ AQ=l cos β∧ AC =lsin β ...(ii)
bc ca ab
HP

⇒ sin 2 ( A2 ) ,sin ( B2 ) , sin ( C2 ) are in HP


2 2

495 (d)
1+ cos C cos ( A−B ) 1−cos ( A+ B ) cos ( A−B)
= ( ∵ A+ Now,
B+C=π CB=) AB−CA
1+ cos ( A−C ) cos B 1−cos ( A−C ) cos ( A+C )
2 2 2 2 ¿ l sin α −lsin β [from Eqs. (i) and (ii)]
1−cos A +sin B sin A +sin B
⇒ 2 2
= 2 2 ¿ l(sin α −sin β )
1−cos A+ sin C sin A +sin C
2 2
and QP= AQ−PA
a +b ¿ l cos β−l cos α
¿ 2 2
a +c ¿ l(cos β −cos α )
496 (c) CB
∴ =l¿ ¿
2
∵ b +c =a 2 2 QP

⇒ =
y
2sin
α− β
2 ( ) ( )
cos
α+ β
2
x
2sin
α+β
2 ( ) ( )
cos
α− β
2

⇒ =cot (
2 )
y α +β
x
∴ △ ABC is right angled triangle and right
⇒ x= y tan (
2 )
angled at A
α+β
Clearly, from figure,
1 1 499 (c)
( AB )( AC ) = ( AD ) ( BC )
2 2
∵ A ( h . k ) , B(1 , 1) and C (2 ,1) are the vertices of
( AB ) (AC )
⇒ AD= a right angled triangle ABC
BC
5∙ 12 60
¿ =
13 13

497 (a)
Since, r 1 <r 2 <r 3
1 1 1
⇒ > >
r1 r2 r3

P a g e | 123

Now, AB= ( 1−h )2+ ( 1−k )2 ⇒ a ,b , c are in AP

Or BC= ( 2−1 )2+ (1−1 )2=1 502 (a)
Or CA=√ ( h−2 ) + ( k −1 )
2 2
45 √ 3 /2
In ∆ ACD , tan30 °=
Now, Pythagorus theorem x
2 2 2
A C =A B + B C
2 2 2 2
⇒ 4+h −4 h+k +1−2 k=h +1−2h+ k +1−2 k +1
⇒ 5−4 h=3−2 h
⇒ h=1 …(i)
Now, given that area of the triangle is 1
1 45 √ 3 135
Then, area ( ∆ ABC )= × AB × BC ⇒ x= = m
2 1 2
1 2×
⇒ 1= × √ ( 1−h ) + ( 1−k ) × 1 √3
2 2
2 and in ∆ ABD ,
⇒2=√ (1−h ) + ( 1−k ) …(ii)
2 2
45 √3 /2 45
tan60 ° = ⇒ y=
On putting h=1 from Eq. (i), we get y 2
2= √ ( k−1 ) 135 45
2
∴ x− y= − 45 m
On squaring both the sides, we get 2 2
2
4=k +1−2 k 503 (d)
2
⇒ k −2 k−3=0 ⇒ k=−1 , 3 For point ( 1 , 3 ) ,3 x +2 y=3+6> 0
Thus, the set of values of k is {−1 , 3 } For point ( 5 , 0 ) , 3× 5+0>0
500 (d) and for point (−1 , 2 ) ,−3+ 4>0
Let the coordinate of mid point of AB is (x 1 , y 1 ) Similarly, other inequalities also hold
Hence, option (d) is correct

504 (c)
Given that, x 1=x , x 2=1 , x3 =0
and y 1=0 , y 2=1 , y 3=2
∴ Area of triangle
1
¿ [ x ( y − y 3 ) + x 2 ( y 3− y 1) + x3 ( y 1− y 2 ) ]
2 1 2
a+ 0 0+b
∴ x1= , y 1= 1
2 2 ¿ [ x (1−2 ) +1 (2−0 ) +0 ( 0−1 ) ]
⇒ a=2 x 1 , b=2 y 1 2
1 1
Given, a+ b=4 ⇒ x 1 + y 1=2 ¿ [−x +2+0]= (2−x)
2 2
Hence, the locus of the mid point is x + y=2 But area of triangle is 4 sq unit
501 (a) 1
∴ ( 2−x )=4
A B C 2
∵ tan , tan , tan are in HP ⇒ 2−x=8 ⇒ x=−6
2 2 2
A B C 505 (c)
∴ cot , cot ,cot are in AP
2 2 2 Given vertices of triangle are
B A C B
⇒ cot −cot =cot −cot O ( 0 , 0 ) , A (cos θ , sin θ) and B(sin θ ,−cos θ),
2 2 2 2
then coordinates of Centroid are
s ( s−b) s (s−a) s (s−c) s ( s−b)




⇒ s−b−s +a=s−c−s +b
=


∆ ( cos θ+3 sinθ , sin θ−cos
3
θ
)
Since, Centroid lies on the line y=2 x
⇒ 2 b=a+ c

P a g e | 124
sin θ−cos θ 2 cos θ+2 sin θ 2
4 P A =9 P B
2
∴ =
3 3 2¿
⇒ 4 ( h + k )=9 ( h−4 ) +9 ( k +3 )¿
2 2 2
⇒ sin θ=−3 cos θ
−1
⇒ θ=tan (−3) ⇒ 4 h2 + 4 k 2=9 ( h2 +16−8 h ) +9 ( k 2+ 9+6 k )
2 2
⇒ 5 h +5 k −72h+ 54 k +225=0
506 (a) ∴Required locus of P(h , k ) is
In order to remove first degree terms from the 2 2
5 x +5 y −72 x +54 y +225=0
equation a x 2 +2 h xy + b y 2+ 2 gx +2 fy+ c=0 the
origin is shifted at (−g /a ,−f /a) 510 (a)
In the equation 2 x 2+7 y 2 +8 x−14 y+ 4=0 , we The coordinates of the orthocentre O ' and
have
a=2 , b=7 , g=4 and f =−7
circumcentre O are (2 , 1) and ( 72 , 52 ) respectively
Hence, the coordinates of the required point are We know that the centroid G divides OO ' in the
(−4/2 ,−7 /7)=(−2 ,1) ratio 1 :2
So, the coordinates of G are
507 (b)

( )
7 5
Let A [ a t 1 t 2 , a ( t 1 +t 2 ) ] , B[a t 2 t 3 , a(t 2+ t 3 )], 1× 2+2× 1 ×1+2 ×
2 2
, ≡(3 , 2)
C [a t 3 t 1 , a(t 3 +t 1)] 1+ 2 1+ 2
a(t 3−t 1) 1
Slope of AB ( mAB ) = = 511 (a)
a t 2 (t 3−t 1) t 2
PR h
Equation of line through C perpendicular to AB is ∆ POR , =
In sin 90 ° 1 …(i)
y−a ( t 3 +t 1 )=−t 2 (x−at 3 t 1 ) sin67 °
2
⇒ y −a ( t 3+ t 1 )=−t 2 x+ a t 1 t 2 t 3 …(i)
Similarly, equation of line through B
perpendicular to CA is
y−a ( t 2 +t 3 )=−t 1 ( x−a t 2 t 3 )
⇒ y −a ( t 2+ t 3 ) =−t 1 x+ a t 1 t 2 t 3 …(ii)
Using t 1 t 2 t 3=−(t 1 +t 2+ t 3 ) in Eqs. (i) and (ii), we
get And in ∆ PQR
y=−t 2 x−a t 2 PR b
= …(ii)
1 sin 45 °
and y=−t 1 x−a t 1 sin 22 °
2
⇒ t 2 ( x +a )=t 1 (x+ a) From Eqs. (i) and (ii)
⇒ x=−a , y=0 1 1
sin 22 ° sin 45 °
sin 45 ° h 2 h 2
508 (d) = ⇒ =
1 sin 90° b sin 45 °
Let the coordinates of the third vertex C be(h , k ). sin 67 ° b
2
Then,
⇒ 2 h=b

| |
h k 1
1
−5 0 1 =± 20 ⇒ k =± 5 512 (b)
2
3 0 1 2
a +b −c
2 2
Using, cos C=
Since, ( h , k ) lies on x− y =2. Therefore, h−k =2 2 ab
Fork =5 , h=7 . For k =−5 , h=−3
Hence, the coordinates of the third vertex C are
(−3 ,−5) or (7, 5)

509 (b)
Let P ( h , k ) be the required point, then

P a g e | 125
| |
A a1 b 1 1
a2 b 2 1 =0, so lines are concurrent

2x
-1 a3 b 3 1
2

+1
x
b=

=c
30 o 516 (c)
C 2 B
a=x +x+1 ( b+ c ) cos A + ( c +a ) cos B+ ( a+b ) cos C
2 2 ¿ b cos A +c cos A +c cos B+a cos B+ a cos C+ b cos C
√ 3 ( x + x +1 ) + ( x −1 ) − (2 x +1 )
2 2 2
⇒ = ¿ ( b cos A +a cos B ) + ( c cos A+ acos C )+(c cos B+b cos C )
2 2 ( x 2 + x+ 1 ) ( x 2−1)
2
¿ c +b+ a
⇒ ( x +2 ) ( x+1 )( x−1 ) x+ ( x −1 )
2

517 (b)
¿ √ 3 ( x 2+ x +1 ) (x 2−1)
sin A sin B
⇒ x 2 +2 x + ( x 2−1 ) =√ 3( x 2 + x+1) By sine rule, =
a b
⇒ 2 ( 2− √ 3 ) x + ( 2− √3 ) x−( √ 3+1 ) =0
2

⇒ x=− ( 2+ √ 3 ) and x=1+ √ 3 Here,


sin ( π2 + B ) = sin B [by sine rule]
But, x=−( 2+ √ 3 ) ⇒ c is negative 5 4
∴ x=1+ √3 is the only solution 4
⇒ tan B=
5
513 (d) Also, ∠ A+∠ B+∠ C=π
It is given that the distance between the points π
P(a cos 48 ° , 0) and Q(0 , a cos 12° ) is d ⇒ +2 ∠ B+∠ C=π
2
i.e., PQ=d
⇒ P Q =d
2 2
2 2

2 2 2
⇒ 2 tan−1
4
5()+∠ C=
π
2
⇒ a cos 48 °+ a cos 12° =d
2 2
⇒ a ( 1+cos 96 ° ) +a ( 1+cos 24 ° ) =2 d
2
π
⇒ ∠ C= −2 tan −1
2
4
5 ()

( )
2 2 2
⇒ 2 a + a ( cos 96 ° +cos 24 ° ) =2 d 8
2 2 2
⇒ 2 a +2 a cos 60 ° cos 36 °=2 d π −1 5
⇒ ∠ C= −tan
(
2 √ 5+1
) 2 16
2
a
=2 d ⇒ d −a = ( √ 5+1) 1−
2 2 2 2
⇒2a +a 25
4 8

514 (b)
ab cos C−ac cos B
π
⇒= −tan
2
−1 40
9
=cot( )
−1 40
9 ( )
¿
2 2 2 2
a +b −c c +a −b
2

2
2 2
⇒ ∠ C=tan−1 ( 409 )
2 2 2 2 2 2
a +b −c −c −a + b 518 (b)
¿
2 1 1 1
2 2 We have, a p1=∆ , b p 2=∆ , c p3 =∆
¿ b −c 2 2 2
Where a , b , c are the sides of a triangle
515 (a) 2∆ 2∆ 2∆

| |
a1 b 1 1 ⇒ p1 = , p 2= , p3 =
a b c
For collinearity, a2 b 2 1 =0 1 1 1 a
2
b
2
c
2 2 2
a +b + c
2

a3 b 3 1 ∴ 2+ 2 + 2= 2+ 2+ 2+ 2
p1 p2 p3 4 ∆ 4 ∆ 4 ∆ 4∆
For concurrency to lines

P a g e | 126

You might also like